Gastro-intestinal System Flashcards

1
Q

A 68-year-old man undergoes barium swallow for dysphagia. During the examination the
patient has an episode of coughing, and barium is noted to enter the larynx and proximal trachea.
What is the appropriate management? [B4 Q62]

a. no action needed
b. physiotherapy
c. prophylactic antibiotics
d. chest radiograph in 48 hours
e. admission to hospital for observation

A

Physiotherapy

Barium aspiration is a recognized complication of barium swallow and may occur particularly in patients with swallowing disorders or recent oesophageal surgery. It is usually clinically insignificant, but complications have been reported, especially with aspiration of larger amounts of barium, and include pneumonitis and granuloma formation. Physiotherapy is the only treatment recommended. Of the water-soluble contrast agents, Gastrografin (ionic and hyperosmolar) may cause pulmonary oedema if aspirated. Gastromiro (non-ionic and iso-osmolar) is safe to use if aspiration is a significant possibility.

How well did you know this?
1
Not at all
2
3
4
5
Perfectly
2
Q

A 32-year-old female patient attends for a barium swallow with a history of a sensation of food
sticking in her throat. The barium swallow reveals (1) uniform horizontally orientated folds in
the lower oesophagus. There is (2) a change in the texture of the mucosa 1 cm above the hiatus,
which is (3) sited 25 cm from the origin of the oesophagus. There is a (3) slight smooth
narrowing noted 2 cm above the hiatus, beyond which there is a (4) slight dilatation of the
oesophagus prior to it joining the stomach. Which of the following is an unusual finding? [B1
Q2]

A. The appearance of the oesophageal folds.
B. The change in mucosal appearance 1 cm above the hiatus.
C. The distance of the hiatus from the origin of the oesophagus.
D. The slight narrowing 2 cm above the hiatus.
E. The distal bulge just before the stomach.

A

The appearance of the oesophageal folds

The oesophageal folds are normally longitudinally orientated. Horizontally orientated folds are described as feline oesophagus.
The change in mucosal appearance is the normal Z line – the squamo-columnar junction. The narrowing described is the A line at the origin of the vestibule of the distal oesophagus. The position of the hiatus is normally stated as being 40 cm. This is the distance from the teeth at gastroscopy – the distance from the origin of the oesophagus is 25 cm. The three anatomic rings of the distal oesophagus are the A (muscular), B (mucosal), and C (diaphragmatic impression) rings.
[Core Radiology]
Z-lines – squamocolumnar junction
A-lines – narrowing at the origin or the vestibule of the distal oesophagus

How well did you know this?
1
Not at all
2
3
4
5
Perfectly
3
Q

Which structure marks the transition from squamous oesophageal to columnar gastric epithelium? [B4 Q71]

a. A-ring
b. B-ring
c. Z-line
d. oesophageal vestibule
e. gastro-oesophageal junction

A

The transition from squamous oesophageal epithelium to columnar gastric epithelium is
marked by the Z-line
, an irregular zigzag line. It is not a reliable indicator of the gastro-
oesophageal junction, however, and may lie some distance above it if there is columnar
transformation of the distal oesophagus, as seen in Barrett’s oesophagus. The
gastrooesophageal junction may be identified by a thin, shelf-like ring known as the B-ring. It
is visible on barium swallow only when the gastro-oesophageal junction lies above the
diaphragmatic hiatus. Approximately 2–4cm above this is a thicker ring produced by active
muscle contraction known as the A-ring. The oesophageal vestibule is the saccular termination
of the lower oesophagus, which lies between the A-ring and the B-ring, and corresponds with
the lower oesophageal sphincter

How well did you know this?
1
Not at all
2
3
4
5
Perfectly
4
Q

A 45-year-old woman is referred by her GP for a barium swallow for investigation of dysphagia.
Gastro-oesophageal reflux into the lower third of the oesophagus is demonstrated and delicate
transverse striations in the lower oesophagus are observed as a transient phenomenon
. What is
the next appropriate action appropriate for the radiologist? [B1 Q39]

A. Recommend a staging CT of chest and abdomen.
B. Recommend oesophagoscope and biopsy of the affected area.
C. Recommend to the GP that the study was unremarkable but for mild reflux.
D. Recommend referral for manometry.
E. Recommend endoscopic ultrasound.

A

Recommend to the GP that the study was unremarkable but for mild reflux.

The findings described are in keeping with a ‘feline’ oesophagus. This is thought to be due to spasm in the muscularis mucosa. It is associated with gastro-oesophageal reflux but is a benign entity

How well did you know this?
1
Not at all
2
3
4
5
Perfectly
5
Q

A neonate is diagnosed with congenital tracheoesophageal (TE) fistula. A plain film
demonstrates a gasless abdomen. Which type of TE fistula is associated with this finding? [B2
Q3]
a. Type B
b. Type C
c. Type D
d. Type E
e. None of the above

A

Type B

Congenital TE fistula and oesophageal atresia occur in approximately 1 in 4000 live births.
They are divided into five subtypes, A to E. Type C is the most common, comprising 75% of
all types and involves oesophageal atresia with a distal TE fistula. Type D involves
oesophageal atresia with both proximal and distal TE fistula, and type E is a TE fistula without
oesophageal atresia. Therefore, types C to E do not typically present with gasless abdomen.
Type B is oesophageal atresia with a proximal TE fistula; there is no communication between
the trachea and the distal oesophagus, and therefore a gasless abdomen is typical. Type A is
oesophageal atresia without TE fistula and therefore may also present with a gasless abdomen
but is not a listed option.
TE fistula classification [STATdx]
Type A: Oesophageal atresia with no TEF (7-9%)
Type B: Oesophageal atresia with proximal TEF (1%)
Type C: Oesophageal atresia with distal TEF (82-86%)
Type D: Oesophageal atresia with proximal & distal TEF (2%)
Type E: TEF without Oesophageal atresia (4-6%)

How well did you know this?
1
Not at all
2
3
4
5
Perfectly
6
Q

A 45-year-old man presents with dysphagia and undergoes a double-contrast barium swallow. This demonstrates a smooth oblique indentation on the posterior wall of the oesophagus. What is the most likely cause of these appearances? [B4 Q37]
a. enlarged left atrium
b. aberrant right subclavian artery
c. aberrant left pulmonary artery
d. right-sided aortic arch
e. coarctation of the aorta

A

Aberrant right subclavian artery
Several anomalies of the major vessels can cause extrinsic impressions upon the oesophagus.
The commonest aortic anomaly is a right-sided aortic arch, which produces an indentation on
the right lateral oesophageal wall in the absence of the normal left aortic arch impression. An
aberrant right subclavian artery originates from the aortic arch just distal to the left subclavian
artery, and passes upwards and to the right, behind the oesophagus, giving rise to an oblique
posterior oesophageal indentation. In aortic coarctation, the preand post-stenotic dilatations of
the aorta produce a characteristic reversed-3 impression upon the left wall of the oesophagus.
An enlarged left atrium and an aberrant left pulmonary artery both cause anterior indentations
upon the oesophagu

How well did you know this?
1
Not at all
2
3
4
5
Perfectly
7
Q

A 76-year-old woman presents with dysphagia and regurgitation of undigested food. She
undergoes barium swallow, which demonstrates a barium-filled pouch extending from the
posterior oesophageal wall at the level of C5–6 that is causing oesophageal compression. What
is the most likely diagnosis? [B4 Q61]
a. intramural pseudodiverticulum
b. epiphrenic diverticulum
c. lateral pharyngeal diverticulum
d. interbronchial diverticulum
e. Zenker’s diverticulum

A

Zenker’s diverticulum
A Zenker diverticulum is a herniation of the mucosa and submucosa through the midline of the
posterior oesophageal wall at the cleavage plane between the oblique and transverse fibres of
crico-pharyngeus (Killian’s dehiscence) at the level of C5–6. The diverticulum is narrow
necked and extends caudally, resulting in trapping of undigested food and compression of the
adjacent oesophagus. Epiphrenic diverticula are rare, usually occurring on the right lateral wall
of the distal oesophagus
in association with hiatus hernia. Lateral pharyngeal diverticula are
herniations of pharyngeal mucosa through the lateral pharyngeal wall, which occur most
commonly in wind instrument players, reflecting increased intra-pharyngeal pressure. An inter-
bronchial diverticulum is a traction diverticulum that occurs in the inter-bronchial segment of
the oesophagus in response to adjacent fibrous adhesions following lymph-node infection
(usually tuberculous). Intramural pseudodiverticula represent dilated excretory ducts of
mucosal glands, which appear as multiple flask-shaped outpouchings and are commonly seen
in association with candidiasis

How well did you know this?
1
Not at all
2
3
4
5
Perfectly
8
Q

A 51-year-old male patient has a barium swallow for the investigation of dysphagia. This
shows a 10 cm tapered stricture in the mid oesophagus along with multiple fine linear
projections perpendicular to the lumen
, each 3–4mm long, in this segment. There are
occasional tertiary contractions and mild gastro-oesophageal reflux. What is the most likely
diagnosis? [B2 Q50]
a. Chagas disease
b. Oesophageal intramural pseudodiverticulosis
c. Oesophageal varices
d. Cytomegalovirus infection
e. Oesophageal carcinoma

A

Oesophageal intramural pseudodiverticulosis
Oesophageal intramural diverticulosis relates to
dilated excretory ducts of the deep mucous glands
of the oesophagus. They are best demonstrated on
barium swallow and have the classical
appearance as described in the question. The
pseudo-diverticular can appear to float outside
the oesophagus when no communication with the
lumen is seen. Most patients have dysphagia at
presentation and associated conditions include
diabetes, candida infection, oesophagitis,
stricture, and alcohol abuse

Case courtesy of Dr Andres Turchetti,
Radiopaedia.org, rID: 12848

How well did you know this?
1
Not at all
2
3
4
5
Perfectly
9
Q

A45-year-old man undergoes barium swallow for dysphagia, which demonstrates multiple
flask-shaped outpouchings of barium arranged in longitudinal rows paralleling the long axis of
the oesophagus. Which of the following is a commonly associated condition? [B4 Q51]

a. scleroderma
b. rheumatoid arthritis
c. chronic obstructive airway disease
d. AIDS
e. diabetes

A

Diabetes

Oesophageal intramural pseudodiverticulosis is a condition causing dilatation of the ducts of
the submucosal glands of the oesophagus. These appear on barium meal as multiple, tiny, flask-shaped collections of barium arranged in longitudinal rows. They may appear to ‘float’ outside
the oesophagus, as the connection to the lumen may not be appreciated. Associated strictures
in the distal oesophagus are common. The condition is commonly associated with diabetes and
chronic alcoholism
but may also occur with severe oesophagitis of any cause. Candida may be
cultured in around half the cases, but this may be a secondary infection due to stasis of
secretions within the glands.

How well did you know this?
1
Not at all
2
3
4
5
Perfectly
10
Q

A 55-year-old man presents with dysphagia. He gives no history of weight loss and
investigations reveal a normal full blood picture. He is referred for a barium swallow, which
reveals a long stricture (several centimetres) in the mid to distal oesophagus with a fine reticular
pattern adjacent to the distal aspect of the stricture
and distal oesophageal widening. What is
the most likely diagnosis? [B1 Q33]

A. Reflux oesophagitis.
B. Candidiasis.
C. Barrett’s oesophagus.
D. Oesophageal adenocarcinoma.
E. Hiatus hernia.

A

Barrett’s oesophagus

This represents progressive columnar metaplasia of
the distal oesophagus secondary to reflux
oesophagitis. It is a premalignant condition
associated with an increased risk of adenocarcinoma,
40-fold that of the general population. Strictures are
more common in the distal, then mid oesophagus,
rather than the classically described proximal third.
The typical finding is of 1-cm- long strictures or
ulceration with associated gastro-oesophageal reflux
and hiatus hernia. These findings are non-specific
and may result from a variety of other causes such as
corrosive ingestion, nasogastric intubation, Crohn’s
disease, or neoplasm (primary or secondary).

However, the presence of a fine reticular pattern
extending distally from the stricture appears to be
specific for Barrett’s. A reticulonodular pattern has
been described in patients with a superficial
spreading adenocarcinoma, but this is rare and not
classically associated with a stricture.

Case courtesy of Dr Matt A. Morgan, Radiopaedia.org, rID: 44421

How well did you know this?
1
Not at all
2
3
4
5
Perfectly
11
Q

A 30-year-old man with a long history of dysphagia presents with food impaction. He has a
past medical history of allergies but nothing else of note. The food bolus passes spontaneously,
and a water-soluble followed by a barium swallow are requested prior to endoscopy, to ensure
there has been no perforation due to chicken/fishbones. The barium study reveals a moderately
long stricture in the lower oesophagus, with multiple distinct ring-like indentations. What is
the most likely diagnosis? [B1 Q56]

A. Idiopathic eosinophilic oesophagitis (IEE).
B. Crohn’s disease.
C. Oesophageal carcinoma.
D. Oesophageal perforation. E. Peptic stricture.

A

Idiopathic eosinophilic oesophagitis (IEE).

The cause of this condition is uncertain, but most authors believe it occurs as an inflammatory
response to ingested food allergens. A history of allergies is more closely correlated in children
with the condition than in adults. Only a minority of adults with IEE have peripheral blood
eosinophilia or eosinophilic gastroenteritis. The condition is most common in males aged 20–
40 who have a history of dysphagia and recurrent food impactions. The appearance of the
stricture, with its distinctive ring-like indentations, has been termed a ‘ringed’ oesophagus.
These indentations are characterized by multiple closely spaced concentric rings that traverse
the stricture.

A similar finding may be seen in congenital oesophageal stenosis, which typically occurs in
the same demographic group, with similar symptoms. The ‘ringed’ oesophagus is thus
relatively specific for IEE but is not a necessary finding (in the study quoted, it was only present
in 7 of the 14 patients, although these 7 all had strictures).

In peptic strictures, the fixed transverse folds are incomplete and further apart, producing a
characteristic step-ladder appearance as a result of trapping of barium between the folds.

Three different rings of distal oesophagus [Radiopaedia]

  • A ring – transient, smooth muscular ring, above the vestibule (V)
  • B ring (Schatzki ring if symptomatic) – thin mucosal ring at the gastro-oesophageal
    junction, associated with hiatus hernia
  • C ring – diaphragmatic indentation, associated with hiatus hernia.

Idiopathic Eosinophilic Oesophagitis
[Radiopaedia]

  • Middle-aged male
  • Food/ allergen triggers eosinophilic
    activation within oesophageal wall
  • Oesophageal strictures, webs, and spasm
  • Radiology
    o Ringed oesophagus
    o Ring-like structures may co-exist with
    long strictures – associated with oesophageal
    spasm, dysmotility and foreshortening.

Two images from an esophagram in a 33-
year-old woman complaining of food sticking in her esophagus demonstrate several ring-like
strictures of the proximal esophagus (white solid arrow) as well as more distal and longer
strictures (white curved arrow). These were persistent on multiple films.

How well did you know this?
1
Not at all
2
3
4
5
Perfectly
12
Q

A 67-year-old man is referred for a barium swallow from the surgical outpatient department
with a history of dysphagia to solids. A mid-oesophageal stricture is demonstrated. Which one
of the following causes is unlikely to be in the differential? [B2 Q12]

a. Barrett’s oesophagus
b. Squamous cell carcinoma of the oesophagus
c. Schatzki ring
d. Caustic substance ingestion
e. Epidermolysis bullosa

A

Schatzki ring
All are reasonable differentials for a mid-oesophageal stricture, albeit with varying degrees of
frequency, except for a Schatzki ring which is found in the lower oesophagus. It occurs near
the squamocolumnar junction and is associated with reflux. It is non distensible and best seen
in the prone position on barium swallow examinations. Schatzki rings are often asymptomatic,
but oesophageal dilatation may be required where dysphagia is severe.

How well did you know this?
1
Not at all
2
3
4
5
Perfectly
13
Q

A 74-year-old female patient undergoes a barium swallow and meal as part of investigation of
anaemia, as she refuses endoscopy. She denies any weight loss, dysphagia, or odynophagia.
The swallow reveals multiple rounded plaques and nodules in the mid oesophagus. What is the
most likely diagnosis? [B1 Q35]

A. Oesophageal candidiasis.
B. Herpes oesophagitis.
C. HIV oesophagitis.
D. Glycogenic acanthosis.
E. Cytomegalovirus oesophagitis.

A

Glycogenic acanthosis

Glycogenic acanthosis is a common condition affecting elderly people. Cytoplasmic glycogen
accumulates in the squamous epithelial lining of the oesophagus, producing the findings
described in the question. Patients usually have no oesophageal symptoms, and the disease is
not a precursor of malignancy (although extensive glycogenic acanthosis has been shown to be
associated with Cowden’s syndrome). The major differential diagnosis is candidiasis, but the
plaques of candidiasis plaques have a more linear, rather than rounded, appearance and it usually occurs
in immunocompromised patients who complain of odynophagia. Options C, D, and E typically
cause ulceration, not plaques.

How well did you know this?
1
Not at all
2
3
4
5
Perfectly
14
Q

A 26-year-old man known to have AIDS presents with a 2-week history of difficult and painful
swallowing. He undergoes double-contrast barium examination of the oesophagus, which
demonstrates multiple, small, superficial, round ulcers in the mid-oesophagus. The intervening
mucosa is normal, and no plaques are seen. What is the most likely diagnosis? [B4 Q34]

a. HIV oesophagitis
b. cytomegalovirus oesophagitis
c. reflux oesophagitis
d. candida oesophagitis
e. herpes simplex oesophagitis

A

Herpes simplex oesophagitis

Candida oesophagitis is the commonest cause of infectious oesophagitis and is particularly
seen in immunosuppressed individuals. It is frequently associated with oral thrush. It tends to
affect the upper half of the oesophagus, and typical appearances are of linear, longitudinally
oriented filling defects
representing heaped-up areas of mucosal plaques consisting of necrotic
debris and fungal colonies. In contrast, a normal intervening mucosa in oesophagitis is
suggestive of a viral aetiology.

Cytomegalovirus and HIV oesophagitis: One large flat seen in the distal oesophagus. Distinction between the two is made by brushings or biopsy at endoscopy.

Herpes simplex infection, the typical features of multiple, small, round superficial ulcers with surrounding radiolucent halo are similar at all sites of potential involvement, including the oesophagus, oral cavity, rectum, and anus.

How well did you know this?
1
Not at all
2
3
4
5
Perfectly
15
Q

A 32-year-old male is referred for a barium swallow by his GP due to dysphagia resistant to
medical treatment. A smooth, lobulated, eccentric mass is seen in the middle third of the
oesophagus containing foci of calcification. The diagnosis is most likely to be which one of
the following? [B2 Q10]

a. Leiomyoma
b. Squamous cell carcinoma
c. Adenocarcinoma
d. Oesophageal web
e. Intramural pseudodiverticulosis

A

Leiomyoma

Oesophageal leiomyoma is the most common benign submucosal
tumour of the oesophagus, typically occurring in young men. The
classical features of oesophageal leiomyoma include a smooth
intramural mass in the lower or middle third of the oesophagus with
intact overlying mucosa. It is the only tumour of the oesophagus that
calcifies, although calcification is rare.

Case courtesy of Dr Mohammadtaghi Niknejad, Radiopaedia.org, rID:
62519

How well did you know this?
1
Not at all
2
3
4
5
Perfectly
16
Q

A 23-year-old man with dysphagia undergoes a double-contrast
barium swallow, which demonstrates a smooth, well-defined, 12 cm submucosal lesion in the
distal oesophagus causing deformity of the lumen. CT demonstrates coarse calcification within
the mass. What is the most likely diagnosis? [B4 Q22]

a. oesophageal lipoma
b. oesophageal duplication cyst
c. oesophageal carcinoma
d. oesophageal varices
e. oesophageal leiomyoma

A

Oesophageal leiomyoma
Leiomyomas are benign tumours of smooth muscle and represent the most common benign
neoplasm of the oesophagus. They are often asymptomatic but may present with dysphagia and
rarely haematemesis. They appear on barium swallow as large, well-defined, intramural masses
causing luminal deformity. A characteristic finding is of coarse calcifications – leiomyoma is
the only calcifying oesophageal tumour. Oesophageal lipomas and duplication cysts also
appear as well-defined submucosal lesions (of fat and of water density respectively on CT), but
are less common, and internal calcification is not a feature. Oesophageal carcinoma usually
appears as an irregular ulcerated stricture. Oesophageal varices are seen as serpiginous filling
defects.

How well did you know this?
1
Not at all
2
3
4
5
Perfectly
17
Q

What is the most common cause of varices affecting the upper third of the oesophagus? [B4
Q52]

a. portal hypertension due to cirrhosis
b. splenic vein thrombosis
c. inferior vena caval obstruction
d. superior vena caval obstruction
e. hepatic vein obstruction

A

Superior vena caval obstruction
Oesophageal varices are dilated submucosal veins, which may be classified by their direction
of flow as uphill or downhill varices. Uphill varices occur in the lower oesophagus and
represent collateral blood flow conveying portal venous blood to the azygos vein. They usually
result from portal hypertension due to liver cirrhosis, but may also occur with splenic vein
thrombosis, and obstruction of the hepatic veins or IVC. Downhill varices result from
obstruction of the SVC. If it is obstructed superior to the entry of the azygos vein, varices will
be confined to the upper third of the oesophagus. If the SVC is obstructed below the entry of
the azygos vein, the varices convey all the systemic venous blood from the upper half of the
body into the portal vein and IVC, and they will run the entire length of the oesophagus. SVC
obstruction is most commonly due to lung cancer or lymphoma.

How well did you know this?
1
Not at all
2
3
4
5
Perfectly
18
Q

Which of the following are CT criteria for T3 rather than T2 oesophageal cancers? [B3 Q30]

A. Depth of 11mm
B. Transmural enhancement
C. Focal wall thickening measuring 14mm
D. A few < 1⁄3 small linear strands of soft tissue extending into fat planes
E. Slight stenosis

A

Depth of 11mm

T3 disease usually involves a large tumour more than 10 mm in depth where > 1 ⁄ 3 tumour
extension or a blurred fat plane around the lesion is associated with moderate/severe stenosis.
Focal wall thickening of 5-15mm is still T2 disease.

Oesophageal Tumour Staging [Radiopaedia]

  • T1 – Before muscularis propria
    o T1a – Lamina propria and muscularis mucosae
    o T1b – submucosa
  • T2 – Muscularis propria
  • T3 – Adventitia
  • T4 – Adjacent Structures
    o T4a – pleura, pericardium, azygous vein, diaphragm, peritoneum
    o T4b – others s/s aorta, vertebra1
How well did you know this?
1
Not at all
2
3
4
5
Perfectly
19
Q

At endoscopic ultrasound scan for staging of an oesophageal carcinoma, the tumour is seen
extending into the hypoechoic fourth layer of the oesophagus but not beyond this. What is the
T staging of the tumour? [B4 Q94]

a. Tis
b. T1
c. T2
d. T3
e. T4

A

T2

Endoscopic ultrasound is the most accurate method for local staging of oesophageal cancer. At
endoscopic ultrasound, the oesophageal wall appears as five distinct alternating hyperechoic
and hypoechoic bands that correspond to the histological layers of the oesophagus.
The innermost hyperechoic layer represents the interface between the lumen and the mucosa.
The second layer is a hypoechoic band that represents the muscularis Mucosa.
The third layer is a hyperechoic band that represents the submucosa.
The fourth layer is a hypoechoic band that represents the muscularis Propria.
The fifth outermost layer is a hyperechoic band that represents the oesophageal adventitia.

The fifth layer in the stomach, duodenum and rectum represents the serosa. For oesophageal cancer, T1 tumours invade the lamina propria or submucosa. T2 tumours invade the muscularis propria, T3 tumours invade the adventitia and T4 tumours invade adjacent tissue. Tis represents carcinoma in situ*

How well did you know this?
1
Not at all
2
3
4
5
Perfectly
20
Q

A 65-year-old man undergoes endoscopy for dysphagia, during which an ulcerated mass is
seen in the distal oesophagus. Biopsy confirms oesophageal adenocarcinoma. What is the most
accurate imaging modality for local staging of oesophageal cancer? [B4 Q3]

a. endoscopic ultrasound
b. CT
c. 18FDG PET/CT
d. MRI
e. barium swallow

A

Endoscopic ultrasound
CT is the most used imaging investigation for staging of oesophageal cancer. However, the
overall accuracy of T staging is poor, particularly with T1 and T2 tumours, and CT also tends
to overestimate tumour length. Endoscopic ultrasound scan is the most accurate imaging
method for local staging but is limited in its assessment of nodal and metastatic disease. 18
FDG PET/CT is useful in evaluation of nodal and metastatic disease, particularly in patients
being considered for surgical resection, but has limited resolution for T staging and often fails
to demonstrate T1 lesions. MR is useful in characterization of indeterminate liver lesions seen
on CT. Barium swallow is not used in the staging of oesophageal cancer.

How well did you know this?
1
Not at all
2
3
4
5
Perfectly
21
Q

A 50-year-old woman presents with dysphagia. At barium swallow, contrast passes sluggishly
into the oropharynx. No peristaltic waves are seen in the upper oesophagus. After swallowing,
the lumen of the hypopharynx and upper oesophagus remain patent and distended. The lower
oesophagus outlines normally. What is the most likely diagnosis?

A. Achalasia.
B. Scleroderma.
C. Polymyositis.
D. Chagas disease.
E. SLE.

A

Polymyositis

This condition and dermatomyositis affect skeletal muscle, which is found at the upper third
of the oesophagus. These conditions begin in the upper oesophagus and extend caudally. Other
findings at fluoroscopy include retention of barium in the valleculae and wide atonic pyriform
fossae, regurgitation and nasal reflux, aspiration, and failure of contrast to progress in the upper
oesophagus without the aid of gravity. Polymyositis and dermatomyositis are associated with
underlying malignancy. The latter also involves a heliotrope rash and Gottron’s papules on
flexor surfaces.

The lower oesophagus is composed of smooth muscle and is affected by conditions such as
scleroderma and SLE, which result in atony and lack of peristalsis in the lower two-thirds,
beginning caudally and moving cranially. Achalasia and Chagas disease result in dilatation of
the whole oesophagus, with a ‘rat-tail’ deformity at the lower end.

How well did you know this?
1
Not at all
2
3
4
5
Perfectly
22
Q

A 71-year-old female with scleroderma undergoes a barium swallow examination. Which one
of the following findings concerning the oesophagus would not be consistent with this
diagnosis? [B2 Q1]

a. Oesophageal dilatation
b. Superficial ulcers
c. Hypoperistalsis in the upper third of the oesophagus
d. Stricture 5cm above the gastro-oesophageal junction
e. Oesophageal shortening

A

Hypoperistalsis in the upper third of the oesophagus
The oesophagus is the most involved location of the gastro-intestinal tract in patients with
scleroderma. Smooth muscle atrophy causes hypoperistalsis and eventually aperistalsis in the
lower two-thirds of the oesophagus. The upper third of the oesophageal wall contains skeletal
muscle and is therefore unaffected by the disease process.

How well did you know this?
1
Not at all
2
3
4
5
Perfectly
23
Q

A 46-year-old woman with a multisystem disorder presents with dysphagia and heartburn.
Barium swallow reveals a dilated oesophagus with aperistalsis of the lower two-thirds of the
oesophagus, a patulous lower oesophageal sphincter and gastro-oesophageal reflux. Which
other organ system is most likely to be affected? [B4 Q1]
a. respiratory
b. cardiovascular
c. skin
d. central nervous
e. renal

A

Skin

The patient is suffering from systemic sclerosis, a multisystem connective tissue disorder of
unknown aetiology, classified by extent of skin involvement and overlap with other
autoimmune disorders. The skin is the most involved organ, demonstrating thickening, atrophic
changes and fibrosis. The gastrointestinal system is the next most affected, with around 50%
of patients having symptomatic disease. The oesophagus is most frequently involved, with
fibrosis of the circular layer of smooth muscle resulting in a dilated oesophagus with absent or
reduced peristalsis in the lower two-thirds. The lower oesophageal sphincter is wide, in contrast
to the tapered narrowing seen in achalasia. Patients suffer from reflux that predisposes to
Barrett’s oesophagus and distal strictures. The cardiovascular, respiratory, central nervous and
renal systems may all be affected in systemic sclerosis, though less commonly than the skin
and gastrointestinal systems.

How well did you know this?
1
Not at all
2
3
4
5
Perfectly
24
Q

A 47-year-old woman with dysphagia undergoes barium swallow, which demonstrates a
persistent smooth posterior bulge at the pharyngo-oesophageal junction at the level of C5–6
with mild proximal pharyngeal dilatation. What is the most likely diagnosis? [B4 Q97]

a. normal findings
b. impaired crico-pharyngeus relaxation
c. pharyngeal web
d. anterior cervical osteophytes
e. thyroid enlargement

A

Imparid crico-phargyneus relaxation
Impaired cricopharyngeus relaxation (or cricopharyngeal achalasia) is hypertrophy of the
cricopharyngeus muscle with failure of relaxation. It is seen in up to 10% of asymptomatic
adults as a normal variant, as a compensatory mechanism in gastro-oesophageal reflux, and in
association with a range of neuromuscular disorders. It appears on barium swallow as a smooth,
shelf-like posterior projection at the level of C5–6 that persists during a swallow. In severe
cases, it may result in functional obstruction or overflow aspiration. Symptomatic patients may
be treated by cricopharyngeal myotomy.

Pharyngeal webs are thin, anterior, shelf-like protrusions into the cervical oesophagus. They
are frequent incidental findings but occasionally cause dysphagia. There is an association with
Plummer–Vinson syndrome. Anterior osteophytes may cause an indentation of the posterior
oesophagus, but these are usually asymptomatic. Thyroid enlargement may cause a smooth
impression on the lateral wall of the oesophagus.

How well did you know this?
1
Not at all
2
3
4
5
Perfectly
25
Q

A 70-year-old man with a history of several months of dysphagia undergoes double-contrast
barium swallow. This demonstrates a moderately dilated oesophagus with reduced peristalsis
and smooth tapering of the distal oesophagus. What is the most likely diagnosis? [B4 Q19]
a. primary achalasia
b. gastric carcinoma
c. scleroderma
d. oesophageal carcinoma
e. presbyoesophagus

A

Gastric carcinoma

Primary achalasia is an abnormality of the myenteric plexus resulting in reduced or absent
peristalsis and failure of relaxation of the lower oesophageal sphincter. The oesophagus is
typically markedly dilated with absent primary peristalsis and a smooth tapered narrowing at
the contracted lower oesophageal sphincter. It usually presents in young adults with long-
standing dysphagia. In contrast, secondary achalasia due to malignancy usually presents in
older patients with a short duration of dysphagia. Decreased peristalsis and distal oesophageal
tapering in these patients result from tumour infiltration of the myenteric plexus of the distal
oesophagus by gastric carcinoma
, lymphoma, or metastatic disease. Distal oesophageal
carcinoma tends to give rise to irregular, asymmetrical narrowing. Scleroderma typically
appears as a dilated oesophagus with a patulous lower oesophageal sphincter.
Presbyoesophagus is a disorder of oesophageal motility, characterized by oesophageal
dilatation and repetitive, non-peristaltic, tertiary contractions in the distal oesophagus.

How well did you know this?
1
Not at all
2
3
4
5
Perfectly
26
Q

A 75-year-old woman presents with severe chest pain radiating to her back and some
haematemesis. The surgical team have considered a differential diagnosis of aortic dissection
or aorto-enteric fistula and requested a CT scan to assess the aorta. No aortic dissection is seen,
but there is a long eccentric filling defect identified within the oesophageal wall, extending
from the level of the carina to the gastro-oesophageal junction. This area did not enhance after
contrast administration but did measure 75 HU on a pre-contrast scan. Barium swallow
revealed a longitudinal impression on the oesophagus, which had resolved on a repeat swallow
6 weeks later. What is the most likely diagnosis? [B1 Q36]

A. Aorto-oesophageal fistula.
B. Mallory–Weiss tear.
C. Boerhaave syndrome.
D. Oesophageal varices.
E. Intramural haematoma of the oesophagus.

A

Intramural haematoma of the oesophagus
Submucosal dissection of the oesophagus may be spontaneous or secondary to direct trauma
or coagulopathy. Patients may present with chest pain, dysphagia, and nausea, often followed
by haematemesis. The high attenuation in the wall of the oesophagus is the clue to the diagnosis.
This feature and lack of enhancement are inconsistent with any alternative diagnosis. Follow-
up with endoscopy is usually performed to exclude a predisposing pathological condition. The
natural history is complete resolution without surgical intervention.

How well did you know this?
1
Not at all
2
3
4
5
Perfectly
27
Q

Which of the following represent a case of oesophageal pouch in the lower 1⁄3 of the
oesophagus? [B3 Q37]

A. Mucosal tear/Mallory-Weiss
B. Intraluminal diverticulosis
C. Defect through Killian-Jamieson space
D. Zenker’s diverticulosis
E. Traction following treatment for TB

A

Mucosal tears from Mallory-Weiss syndrome or post endoscopy are typically in the lower 1/3
of the oesophagus.

How well did you know this?
1
Not at all
2
3
4
5
Perfectly
28
Q

A 56-year-old man presents acutely with chest pain after a night out. On examination he is
febrile, tachycardic and hypotensive. Chest radiograph demonstrates extensive
pneumomediastinum, with left pleural effusion and left lower lobe atelectasis. What is the most
likely diagnosis? [B4 Q24]

a. acute pulmonary embolism
b. spontaneous oesophageal rupture
c. aortic dissection
d. lobar pneumonia
e. acute pancreatitis

A

Spontanous oesophageal rupture

Iatrogenic injury is the most common cause of oesophageal rupture, but, in 15% of cases,
rupture is spontaneous and occurs during vomiting (Boerhaave’s syndrome). Patients present
with pain and dysphagia, and rapidly develop sepsis. Characteristic chest radiograph findings
are of extensive pneumomediastinum and subcutaneous emphysema, pleural effusion or
hydropneumothorax, and left lower lobe atelectasis. Widening of the mediastinum may
accompany the development of mediastinitis. Pleural effusion and atelectasis may be seen in
acute pulmonary embolism, acute pancreatitis, and aortic dissection, but pneumomediastinum
is not a recognized feature of these conditions. Other common causes of pneumomediastinum
include asthma, chest trauma and perforation of a hollow viscus with extension of gas via the
retroperitoneum.

How well did you know this?
1
Not at all
2
3
4
5
Perfectly
29
Q

A patient is undergoing a Ba meal. What is the best position to place the patient in to see an
enface view of lesser curvature? [B1 Q13]

A. Left lateral
B. Left anterior oblique (LAO)
C. Supine
D. Right anterior oblique (RAO)
E. Right lateral

A

LAO: Lesser curvature.

RAO: Body and Antrum.
Supine: Greater curve and Antrum.
Left lateral: Fundus.

How well did you know this?
1
Not at all
2
3
4
5
Perfectly
30
Q

You are left in charge of a barium meal list. Due to an acute staff shortage, there is only a
student radiographer with you, who wants to know about which barium to use and why. Which
one of the following statements regarding barium contrast media is correct? [B1 Q19]

A. Simethicone is added to reduce flocculation.
B. The weight/volume ratio of barium for barium meals is 150%.
C. The same weight/volume ratio is used for barium meals and follow-through examinations.
D. Uniform particle size improves mucosal coating.
E. Gastrografin can be added to improve transit time.

A

Gastrografin can be added to improve transit time
Simethicone is an antifoaming agent. While a uniform particle size helps reduce flocculation,
a heterogeneous particle size improves mucosal coating. The barium densities used for
different examinations are barium swallow 150%, barium meal 250%, barium follow-through
50%, barium small bowel enteroclysis 18%, double-contrast barium enema 125%, and single-
contrast barium enema 70%.

How well did you know this?
1
Not at all
2
3
4
5
Perfectly
31
Q

Which is the most appropriate contrast medium for a barium follow-through examination of
the small bowel? [B4 Q65]

a. 100 ml of 250% w/v barium sulphate
b. 135 ml of 250% w/v barium sulphate
c. 300 ml of 100% w/v barium sulphate
d. 1500 ml of 20% w/v barium sulphate
e. 500 ml of 115% w/v barium sulphate

A

300 ml of 100% w/v barium sulphate
The recommended contrast is 300ml of 100% w/v barium sulphate. Transit time may be
reduced by the addition of 10ml of Gastrografin to the barium. Non-ionic, water-soluble
contrast media may be used as an alternative where barium is contraindicated. The
recommended concentrations and volumes for the other gastrointestinal contrast examinations
are barium swallow (a), barium meal (b), small bowel enema (d) and double-contrast barium
enema (e).

How well did you know this?
1
Not at all
2
3
4
5
Perfectly
32
Q

A six-week-old child has an ultrasound scan of the abdomen performed for non-bilious
projectile vomiting. Which one of the following features would support a diagnosis of infantile
pylorospasm over a diagnosis of hypertrophic pyloric stenosis? [B1 Q36]

a. Pyloric muscle wall thickness of 2 mm
b. Pyloric canal length of 19 mm
c. Target sign
d. Antral nipple sign
e. Transverse pyloric diameter of 14 mm

A

Pyloric muscle wall thickness of 2 mm
Hypertrophic pyloric stenosis presents between four and six weeks of life with non-bilious
vomiting, typically in first-born males. A palpable olive-shaped mass is a sign with reported
sensitivity of up to 80%, but ultrasound is the most frequently used imaging modality. Typical
ultrasound features include the target sign (central hyperechoic mucosa with surrounding
hypoechoic pyloric muscle), the nipple sign (pyloric mucosa indenting the gastric antrum),
pyloric canal length > 16 mm, transverse pyloric diameter >13 mm and pyloric muscle wall
thickness > 3 mm. Pyloric stenosis can be difficult to differentiate radiologically from infantile
pyloro-spasm. Typically, with pyloro-spasm the appearances change with time, and so if the
pyloric muscle thickness is measured at less than 3 mm this makes infantile pyloro-spasm the
more likely diagnosis.

Hypertrophic Pyloric Stenosis [STATdx]

  • Idiopathic pyloric muscle thickening in young infants causing progressive gastric outlet
    obstruction
  • Near-complete gastric outlet obstruction due to abnormally elongated & thickened pyloric
    muscle
  • Pathology
  • Idiopathic, prostaglandin or erythromycin induced, neural mediated, familial
  • Tends to run in families.
  • Clinical Features
  • 2-12 weeks old, progressive, nonbilious projectile vomiting
  • Feedings previously tolerated
  • Male preponderance, common
  • Gastroesophageal reflux > > HPS
  • Ultrasound * Hypertrophied circumferential hypoechoic muscle & elongated pyloric canal filled with
    echogenic mucosa
  • Pyloric channel length > 15-16 mm
  • Single wall thickness of pyloric muscle > 3 mm
  • Failure of thickened pylorus to change during exam in contrast to pylorospasm.
  • Fluoroscopy
  • Minimal barium traversing narrowed & elongated pyloric channel
  • Mass effect on gastric antrum & duodenum by hypertrophied muscle, hyperperistaltic
    gastric contractions, and
  • Gastroesophageal reflux/emesis
  • HPS in 1/5 of infants imaged for vomiting
How well did you know this?
1
Not at all
2
3
4
5
Perfectly
33
Q

On a barium meal examination, the incisura angularis marks the border between which
structures? [B4 Q76]

a. lesser and greater curvatures of the stomach
b. antrum and pylorus of the stomach
c. fundus and body of the stomach
d. body and antrum of the stomach
e. oesophagus and the stomach

A

Body and antrum of the stomach

The stomach is divided into the fundus, body, antrum, and pylorus. The fundus is that part of
the stomach extending superiorly and to the left of the cardiac orifice. The body extends from
the cardiac orifice to the incisura angularis, which is a constant notch at the lower end of the
lesser curvature marking the border between the body and the antrum of the stomach. The
antrum extends from the incisura angularis to the proximal pylorus.

How well did you know this?
1
Not at all
2
3
4
5
Perfectly
34
Q

At abdominal ultrasound scan, when scanning the abdomen in a transverse plane at the level
of the pancreas, which of the following structures may normally be seen lying between the
superior mesenteric artery and the aorta? [B4 Q77]

a. splenic vein
b. left renal vein
c. neck of the pancreas
d. inferior vena cava
e. common bile duct

A

Left renal vein
Ultrasound scan of the pancreas may be difficult and vascular landmarks are important in its
identification. In the transverse plane, the splenic vein can be seen coursing from the splenic
hilum towards the liver, and the body and tail of the pancreas lie immediately anterior to this.
The neck of the pancreas lies immediately anterior to the confluence of the splenic and superior
mesenteric veins, and the head and uncinate process of the pancreas lie around this confluence,
anterior to the inferior vena cava. At this level, the left renal vein can be seen entering the
inferior vena cava, passing between the superior mesenteric artery and aorta

How well did you know this?
1
Not at all
2
3
4
5
Perfectly
35
Q

A 50-year-old male is admitted with epigastric pain, diarrhoea, and vomiting. Ascites is present
clinically. Serum albumin is low and the patient is anaemic. Colonoscopy is normal but the
patient is intolerant of upper gastro-intestinal endoscopy. Barium meal reveals a normal antrum
but elsewhere there are diffusely thickened and enlarged gastric folds despite good gastric
distension. Which one of the following is the most likely diagnosis? [B2 Q26]

a. Gastric lymphoma
b. Menetrier’s disease
c. Gastric adenocarcinoma
d. Acute gastritis
e. Linitis plastica

A

Menetrier’s disease
Menetrier’s disease is a condition characterised by gastric mucosal hypertrophy and protein-
losing enteropathy.
It is often associated with anaemia. The changes are most marked along the
greater curve and the antrum is spared in approximately 50% of cases.

Gastric lymphoma typically involves the antrum.
Gastric adenocarcinoma and linitis plastica, stomach distension is not typically preserved.

How well did you know this?
1
Not at all
2
3
4
5
Perfectly
36
Q

A 48-year-old man presents with epigastric pain, weight loss and peripheral oedema. Blood
tests demonstrate hypo-albuminaemia. At barium meal the stomach is well distended, but there
is poor mucosal coating. Markedly enlarged and tortuous gastric rugae are seen in the fundus
and body of the stomach, with sparing of the antrum. What is the most likely diagnosis? [B4
Q75]

a. lymphoma
b. Menetrier’s disease
c. gastric carcinoma
d. Zollinger–Ellison syndrome
e. eosinophilic gastroenteritis

A

Menetrier’s disease
Meneterier’s disease is characterized by mucosal hypertrophy of the fundus and body of the
stomach, with excessive mucus secretion and a protein-losing enteropathy. There may be
associated gastric ulceration. Barium meal shows impaired mucosal coating due to
hypersecretion and marked gastric fold thickening, though the stomach distends normally. The
stomach is the commonest site for gastrointestinal lymphoma, which may be polypoid,
ulcerating or infiltrative. The infiltrative form may cause pronounced thickening of gastric folds,
with preserved stomach distensibility, but hypersecretion is not a feature. Infiltrating gastric
carcinoma may also cause thickened gastric folds, but associated desmoplastic reaction results in a rigid, poorly distensible stomach. Zollinger–Ellison syndrome results in hypersecretion of
gastric acid which impairs mucosal coating of barium and is associated with ulceration and
enlargement of rugal folds, but hypoproteinaemia is not a feature. Eosinophilic gastroenteritis
may cause enlarged gastric folds and be associated with protein-losing enteropathy if the small
bowel is involved. However, the antrum is most involved.

How well did you know this?
1
Not at all
2
3
4
5
Perfectly
37
Q

A 65-year-old woman, with a history of previous partial gastrectomy 10 years earlier, presents
with upper abdominal pain and early satiety. She undergoes a double-contrast barium meal,
which demonstrates a 4 cm intraluminal, mottled filling defect in the gastric remnant with no
fixed attachment to the gastric wall. What is the most likely diagnosis? [B4 Q21]

a. suture granuloma
b. trichobezoar
c. phytobezoar
d. gastric carcinoma
e. villous adenoma

A

Phytobezoar

Bezoars are masses of accumulated ingested material forming in the stomach or intestines.
Phytobezoars are the commonest type, composed of poorly digested fibre and vegetable matter.
They are seen particularly in patients with previous gastric surgery, probably due to diminished
gastric emptying. Patients may be asymptomatic or present with early satiety or symptoms of
gastritis, as phytobezoars are irritant. Occasionally, they may obstruct the stomach with a ball–
valve mechanism. They are seen as relatively mobile filling defects, the interstices of which
are filled with barium. Trichobezoars are composed of hair, and are usually larger, and found
in younger patients, particularly those with a psychiatric history. Gastric carcinoma, villous
adenoma and suture granuloma are all causes of gastric filling defects but have a constant
relationship to the gastric wall

How well did you know this?
1
Not at all
2
3
4
5
Perfectly
38
Q

A 65-year-old man presents with early satiety and bloating and undergoes barium meal. This
demonstrates a smoothly marginated, 15 cm mass in the body of the stomach, making an obtuse
angle with the gastric wall. CT demonstrates peripheral enhancement of the mass with central
areas of low attenuation and extra-gastric extension into the lesser sac. There is no associated
lymphadenopathy. What is the most likely diagnosis? [B4 Q99]

a. gastrointestinal stromal tumour
b. gastric carcinoma
c. gastric lymphoma
d. adenomatous polyp
e. gastric carcinoid

A

GIST

Gastrointestinal stromal tumours are the commonest mesenchymal tumours of the
gastrointestinal tract. They are characterized by expression of KIT, a tyrosine kinase growth
factor receptor, which distinguishes them from leiomyomas and leiomyosarcomas. They occur
most commonly in the stomach
and have the classic appearance of a submucosal mass on
barium meal, forming an obtuse angle with the gastric wall in profile. Focal areas of ulceration
are seen in 60%. On CT, the tumours measure up to 30 cm and are often predominantly extra-gastric. Typical features are of peripheral enhancement, with central low attenuation representing necrosis, haemorrhage, and cyst formation. NO Lymphadenopathy.

Gastric carcinoma and lymphoma rarely demonstrate exophytic growth and commonly have associated lymphadenopathy. Adenomatous polyps are mucosal lesions. Gastric carcinoid is usually seen in the antrum and characteristically shows associated ulceration.

How well did you know this?
1
Not at all
2
3
4
5
Perfectly
39
Q

Which of the following favours gastric lymphoma rather than other gastric malignancies? [B3
Q45]

A. Preservation of the fat plane around the stomach
B. Luminal narrowing
C. Involvement of the proximal half of the stomach
D. Heterogenous gastric wall thickening
E. A single site of disease within the stomach

A

Preservation of the fat plane around the stomach, diffuse and homogenous wall thickening,
multifocal disease within the stomach, nodal disease either side of the mesenteric vessels, nodal
disease extending below the level of the renal veins and a propensity for the distal half of the
stomach are all features of gastric lymphoma.

How well did you know this?
1
Not at all
2
3
4
5
Perfectly
40
Q

Which of the following is the most correct statement with regards to Gastrointestinal Stromal
tumour (GIST) of the stomach? [B3 Q46]

A. Most patients present below the age of 50
B. GISTS of the stomach tend to have a more aggressive histology when compared with GISTS
from other sites
C. Mural calcification is a common feature
D. Larger lesions tend to be more homogenous in enhancement
E. Central fluid attenuation/necrosis is common

A

Central fluid attenuation/ necrosis is common
Stomach GISTs tend to present in patients over 50. CT usually shows a well-defined
heterogeneously enhancing, round, exophytic mass, commonly with central necrosis. Mural
calcification is recognised but not common. Stomach GISTs tend to be less aggressive
histologically than GISTs at other sites.

How well did you know this?
1
Not at all
2
3
4
5
Perfectly
41
Q

A 54-year-old man with known metastatic malignant melanoma presents with epigastric pain
and haematemesis. What is the most likely finding in the stomach on double-contrast barium
meal? [B4 Q23]

a. multiple submucosal nodules with central ulceration
b. solitary ulcerated mass in the gastric antrum
c. linitis plastica
d. solitary, well-defined, pedunculated filling defect
e. thickened tortuous gastric folds

A

Multiple submucosal nodules with central ulceration
GI tract metastases are seen in 4–8% of patients with malignant melanoma. The small intestine
is most affected, followed by the colon and stomach. Typical features are of multiple
submucosal nodules, with a target appearance due to central ulceration. This appearance is
particularly seen with malignant melanoma metastases but may also be seen with gastric
metastases from breast, lung, and renal cell carcinoma. Other common appearances of gastric metastases include linitis plastica in 20%, most typically from breast cancer, and a solitary mass in 50%.

How well did you know this?
1
Not at all
2
3
4
5
Perfectly
42
Q

A 69-year-old man undergoes staging of gastric carcinoma diagnosed at upper gastrointestinal
endoscopy. CT of the abdomen demonstrates focal gastric wall thickening with extension into
the perigastric fat, but no invasion of adjacent structures. Five local lymph nodes measuring
10–12 mm in short axis diameter is identified. There is no distant metastatic disease. What is
the TNM staging of the tumour? [B4 Q89]

a. T2 N0 M0
b. T2 N1 M0
c. T2 N2 M0
d. T3 N1 M0
e. T3 N2 M0

A

T3 N2 M0
T3 tumours penetrate the subserosa but do not invade adjacent structures. On CT, this may be
appreciated as blurring of the tumour margin or wide reticular strands radiating from the
tumour edge. Nodal staging depends on the number of regional nodes visible, with nodes larger
than 8mm being regarded as pathological. The presence of 1–6 regional nodes results in a stage
of N1, with 7–15 nodes and >15 nodes representing nodal stages of N2 and N3 respectively.
Non-regional nodes such as para-aortic and retro-pancreatic nodes are considered M1 disease

How well did you know this?
1
Not at all
2
3
4
5
Perfectly
43
Q

A 66-year-old woman with a known large para-oesophageal hiatus hernia presents with sudden
onset of severe epigastric pain and vigorous retching without production of vomitus. Passage
of a nasogastric tube is unsuccessful. Plain abdominal radiograph demonstrates a markedly
distended stomach in the left upper quadrant extending into the chest. What is the most likely
diagnosis? [B4 Q53]

a. pyloric stenosis
b. ‘cup-and-spill’ stomach
c. acute gastric volvulus
d. acute gastric dilatation
e. paraduodenal hernia

A

Acute gastric volvulus
Acute gastric volvulus is abnormal rotation of one part of the stomach around another part,
which may be classified as organoaxial, mesenteroaxial or combination type, depending on the
axis of rotation. Predisposing factors include ligamentous laxity, hiatus hernia and
diaphragmatic eventration. The classic presentation is with the Borchardt triad of sudden severe
epigastric pain, intractable retching with no vomitus produced, and inability to pass a
nasogastric tube into the stomach. Other plain film findings include unexpected location of the
gastric bubble and air–fluid levels in the mediastinum or upper abdomen, but definitive
diagnosis is by barium meal. The condition is a surgical emergency, as it may result in gastric
ischaemia or perforation. Acute gastric dilatation and pyloric stenosis may result in gastric
distension on plain film but would not present with intractable retching or difficulty with
nasogastric tube passage. A ‘cup-and-spill’ stomach is an anatomical variant on barium meal,
which may simulate an organoaxial volvulus. A para-duodenal hernia usually presents acutely
as small bowel obstruction.

Gastric Volvulus [Radiopaedia]

Case courtesy of Dr Maxime St-Amant, Radiopaedia.org, rID: 19

How well did you know this?
1
Not at all
2
3
4
5
Perfectly
44
Q

A patient presents to an outpatient barium meal list with a history of epigastric discomfort and
weight loss of 8 kg over 6 months. The barium meal reveals an ulcer on the greater curve of
the stomach, near the pylorus. This ulcer has a surrounding mound. It is demonstrated to project
slightly beyond the lumen of the stomach. There is a thin line noted which crosses the base of
the ulcer and a degree of retraction of the greater curve around the ulcer. What type of ulcer is
this likely to be? [B1 Q6]

A. Benign due to the line noted crossing the base of the ulcer.
B. Benign due to the ulcer projecting beyond the lumen of the stomach.
C. Benign due to the surrounding mound.
D. Malignant due to the finding of scar retraction of the greater curve.
E. Malignant due to being found on the greater curve.

A

Benign due to the line noted crossing the base of the ulcer - Hampton’s line
This line—Hampton’s line—represents undermining of the mucosa by the more vulnerable
submucosa. It is not commonly seen but is taken to be virtually diagnostic of a benign ulcer
when present. Projection beyond the lumen and a symmetrical mound are features of a benign
ulcer along with smooth radiating mucosal folds. Scar retraction can be seen with benign ulcers.
Both benign and malignant ulcers are more commonly seen on the lesser curve.

Benign Vs Malignant Ulcers [Core Radiology]

Benign gastric ulcer

o Radiating gastric folds are smooth and symmetric
o Ulcer extends beyond the normal contour of the gastric lumen.
o The Hampton line represents nonulcerated acid-resistant mucosa surrounding the ulcer
crater.
o Most benign ulcers occur along the lesser curvature of the stomach, although benign ulcers
associated with aspirin ingestion can occur in the greater curvature and antrum, which are
dependent locations.

Malignant

o Asymmetric ulcer crater, with surrounding nodular tissue
o Abrupt transition between normal gastric wall and surrounding tissue.
o Ulcer crater does not project beyond the expected location of gastric wall.
o The Carman meniscus sign is considered pathognomonic for tumour. It describes the
splaying open of a large, flat malignant ulcer when compression is applied.

How well did you know this?
1
Not at all
2
3
4
5
Perfectly
45
Q

A64-year-old man undergoes a barium meal examination for upper abdominal pain. A 10 mm
ulcer is demonstrated at the gastric antrum. Which radiological feature would favour a
diagnosis of malignant rather than benign gastric ulcer? [B4 Q4]

a. round ulcer shape
b. ulcer crater confined within the gastric contour
c. gastric folds identified up to the edge of the ulcer crater
d. associated duodenal ulcer disease
e. uniform mucosal collar around a centrally located ulcer

A

Ulcer crater confined within gastric contour
Many distinguishing features of gastric ulceration have been proposed to classify gastric ulcers
as benign or malignant, but there is significant overlap between the two categories. One reliable
sign of a benign ulcer is the projection of the ulcer outside the gastric contour in profile, due to
excavation into the mucosal wall. In contrast, a malignant ulcer occurring within a tumour mass
does not usually extend beyond the confines of the gastric wall. Other features indicative of
benignity include a round, centrally located ulcer with a uniform collar of oedematous mucosa,
gastric folds extending to the edge of the ulcer crater and associated duodenal ulcer disease.

How well did you know this?
1
Not at all
2
3
4
5
Perfectly
46
Q

A 63-year-old man is day 7 post-operative following a Billroth II partial gastrectomy for a
gastric carcinoma. The initial post-operative phase was uncomplicated, but the patient has
begun complaining of increasing abdominal pain. Inflammatory markers have increased with
white cell count (WCC), rising from 12 to 42, and CRP increased from 8 to 56. A CT scan
carried out with oral and intravenous contrast demonstrates no evidence of contrast leakage
into the peritoneum. A skiff of free air is noted in the abdomen. A fluid collection is noted in
the right subhepatic space, which extends toward the peripancreatic area. What is the most
likely diagnosis? [B1 Q1]

A. Leakage from the gastroduodenal anastomosis site.
B. Leakage from the duodenal stump.
C. Post-operative pancreatitis.
D. Tumour recurrence.
E. Pseudocyst formation following post-operative pancreatitis.

A

Leakage from the duodenal stump.
There is no gastro-duodenal anastomosis in a Billroth II procedure. The amylase is not
sufficiently elevated for pancreatitis in most cases and there is no described abnormality in the
pancreas. It is too early for pseudocyst formation and tumour recurrence.

Different Surgeries for Peptic Ulcer Disease

  1. Billroth I partial gastrectomy – Seldom performed
    * Distal gastrectomy
    * Gastro-duodenal anastomosis (proximal stomach to D1)
  2. Billroth II/ Polya partial gastrectomy
    * Distal gastrectomy
    * Closure of duodenal stump
    * Gastro-jejunal anastomosis (retro or antecolic, iso or anti-peristaltic direction)
  3. Subtotal gastrectomy with Roux Loop reconstruction
    * Distal gastrectomy
    * Closure of duodenal stump
    * Continuity is restored with a Roux loop reconstruction
  4. Total gastrectomy (in Ca stomach)
    * Total gastrectomy with lymphadenectomy
    * Duodenal stump is stapled or over sewn
    * A Roux loop is anastomosed to distal oesophagus (end to side) with blind limb
    * Jejuno-jujunal anastomosis 50-60 cm distal to oesophago-jejunal anastomosis
  5. Truncal vagotomy
  6. Wedge Resection
  7. Gastro-enterostomy (to overcome the GOO)
    Side to side gastro-jejunal anastomosis
    No Roux loop → increased risk of bile gastritis and associated stomach cancer.

Post-operative Complications

  1. Anastomotic leak

  • Most serious, highest after total gastrectomy
  • Extra-luminal contrast leak, duodenal stump is not usually evaluated in Ba study
  • Disproportionate pneumoperitoneum
  • Collections in unusual place (left subphrenic space) as anatomy is disrupted

  1. Duodenal stump leak

  • Right subhepatic space fluid collection, can be precipitated by afferent loop obstruction
  • Disruption of duodenal stump metal staple line

  1. Afferent Loop Obstruction/Syndrome

  • Associated with Billroth II gastrectomy and gastrojejunostomy
  • Adhesion or twisting of the bowel proximal to gastroenterostomy site
  • Acute obstruction can lead to duodenal stump blow out
  • Risks – long afferent limb (>30-40 cm), antecolic gastrojejunostomy.
  • Ba study is of limited value, as doesn’t normally fill the afferent limb
  • CT → fluid filled dilated afferent limb with associated biliary dilatation

  1. Anastomotic stricture
  2. Parastomal jejunal ulcer

  • Common after Billroth II as gastric acid goes into jejunum.
  • Very rarely, can lead to gastro-jejuno-colic fistula.
    6. Gastric Remnant Complications
  • Bile reflux gastritis – Billroth I and II
  • Gastroparesis
  • B12 deficiency
  • Gastric remnant malignancy – commonest at the anastomosis

  1. Efferent Complications

  • Obstruction secondary to adhesions, local recurrence, intussusception or internal hernia.
  • Hiatus hernia
  • Early dumping syndrome secondary to rapid fluid shift.
  • Late dumping syndrome secondary to rapid CHO absorption.
How well did you know this?
1
Not at all
2
3
4
5
Perfectly
47
Q

Which is most correct regarding internal hernia after bariatric laparoscopic Roux-en-Y gastric
bypass? [B3 Q26]

A. Incidence of SBO secondary to internal lesions 0.2-5.0 %
B. Occur more frequently in open than laparoscopic procedures
C. The obstruction commonly occurs in the efferent limb
D. Physical examination detects the problem with high sensitivity
E. Following Roux-en-Y gastric bypass, the stomach should be relatively distended/fluid filled

A

Incidence of SBO secondary to internal lesions 0.2-5.0 %
Hernia more common in laparoscopic approaches occur in the mesocolic, Peterson’s and meso-
mesenteric defects. Obstruction commonly occurs in the afferent (biliopancreatic) limb causing
distension of the very proximal jejunum, duodenum, and remnant stomach. Remnant stomach
should be relatively decompressed.

How well did you know this?
1
Not at all
2
3
4
5
Perfectly
48
Q

Regarding laparoscopic adjustable gastric banding: [B3 Q27]

A. Is connected by tubing to a port anterior to the rectus sheath
B. On scout films, the band should be parallel to the Gastro Oesophageal Junction (GOJ)
C. When patient ingests contrast, position of band best assessed on lateral decubitus projection
D. With band slippage, the pouch is narrowed
E. Perforation rates are typically 1-2%

A

Is connected by tubing to a port anterior to the rectus sheath

Band should be perpendicular to GOJ on scout film. Position is best assessed straight AP or
slightly RPO. Band slipping leads to eccentric pouch dilation. Perforation typically <0.5%

How well did you know this?
1
Not at all
2
3
4
5
Perfectly
49
Q

A 41-year-old woman with morbid obesity presents with a plateau in weight loss 12 weeks
after laparoscopic gastric banding. She undergoes a contrast swallow, which demonstrates
concentric dilatation of the neo-stomach with a widely patent stoma. What is the most
appropriate management? [B4 Q81]

a. no action necessary
b. nutritional advice
c. prompt decompression of the stoma by the radiologist
d. fluoroscopically guided band inflation
e. surgical replacement of the gastric band

A

Nutritional advice

Laparoscopic gastric banding involves laparoscopic placement of an inflatable gastric band
across the proximal stomach, forming a small fundal neo-stomach or pouch. The band is
connected to a subcutaneous port that can be accessed percutaneously to allow inflation or
deflation of the band and adjustment of stomal width and degree of hold-up. The commonest
postoperative complication is dilatation of the pouch. Three main types are described. (1) Acute
concentric pouch dilatation is due to band overinflation and is seen as a pre-stenotic dilatation proximal to an obstructed stoma. It presents as acute dysphagia and requires prompt
decompression of the stoma. (2) Chronic concentric pouch dilatation with a widely patent
stoma is seen in patients who continue to overfill their neo-stomach after surgery. Nutritional
advice
is required. (3) Eccentric pouch dilatation occurs due to slippage of the band and
requires complete decompression and surgical replacement of the band. A plateau in weight
loss may also be due to loss of effect of band tightening. At fluoroscopy, fluid may be injected
to tighten the band to achieve an optimal stomal width of 3–4mm.

How well did you know this?
1
Not at all
2
3
4
5
Perfectly
50
Q

Small Bowel Obstruction (Includ: Large Bowel)

A patient presents to the surgical team with central abdominal pain and vomiting associated
with abdominal distension. The abdominal x-ray (AXR) reveals numerous dilated loops of
small bowel. A CT scan is carried out. Which of the following statements regarding CT
imaging in small bowel obstruction is accurate? [B1 Q20]

A. Small bowel mural hyper-density is a feature and is due to vasodilatation seen in early
ischaemia.
B. Oral contrast is mandatory for the investigation of small bowel obstruction.
C. Small bowel mural thickening is due to increased venous pressure.
D. Absence of small bowel mural enhancement is a feature of ischaemic gut secondary to
emboli rather than small bowel obstruction.
E. Lack of small bowel pneumatosis excludes ischaemia of the gut.

A

Small bowel mural hyper-density is a feature and is due to vasodilatation seen in early
ischaemia.

Multi-detector CT (MDCT) has been found to correlate with pathological processes in small
bowel obstruction. The earliest appearance is increased mural density due to hyperaemia.

Wall thickening is due to increasing capillary permeability, which causes submucosal oedema.
Dilatation is secondary to oedema that limits peristalsis. Lack of enhancement occurs when the
bowel dilates and compresses the capillary bed. Pneumatosis is secondary to mucosal
ischaemic change, which allows luminal air to track into the wall. Lack of enhancement is also
seen in embolic ischaemia but is not a specific sign of this process. Whilst oral contrast is
preferred in many centres, as it can help define if complete obstruction is present, it is not
mandatory. Some centres prefer the negative contrast provided by the fluid in the bowel lumen.
Patients with small bowel obstruction are also often unable to tolerate oral contrast due to
vomiting.

How well did you know this?
1
Not at all
2
3
4
5
Perfectly
51
Q

A 20-year-old woman with anorexia nervosa presents with intermittent abdominal pain and
vomiting relieved by lying prone. Barium meal examination reveals a vertical band-like
narrowing of the third part of the duodenum, with proximal duodenal dilatation and vigorous
to-and-fro peristalsis. What is the most likely diagnosis? [B4 Q28]

a. duodenal duplication cyst
b. annular pancreas
c. Ladd’s bands
d. superior mesenteric artery syndrome
e. duodenal atresia

A

Superior mesenteric artery syndrome
The third part of the duodenum is bounded posteriorly by the aorta, and anteriorly by the root
of the mesentery carrying the superior mesenteric artery. In superior mesenteric artery
syndrome, the third part of the duodenum is compressed by the superior mesenteric artery, and the angle between it and the aorta narrows to 10–22 (normal 45–65 ). The condition is
associated with severe weight loss, prolonged bedrest (particularly in a body cast), lumbar
lordosis and pregnancy. Patients report intermittent abdominal pain and vomiting, relieved by
lying prone or in the knee–elbow position. Duodenal atresia causes complete obstruction
usually distal to the ampulla of Vater and presents in neonates with a ‘double-bubble’ sign on
plain abdominal radiograph. Annular pancreas is usually asymptomatic but may present with
abdominal pain and vomiting, and barium meal demonstrates narrowing of the second part of
the duodenum. Duodenal duplication cysts cause extrinsic compression of the first and second
portions of the duodenum. Ladd’s bands are congenital peritoneal bands occurring in
association with malrotation that may cause obstruction of the second part of the duodenum,
but presentation is usually in infants and children.

How well did you know this?
1
Not at all
2
3
4
5
Perfectly
52
Q

A patient presents to A&E with severe upper abdominal pain 4 days following a barium enema.
There is no free air under the diaphragm on the erect CXR. There is mild elevation of the
inflammatory markers, but the surgeon is concerned with the degree of peritonism and requests
a CT scan of abdomen. On this, the small bowel is dilated to 5 cm, but is not thick walled. The
vascular structures enhance normally. There is inflammatory change noted around the
duodenum. Linear areas of low attenuation are noted extending from the porta hepatis into the
liver parenchyma. These do not extend to the margin of the liver and are in general central in
their location. The Hounsfield attenuation value of these areas is approximately –1500 HU.
Barium in the rectum obscures the images of the pelvis. What is the most likely pathology?
[B1 Q34]

A. Cholecysto-duodenal fistula.
B. Mesenteric infarction.
C. Acute bowel obstruction.
D. Perforated duodenal ulcer.
E. Complication of barium enema.

A

Cholecysto-duodenal fistula
The other answers are all causes of portal air, whereas the salient description is for air in the
biliary tree.

How well did you know this?
1
Not at all
2
3
4
5
Perfectly
53
Q

A 55-year-old man with a previous history of liver transplantation presents with a 1-week
history of abdominal pain and distension. An AXR shows some distended small bowel loops
centrally within the abdomen. You are asked to perform a CT scan of abdomen for further
evaluation. This shows a cluster of non-encapsulated dilated small bowel loops adjacent to the
anterior abdominal wall on the right side. There are adjacent crowded mesenteric vessels. What
is the most likely diagnosis?

A. Small bowel adhesions.
B. Left paraduodenal hernia.
C. Right paraduodenal hernia.
D. Foramen of Winslow hernia.
E. Trans-mesenteric hernia.

A

Trans-mesenteric hernia
This is when small bowel herniates through a defect in the mesentery and is compressed against
the abdominal wall, with little overlying omental fat at most levels of anatomic section through
the herniated bowel. There will be some degree of compression, crowding, displacement, and
obstruction of both the bowel and blood vessels. They are usually seen in association with
previous abdominal surgery and the creation of a Roux-en-Y anastomosis when the hernia
occurs in a surgically created defect in the mesentery.

A left-sided paraduodenal hernia is via the paraduodenal (lateral to the fourth part) mesenteric
fossa of Landzert, close to the ligament of Treitz. The characteristic features include a sac-like
mass of dilated bowel lateral to the ligament of Treitz, which displaces and indents the adjacent
stomach and transverse colon.

A right paraduodenal hernia occurs via the jejunal mesentericoparietal fossa of Waldeyer. A
cluster of dilated small bowel loops is seen lateral and inferior to the descending duodenum.

How well did you know this?
1
Not at all
2
3
4
5
Perfectly
54
Q

A 60-year-old female has a plain abdominal film which shows a grossly distended segment of
bowel. Which one of the following features makes a diagnosis of caecal volvulus more likely
than sigmoid volvulus? [B2 Q4]

a. Pelvic overlap sign
b. Apex lying above the level of T10
c. Liver overlap sign
d. Coffee bean sign
e. Presence of haustral markings

A

Presence of haustral markings
Sigmoid and caecal volvulus can sometimes be difficult to differentiate on plain abdominal
film. With caecal volvulus the haustral markings are typically present, whereas these are
usually absent in sigmoid volvulus. The pelvic overlap, liver overlap and coffee bean signs are
typical of sigmoid volvulus. In sigmoid volvulus the apex lies high in the abdomen underneath
the left hemi-diaphragm, typically above the level of T10.

Signs associated with Sigmoid Volvulus [My note]

Coffee Bean Sign
Pelvic Overlap Sign
Liver Overlap Sign
Northern Exposure Sign
Loss of Haustration

How well did you know this?
1
Not at all
2
3
4
5
Perfectly
55
Q

A 78-year-old man presents with abdominal pain. A plain abdominal radiograph demonstrates
a distended, inverted U-shaped loop of bowel devoid of haustra, extending from the left iliac
fossa inferiorly to just beneath the left hemidiaphragm superiorly. What is the most likely
diagnosis? [B4 Q26]

a. caecal volvulus
b. sigmoid volvulus
c. paralytic ileus
d. large bowel obstruction due to distal malignancy e. small bowel malrotation and volvulus

A

Sigmoid volvulus
Sigmoid volvulus usually occurs when the sigmoid loop twists around its mesenteric axis,
creating a closed loop obstruction. Typical features are of an inverted U-shaped loop
converging on the left side of the pelvis. The bowel loop is usually markedly distended, appears
ahaustral, and may overlap the lower border of the liver (liver overlap sign) or the haustrated
dilated descending colon (left flank overlap sign). The apex of the volvulus usually lies under
the left hemidiaphram with its apex above the level of T10. Caecal volvulus occurs when the
caecum is on a mesentery and involves the caecum either twisting and inverting so the caecal
pole lies in the left upper quadrant or twisting in an axial plane so that the caecum remains right
sided or central. Appearances are of a large, gas-distended viscus, usually with haustral
markings, and occasionally the gas-filled appendix may be identified.

How well did you know this?
1
Not at all
2
3
4
5
Perfectly
56
Q

A 27-year-old male has recurrent admissions for intermittent low-grade small bowel
obstruction of unknown cause. Which one of the following investigations would be most
appropriate? [B2 Q15]

a. Contrast-enhanced CT abdomen and pelvis
b. Barium meal
c. Small bowel enteroclysis
d. Serial abdominal plain films
e. Barium follow-through

A

Small bowel enteroclysis
Small bowel enteroclysis is the most appropriate examination. CT is sensitive for high-grade
obstruction as it will readily identify the level of obstruction and can demonstrate
complications such as ischaemia and perforation. Enteroclysis is the preferred investigation for
recurrent low-grade obstruction as it is more likely to demonstrate the presence of a transition
point (for example from non-obstructing adhesions) because the bowel is distended.

The examination involves passing a nasojejunal tube just distal to the duodenojejunal flexure
and distending the small bowel using either dilute barium or a double-contrast examination
with high-density barium and methylcellulose.

How well did you know this?
1
Not at all
2
3
4
5
Perfectly
57
Q

A 71-year-old female is admitted via A&E with abdominal pain, abdominal distension, and
vomiting. Plain abdominal film shows multiple dilated loops of small bowel. In addition, there
is gas projected over the liver shadow which is prominent centrally and has a branching
appearance. Gas is not visible over the periphery of the liver. No other abnormality is seen on
the plain film. Which of the following diagnoses is most likely? [B2 Q29]

a. Small bowel perforation
b. Small bowel infarction
c. Gallstone ileus
d. Emphysematous cholecystitis
e. Pneumatosis intestinalis

A

Gallstone ileus
Specific signs of gallstone ileus can be seen on the plain abdominal film in up to 40% of
patients. Fifty per cent of patients have evidence of small bowel obstruction and up to 30%
have gas in the biliary tree. Biliary tree gas is typically more prominent centrally and spares
the periphery of the liver, whereas portal venous gas is more easily visualised in the periphery
of the liver, which may be associated with small bowel infarction. The gallstone most
frequently lodges in the terminal ileum but is often not seen on the plain film. The presence of
small bowel obstruction, pneumobilia and a visible stone are called Rigler’s triad.

How well did you know this?
1
Not at all
2
3
4
5
Perfectly
58
Q

13 months post-Roux-en-Y bariatric gastric bypass surgery a 45-year-old woman presents with
abdominal pain, vomiting and constipation. Abdominal X-ray shows Small Bowel Obstruction
(SBO) on CT. Which is the most likely cause of the SBO if the site of the obstruction is in the
alimentary limb? [B3 Q12]

A. Internal hernia
B. Jejeno-jejunostomy stricture
C. Intussusception of the Roux limb
D. Intraluminal haematoma
E. Meso-colic haematoma

A

B-D occur < 12 months, whereas internal hernias and meso-colic constriction of the Roux limb
are more chronic complications. Chronic complications with obstruction at the biliopancreatic
limb, internal hernias and anastomotic strictures at the common channel are internal hernias
and adhesions.

How well did you know this?
1
Not at all
2
3
4
5
Perfectly
59
Q

A 68-year-old woman presents with small bowel obstruction and undergoes contrast-enhanced
CT of the abdomen. This demonstrates dilated small bowel to the level of the mid-ileum, where
a herniated loop of small bowel is seen emerging inferolateral to the left pubic tubercle. What
is the most likely cause of small bowel obstruction in this patient? [B4 Q8]

a. femoral hernia
b. indirect inguinal hernia
c. direct inguinal hernia
d. spigelian hernia
e. obturator hernia

A

Femoral hernia
External hernias are the second most common cause of small bowel obstruction after adhesions.
A femoral hernia protrudes through the femoral ring, lying medial to the femoral vein, which
may be compressed. On CT the hernia is seen inferolateral to the pubic tubercle, in contrast to
inguinal hernias, which usually lie superomedial to the tubercle, though differentiation may be
difficult in non-incarcerated cases. Femoral hernias are more prone to incarceration due to the
inflexible margins of the femoral ring. Inguinal hernias may be classified as indirect (passing
down the inguinal canal, seen lateral to the inferior epigastric vessels) or direct (protruding
directly through the lower abdominal wall medial to the inferior epigastric vessels). A spigelian
hernia protrudes through a defect in the inferolateral anterior abdominal wall. Obturator hernias
protrude through the obturator foramen, between the pectineus and external obturator muscles.

How well did you know this?
1
Not at all
2
3
4
5
Perfectly
60
Q

A 68-year-old woman presents with abdominal pain, distension, and vomiting. Plain abdominal
radiograph demonstrates bowel obstruction, gas within the biliary tree, and an ectopic, calcified,
3 cm gallstone. What is the most likely site of bowel obstruction? [B4 Q70]

a. pylorus
b. duodenum
c. proximal ileum
d. terminal ileum
e. sigmoid

A

Terminal ileum
Gallstone ileus accounts for up to 5% of intestinal obstruction, increasing in prevalence with
age. It involves erosion of a large gallstone from the gallbladder or common bile duct into the
bowel, which goes on to cause obstruction. The classic appearance on plain film (Rigler’s triad)
is only seen in 10% of cases and consists of partial or complete intestinal obstruction (usually
small bowel), gas in the biliary tree and an ectopic calcified gallstone. The most common site
of fistulous communication is between the gallbladder and the duodenum, seen in 60%, and
this may be demonstrated on barium meal as a contrast collection lateral to the first part of the
duodenum representing barium within the gallbladder. Fistulas occur less commonly between
the common bile duct and duodenum, gallbladder and colon. The ectopic gallstone most often
causes obstruction at the terminal ileum (60–70%), followed by the proximal ileum, distal
ileum, pylorus, sigmoid and duodenum.

How well did you know this?
1
Not at all
2
3
4
5
Perfectly
61
Q

A 35-year-old male patient from the Indian subcontinent presents with a 2-month history of
lower abdominal pain, per rectum (PR) bleeding, and weight loss. His haemoglobin is 9.4 and
C-reactive protein (CRP) is 123. The patient is tender in the RIF. A CT scan is performed due
the suspicion of appendiceal pathology, but unusual history. This shows bowel wall thickening
of the terminal ileum with mild proximal bowel dilatation. The inner bowel wall is hypodense
with enhancement of the outer bowel wall. There is stranding in the fat, which causes mass
effect displacing other loops of bowel. Mild regional adenopathy is noted. The appendix is not
visualized, but the caecum appears normal. There is a similar area of bowel wall thickening in
the sigmoid colon. What is the most likely diagnosis? [B1 Q22]

A. Yersinia.
B. Tuberculosis.
C. Lymphoma.
D. Crohn’s disease.
E. Carcinoid

A

Crohn’s disease
The findings described are classical for Crohn’s disease and lymphadenopathy is seen in up to
30% of cases. Tuberculosis more typically involves the caecum. Lymphoma usually causes a
nodular appearance to the bowel. It is not associated with stricturing of the affected segment
and is more classically associated with dilatation of the affected segment due to destruction of
the myenteric plexus.

How well did you know this?
1
Not at all
2
3
4
5
Perfectly
62
Q

A 40-year-old male with a 22-year history of Crohn’s disease presents with abdominal pain,
diarrhoea, and low-grade fever. To attempt to limit his lifetime radiation exposure he is
investigated via MR enterography. Which of the following MRI findings is the earliest in active
inflammation? [B1 Q73]

A. Increased mesenteric vascularity.
B. Small bowel wall thickening.
C. Mucosal hyperenhancement.
D. Peri-enteric inflammation.
E. Reactive adenopathy

A

Mucosal hyperenhancement
The lack of ionizing radiation is a major advantage to MRI for patients with Crohn’s disease
given the chronic nature of this condition necessitating frequent investigation. MRI can be
performed via enterography or enteroclysis. In enterography, large volumes of fluid (or a fluid
inducing laxative) are ingested. Enteroclysis involves administration of enteric contrast
material via a naso-enteric tube. Sequence acquisition involves fat suppression and intravenous
contrast. Increased mucosal hyperenhancement (compared with that seen in normal
surrounding loops) may be the earliest sign of active inflammation, even in the absence of wall
thickening. Increased vascularity, peri-enteric inflammation, and reactive adenopathy are other
signs of active Crohn’s disease. In severe Crohn’s disease mucosal hyperenhancement
combined with submucosal oedema gives a ‘stratified’ appearance. Serosal hyperenhancement may also be seen, giving a ‘target’ appearance. Mural thickening is defined as greater than 3
mm, although an under-distended bowel may mimic this finding.

How well did you know this?
1
Not at all
2
3
4
5
Perfectly
63
Q

Which of the following most favours Crohn’s versus pseudomembranous colitis? [B3 Q6]

A. Ascites
B. Absence of small bowel involvement
C. Fibro fatty mesenteric proliferation around involved colon
D. Colonic wall thickening of 11mm
E. Low attenuation mural thickening (accordion sign)

A

Fibro fatty mesenteric proliferation around involved colon is more likely to indicate Crohn’s
disease.

How well did you know this?
1
Not at all
2
3
4
5
Perfectly
64
Q

A small bowel series is requested for a patient who has a history of systemic sclerosis. Which
of the following is a feature of small bowel systemic sclerosis? [B1 Q8]

A. Stacked coin appearance due to infiltration of small bowel loops.
B. Pseudo-diverticula affecting the anti-mesenteric side of the bowel.
C. Decreased intestinal transit time.
D. Small bowel systemic sclerosis is only seen in 10% of patients with systemic sclerosis, but
the disease is rapidly progressive when it is present.
E. Pneumatosis intestinalis.

A

Pneumatosis intestinalis
The stacked coin appearance is seen secondary to intramural haemorrhage—the appearances
of systemic sclerosis are of tightly packed folds of normal thickness in a dilated portion of
bowel, which has been given the title ‘accordion’ or ‘hidebound’ bowel.

The pseudo-diverticula (10–40%) are seen on the mesenteric side of the bowel, unlike colonic
diverticula.

The transit time is prolonged, as there is reduced intestinal motility. Another classical feature
is of a markedly dilated duodenum, due to the loss of the enteric innervations—mega
duodenum. This classically terminates abruptly at the level of the superior mesenteric artery
(SMA).

Pneumatosis cystoides can occur in systemic sclerosis of the small bowel. Small bowel disease
is seen in up to 40% of patients with systemic sclerosis and indicates rapidly progressing
disease.

Scleroderma [Core Radiology]

  • Def: a systemic disease characterized by the deposition of collagen into multiple internal
    organs and the skin.
  • Replacement of the muscular layers with collagen, resulting in
  • Slowed transit
  • Bacterial overgrowth
  • Progressive dilatation
  • Radiographically
  • Sacculation on the antimesenteric border
  • Hidebound Bowel – thin, straight bowel fold stacked together.

Intestinal Scleroderma [STATdx]

  • Progressive systemic sclerosis
  • GI tract: Most common internal organ system involvement (80-90%)
    o Oesophagus > duodenum > anorectal > small bowel > colon
  • Oesophagus
    o Atony or aperistalsis (lower 2/3 smooth muscle)
    o Mild to moderate dilatation
    o Patulous lower oesophageal sphincter (early)
    o Ulcers, fusiform peptic stricture (late)
    o GORD, Barrett’s oesophagus
  • Small bowel
    o Marked dilatation of small bowel, especially duodenum and jejunum
    o Duodenal findings identical to superior mesenteric artery (SMA) syndrome
    o Hidebound small bowel: Atonic with closely spaced, thin folds, sacculation
    (pathognomonic of scleroderma)
    o Prolonged transit time with barium retention in duodenum and small bowel up
    to 24 hours
    o Penumatosis intestinalis and pneumoperitoneum
    o Transient, nonobstructive intussusceptions
  • Colon
    o Sacculation on border of transverse and descending colon
    o Loss of haustrations
    o Stercoral ulceration (from retained faecal material in rectosigmoid)
  • Top Differential Diagnoses
    o SMA syndrome
    o Celiac-sprue disease
    o Ileus
How well did you know this?
1
Not at all
2
3
4
5
Perfectly
65
Q

A 71-year-old female with scleroderma undergoes a barium swallow examination. Which one
of the following findings concerning the oesophagus would not be consistent with this
diagnosis? [B2 Q1]

a. Oesophageal dilatation
b. Superficial ulcers
c. Hypoperistalsis in the upper third of the oesophagus
d. Stricture 5cm above the gastro-oesophageal junction
e. Oesophageal shortening

A

Hypoperistalsis in the upper third of the oesophagus
The oesophagus is the most involved location of the gastro-intestinal tract in patients with
scleroderma. Smooth muscle atrophy causes hypoperistalsis and eventually aperistalsis in the
lower two-thirds of the oesophagus. The upper third of the oesophageal wall contains skeletal
muscle and is therefore unaffected by the disease process.

How well did you know this?
1
Not at all
2
3
4
5
Perfectly
66
Q

A 41-year-old female with a background of arthralgia, chronic abdominal pain, and diarrhoea
is investigated via a small bowel series. Findings include a prolonged transit time, and dilated
loops of small bowel with normal appearing valvulae and pseudodiverticula. What is the most
likely diagnosis? [B1 Q53]

A. GI scleroderma.
B. Behcet’s disease.
C. Whipple disease.
D. Small bowel lymphoma. E. Coeliac disease.

A

GI scleroderma
Deeply penetrating ulcers are seen in Behcet’s disease. Whipple disease is an extremely rare
form of intestinal lipodystrophy. Thickening of jejuna folds is seen, but there is little or no
small bowel dilatation and small bowel transit time is normal. Pseudodiverticula are not seen
in coeliac disease. The valvulae are thickened in lymphoma.

How well did you know this?
1
Not at all
2
3
4
5
Perfectly
67
Q

Which is the most affected site in systemic sclerosis after the oesophagus? [B3 Q16]

A. Anorectal
B. Small bowel
C. Colon
D. Stomach
E. Oropharynx

A

Oesophagus is most frequently affected (75-90%) followed by anorectum (50-70%), small
bowel (40%) and colon (10-50%). The stomach is the least affected in the GIT

How well did you know this?
1
Not at all
2
3
4
5
Perfectly
68
Q

Which of the following is a case of decreased/absent duodenal folds? [B3 Q38]

A. Scleroderma
B. Pancreatitis
C. Lymphoma
D. Melanoma metastases
E. Whipple’s disease

A

Scleroderma

Scleroderma, strongyloides, and cystic fibrosis are causes of decreased/absent folds. Crohn’s
and amyloidosis can cause decreased or increased folds.

How well did you know this?
1
Not at all
2
3
4
5
Perfectly
69
Q

A 44-year-old man presents with a vague history of central abdominal pain and mild weight
loss. On further questioning, there are other features in the history suggestive of malabsorption.
Amongst other investigations, a CT scan of abdomen is requested. This shows dilated fluid-
filled small bowel loops and multiple enlarged mesenteric lymph nodes, encasing the
mesenteric vessels. The lymph nodes are of homogeneous soft tissue density. What is the most
likely cause of the CT findings? [B1 Q59]

A. Whipple’s disease.
B. Coeliac disease complicated by lymphoma.
C. Cavitating mesenteric lymph node syndrome.
D. Abdominal tuberculosis.
E. Castleman disease.

A

Coeliac disease complicated by lymphoma
Whipples disease, cavitating mesenteric lymph node syndrome, and abdominal TB more
typically have mesenteric lymph node enlargement that has central low attenuation, rather than
being of homogeneous soft-tissue density. Whipple disease is a systemic bacterial infection
caused by Tropheryma whippelii. Lymph nodes affected by Whipple disease typically have a
high fat content, causing the low attenuation, usually between 10 and 20 HU.

Cavitating mesenteric lymph node syndrome is associated with coeliac disease. The lymph
nodes are truly cavitating and usually regress following a gluten-free diet.

The lymph nodes in abdominal tuberculosis typically have caseous necrosis and thus central
low density on CT.

Castleman disease causes benign masses of lymphoid tissue of unknown aetiology. It can cause
mesenteric lymphadenopathy, which is homogeneous, but the disease itself is rare and
mesenteric involvement is much less common than mediastinal involvement.

How well did you know this?
1
Not at all
2
3
4
5
Perfectly
70
Q

A 45-year-old man has a long history of intermittent diarrhoea, abdominal bloating, and cramps,
but has neglected to seek medical advice until now. His GP is worried about undiagnosed
Crohn’s disease and sends him for a small bowel series. This shows some dilatation of the
proximal small bowel, with segmentation and flocculation of the barium and an increased
number of normal thickness folds seen in the ileum. There is no evidence of stricture formation
or ulceration. What is the most likely underlying diagnosis? [B1 Q72]

A. Amyloidosis.
B. Chronic ischaemic enteritis.
C. Whipple’s disease.
D. Coeliac disease.
E. Lymphoma.

A

Coeliac disease

The segmentation and flocculation of barium are findings on a small bowel series that are
typical of malabsorption and therefore the most likely diagnosis is coeliac disease. Other findings in coeliac disease include dilatation, a granular appearance to the barium secondary to
hypersecretion, jejunization of the ileum, and the ‘moulage’ sign. The latter refers to a smooth,
tubular appearance to the jejunum in longstanding coeliac disease, secondary to atrophy and
effacement of the jejunal mucosal folds.

Lymphoma can be a complication of coeliac disease and generally causes shallow, ulcerated
masses or the development of thickened, nodular small bowel folds.

How well did you know this?
1
Not at all
2
3
4
5
Perfectly
71
Q

Whipple’s disease, amyloidosis, and chronic ischaemic enteritis all cause thickening of the
small bowel folds.

A 31-year-old male is investigated as an outpatient for diarrhoea. A small bowel meal study
reveals jejunal dilatation with thickened valvulae conniventes. In the ileum an increased
number of mucosal folds are seen. Which of the following diagnoses is most likely? [B2 Q27]

a. Lymphoma
b. Crohn’s disease
c. Coeliac disease
d. Whipple disease
e. Behcet syndrome

A

Coeliac disease
Jejunal dilatation and jejunisation of the ileal loops are characteristic features of coeliac disease.
This is an immunological intolerance to gluten that causes villous atrophy in the small intestine.
In Whipple disease there is thickening of the jejunal and duodenal mucosal folds but typically
no luminal dilatation. Dilatation of the small bowel does occur with lymphoma but jejunisation
of the ileum is not a feature.

How well did you know this?
1
Not at all
2
3
4
5
Perfectly
72
Q

A 34-year-old female is investigated for intermittent abdominal pain and malabsorption. Small
bowel meal shows dilatation of the proximal small bowel loops but a normal mucosal fold
pattern. Which one of the following is the most likely underlying diagnosis? [B2 Q35]

a. Coeliac disease
b. Amyloid
c. Whipple disease
d. Giardiasis
e. Eosinophilic gastroenteritis

A

Coeliac disease
All of these may cause malabsorption. Amyloid can cause dilatation but also causes diffuse
thickening of the valvulae conniventes throughout the small bowel. With Whipple disease and
eosinophilic gastroenteritis, one would not see dilatation of the bowel, but thickening of the
mucosa is again a prominent feature. Giardiasis causes thickening and marked distortion of the
mucosal folds in the duodenum and jejunum. One of the hallmark features of untreated coeliac
disease is jejunal dilatation. Typically, the mucosal folds are of normal thickness.

How well did you know this?
1
Not at all
2
3
4
5
Perfectly
73
Q

A 34-year-old woman with a history of steatorrhoea and weight loss undergoes a small bowel
follow-through examination that demonstrates dilatation of the proximal small bowel with flocculation and segmentation of the barium column. Fold thickness is normal. What is the
most likely diagnosis? [B4 Q5]

a. Crohn’s disease
b. Zollinger–Ellison syndrome
c. coeliac disease
d. small bowel lymphoma
e. Whipple’s disease

A

Coeliac disease
Coeliac disease is characterized by malabsorption due to intolerance to the alpha-gliadin
component of gluten, which causes small intestinal villous atrophy. Typical findings are of
dilatation of the proximal small bowel, together with dilution of the barium column due to
hypersecretion of fluid. Artefacts such as segmentation (breaking up of the barium column) or
flocculation (clumping of disintegrated barium) were traditionally classic features of coeliac
disease but are less often seen nowadays with improved barium suspensions. In Whipple’s
disease the small bowel is typically non-dilated and shows moderate fold thickening. Crohn’s
disease usually causes nodular fold thickening, and predominantly involves the distal small
bowel. Zollinger–Ellison syndrome results in dilatation of proximal small bowel due to
hypersecretion, but typically causes thickened folds. Small bowel lymphoma is usually
associated with fold thickening.

How well did you know this?
1
Not at all
2
3
4
5
Perfectly
74
Q

A 60-year-old woman presents with weight loss and diarrhoea. CT of the abdomen
demonstrates multiple, enlarged, low-attenuation mesenteric lymph nodes containing fat–fluid
levels and splenic atrophy. What is the most likely diagnosis? [B4 Q55]

a. tuberculosis
b. coeliac disease
c. Whipple’s disease
d. lymphoma
e. metastatic squamous cell carcinoma

A

Coeliac disease
Cavitating mesenteric lymph node syndrome is a rare complication of coeliac disease, in which
multiple enlarged lymph nodes are seen in the jejunoileal mesentery. The nodes have central
low attenuation and may contain fat or fluid, or fat–fluid levels. Splenic atrophy is usually seen,
and jejunal or duodenal biopsy confirms villous atrophy of the small bowel mucosa. Low-
attenuation lymphadenopathy may also be seen in tuberculosis, Whipple’s disease, lymphoma,
and necrotic metastases, but fat–fluid levels have been reported only in coeliac disease

How well did you know this?
1
Not at all
2
3
4
5
Perfectly
75
Q

A 68-year-old male patient has a 20-year history of RA. During a recent flare he was
commenced on steroid therapy, although this has now been discontinued. The patient is now
complaining of mild abdominal discomfort, diarrhoea, and mild weight loss. A barium meal is
performed, but is suboptimal, as the patient is poorly mobile. Within the limitations of the study,
there is reduced peristalsis in the oesophagus and mild reflux. The antrum of the stomach is
felt to be mildly narrowed and rigid. Thickened rugal folds are noted. A subsequent small bowel
series is carried out. The jejunal folds measure 4 mm and the ileal folds appear more plentiful,
and measure 3 mm. Contrast is present in the caecum at 4 hours. Spot screening of the terminal
ileum reveals the same findings as those described above. What is the most likely diagnosis?
[B1 Q16]

A. Gastric erosions.
B. Whipple’s disease.
C. Mastocytosis.
D. Amyloidosis.
E. Crohn’s disease.

A

Amyloidosis.

This patient probably has amyloidosis secondary to prolonged RA. GI involvement is more
common in primary (70%) than secondary (13%) amyloidosis. Nevertheless, the small bowel
is involved in 74% of cases of GI amyloidosis and secondary amyloidosis is the most common
type of amyloid disease. Amyloidosis is secondary to the deposition of insoluble amyloid
protein in soft tissues and organs. In primary amyloidosis the heart (90%), followed by the
small bowel and the lungs (70%), are the most affected organs. The kidneys are affected in 90%
of cases of secondary amyloidosis. Amyloidosis classically causes a diffuse thickening of
bowel folds. It may cause dilated bowel folds if the myenteric plexus is involved. The main
differential for amyloid is Whipple’s disease and intestinal lymphangiectasia. Whipple’s
disease does not cause bowel dilatation or rigidity, as described in the antrum in this patient.
Crohn’s disease can also present with thickened folds, but it is more commonly focal with the
most pronounced abnormality in the terminal ileum. Ulceration is also commonly seen in
Crohn’s, but 68 years old would be a late first presentation for Crohn’s. Whilst option A is true,
this is not what the question asked. Patients with mastocytosis most commonly present in
infancy .

GI Amyloidosis [STATdx]

Fluoroscopic findings:

  • Stomach
    o Diffuse or focal thick folds (nodular or mass-like) with calcification
  • Small intestine
    o Symmetrical regular thickening of folds (with or without nodular component)
    o Impaired motility often with small bowel dilatation, and
    o Slow transit
  • Colon
    o Luminal narrowing
    o Loss of haustrations, and o Thickened transverse folds

CT findings:

  • Small bowel, large bowel, and stomach
    o Wall thickening – focal/nodular mass-like or diffuse, calcified, may infiltrate
    into mesentery
    o distention (due to hypomotility)
    o intramural haemorrhage,
    o luminal narrowing (due to wall thickening), and
    o intussusceptions
  • Liver and spleen
    o Hepatomegaly or splenomegaly may be only manifestation
    o Focal, geographic, or diffuse hypoattenuation in liver (mimic steatosis)
    o Frequently in periportal distribution (with parenchymal calcification)
  • Spleen
    o Splenomegaly
    o Focal or diffuse hypoattenuation (with parenchymal calcification)
    o Increased risk of spontaneous rupture
  • Gallbladder wall thickening (extremely rare)
  • Retroperitoneal soft tissue infiltration resembling retroperitoneal fibrosis
  • Mesenteric infiltration resembling sclerosing mesenteritis
  • Lymphadenopathy with calcification
  • Mass-like amyloidomas in soft tissues with calcification

MRI findings:
Involved site – Low T2, intermediate to high T1

Whipple Disease [Core Radiology]

Def: – A rare systemic disease caused by Tropheryma whipplei, multiple systemic
manifestations, including chronic diarrhoea and malabsorption

Imaging:

  1. Fluoroscopic small bowel follow-through or enteroclysis
    * Common site – Distal duodenum and jejunum
    * Severe cases – Distal Ileum
    * Thickened, irregular folds with sand-like micronodules
    * Normal or mildly dilated lumen
    * Thickened mesentery and separation of bowel loops.
  2. CT
    * Mesenteric and retroperitoneal lymphadenopathy with near fat density. (Digested bacilli)
    * Thickened proximal small bowel folds with or without submucosal oedema due to
    hypoalbuminemia * Ascites, splenomegaly, pneumatosis intestinalis.
  3. MR
    Lymph nodes may show increased T1 signal due to fat

Top Differential Diagnoses:

  • Celiac disease
  • Intestinal opportunistic infections
  • Dysgammaglobulinemia
  • Intestinal metastases and lymphoma

Clinical Issues:

  • Mostly affects middle-aged White men
  • Prodromal phase: Fevers, fatigue, arthralgias, arthritis
  • Late phase: Diarrhea, malabsorption, steatorrhea, adenopathy, abdominal pain
  • GI symptoms generally later manifestation of disease
  • Can be fatal without therapy (long-term antibiotics)
  • Clinical symptoms often subside quickly after therapy
How well did you know this?
1
Not at all
2
3
4
5
Perfectly
76
Q

A 45-year-old man, with a history of AIDS, has a 3-month history of abdominal pain and
weight loss. A CT scan of abdomen is performed which shows ascites with peritoneal
thickening, several areas of mural thickening in the small bowel, and multiple low attenuation
lymph nodes. Which one of the following infections is most likely? [B1 Q59]

A. CMV infection.
B. TB.
C. Cryptosporidiosis.
D. Amoebiasis.
E. Campylobacter.

A

TB.

Cryptosporidiosis is the most common cause of enteritis in AIDS patients. It more commonly
causes proximal small bowel thickening in the duodenum and jejunum, and CT may show
small lymph nodes.

CMV infection of the small bowel can show a terminal ileitis indistinguishable from Crohn’s
disease.
The typical CT findings in amoebiasis are thickening of the right colonic wall and a rounded
abscess in the right lobe of liver with a peripheral zone of oedema.

TB usually shows ileocaecal involvement, low attenuation mesenteric nodes, and ascites with
peritoneal thickening. Mycobacterium avium intracellulare may also occur with low
attenuation mesenteric nodes and thickening of small bowel folds.

How well did you know this?
1
Not at all
2
3
4
5
Perfectly
77
Q

What is the most common site of involvement in tuberculosis of the gastrointestinal tract? [B4
Q83]

a. stomach
b. duodenum
c. ileo-caecal region
d. splenic flexure
e. rectum

A

Ileo-caecal region
Tuberculosis of the gastrointestinal tract may occur through ingestion of infected sputum, or
by haematogenous spread to submucosal lymph nodes from a pulmonary tuberculous focus. It
most commonly affects the ileo-caecal region due to its abundance of lymphoid tissue and
relative stasis of gut contents. Typical features at this site include circumferential thickening
of the terminal ileum and caecum, a thickened ileo-caecal valve and ulceration following the
orientation of lymphoid follicles (longitudinal in the terminal ileum and transverse in the colon).
Marked enlargement of adjacent mesenteric lymph nodes with central areas of low attenuation
may be seen

How well did you know this?
1
Not at all
2
3
4
5
Perfectly
78
Q

A 38-year-old patient with AIDS presents with diarrhoea and steatorrhoea. As part of the work-
up, small bowel enteroclysis shows thickened jejunal folds with nodularity and evidence of marked jejunal spasm. The ileum has normal appearances. Which one of the following is the
most likely underlying cause? [B2 Q32]

a. Cytomegalovirus
b. Tuberculosis
c. Mycobacterium avium intracellulare
d. Cryptosporidium
e. Giardiasis

A

Giardiasis

All the stems are potential causes for these symptoms in a patient with AIDS, however
giardiasis is the most likely cause given these imaging appearances. Cytomegalovirus most
typically affects the caecum, and tuberculosis affects the caecum and ileocaecal valve.
Mycobacterium avium intracellulare can affect the ileum and jejunum but does not usually
cause spasm. Cryptosporidium affects the duodenum, and the jejunum can be affected, but
dilatation is more common than spasm.

How well did you know this?
1
Not at all
2
3
4
5
Perfectly
79
Q

A 55-year-old woman is admitted to hospital after several episodes of melaena. She has an
upper GI endoscopy performed, which is normal. A CT scan of abdomen is requested, and this
demonstrates a large exophytic mass arising from the jejunum in the left upper quadrant. It is
heterogeneous in density and has some peripheral enhancement and central necrosis. There is
no calcification, intestinal obstruction, or evidence of aneurysmal dilatation of the affected
segment of jejunum. There is no adjacent lymphadenopathy or ascites. What is the most likely
diagnosis? [B1 Q66]

A. Adenocarcinoma.
B. Lymphoma.
C. Carcinoid tumour.
D. Metastasis.
E. Gastrointestinal stromal tumour (GIST).

A

Gastrointestinal stromal tumour (GIST)
At contrast-enhanced CT, GISTs appear as large exophytic masses with peripheral
enhancement. They usually have an attenuation like that of muscle, but they may have
heterogeneous attenuation, depending on their level of aggressiveness. More aggressive GISTs
may also contain a central area of necrosis.

Adenocarcinoma of the jejunum is rare, more commonly occurring in the duodenum. They also
tend to be stricturing lesions, rather than exophytic masses and may present with obstruction.

Carcinoid is also rare in the proximal small bowel, the distal ileum being a more usual location.
The primary lesion is often quite small, with the nodal metastatic lesion in the small bowel
mesentery being more conspicuous on CT. This is often spiculated (surrounding desmoplastic
reaction) and may contain calcification.

Lymphoma can have several manifestations in the small bowel, from nodular thickening of the
mucosal folds to large masses with aneurysmal dilatation of the small bowel in the affected
segment. Associated lymphadenopathy is typical.

How well did you know this?
1
Not at all
2
3
4
5
Perfectly
80
Q

A 57-year-old diet-compliant male patient with coeliac disease has a CT abdomen and pelvis
for the investigation of cachexia and two stone weight loss over six months. A 7 cm segment
of ileum shows mild dilatation and circumferential thickening, with multiple low-attenuation
mesenteric and para-aortic lymph nodes. Which one of the following is the most likely
diagnosis? [B2 Q59]

a. Tuberculosis
b. Gastro-intestinal lymphoma
c. Coeliac disease
d. Whipple disease
e. Crohn’s disease

A

**Gastro-intestinal lymphoma **
Hypoattenuating lymph nodes can be attributed to many causes, but lymphoma and
tuberculosis are the most common. Lymphoma of the gastro-intestinal tract most commonly
affects the ileum, although lymphoma associated with coeliac disease most commonly affects
the jejunum. Although 90% of tuberculosis of the gastro-intestinal tract occurs in the ileum,
lymphoma is most likely in this scenario. Dilatation of the small bowel with lymphoma is
common but obstruction is rare due to the soft pliable nature of the tumour.

How well did you know this?
1
Not at all
2
3
4
5
Perfectly
81
Q

A 25-year-old male presents following blunt abdominal trauma following a motor vehicle
accident. Which is the most common CT finding in the ‘shock bowel’? [B3 Q8]

A. Increased small bowel mucosal enhancement
B. Small bowel luminal dilation
C. Fluid-filled loops of small bowel
D. Colonic involvement greater than small bowel
E. Focal involvement of the small bowel

A

Increased small bowel mucosal enhancement (HU > psoas) and mural thickening > 3mm are
the most common signs. Colon involvement is infrequent, and the small bowel is typically
diffusely involved.

How well did you know this?
1
Not at all
2
3
4
5
Perfectly
82
Q

A 73-year-old woman is referred from surgical outpatients for a barium enema. She has a 3-
month history of weight loss and a microcytic anaemia. The procedure is unremarkable, and
you leave the screening room to go and continue some plain film reporting. Ten minutes later
you are contacted by one of the radiographers who was helping during the enema. She is
distressed and tells you that she found the patient collapsed in the bathroom having what
appeared to be a seizure. You immediately attend and assess the patient. She is drowsy, but
heart rate, blood pressure, and SaO 2 are normal. What is the most likely complication to have
caused her acute illness? [B1 Q43]

A. Cardiac arrhythmia secondary to rectal distension.
B. Venous intravasation.
C. Water intoxication.
D. Intramural barium.
E. Side-effect of hyoscine butyl bromide (Buscopan).

A

Water intoxication.
All five options are complications of barium enema. Additional potential complications include
bowel perforation, barium impaction, and transient bacteraemia. Complications during barium
enema are rare. Perforation of the bowel is the most frequent serious complication, occurring
in approximately 0.02–0.04% of patients. Venous intravasation may result in a barium
pulmonary embolus, which carries an 80% mortality. Water intoxication causes drowsiness
and convulsions, as in this case. There is an increased risk in megacolon because of the large
area of bowel mucosa available for the absorption of water. Water intoxication has also been
attributed to the preparatory laxatives used. Buscopan may cause cardiac arrhythmia and
should be used with caution in those with cardiac disease; other relative contraindications
include angle-closure glaucoma, myasthenia gravis, paralytic ileus, pyloric stenosis, and
prostatic enlargement.

How well did you know this?
1
Not at all
2
3
4
5
Perfectly
83
Q

About the use of glucagon in barium enema examinations, which of the following statements
is correct? [B1 Q60]

A. 0.1mg of glucagon is an appropriate dose.
B. Diabetes is a contraindication to the use of glucagon.
C. Insulinoma is a contraindication to the use of glucagon.
D. Glucagon can be safely used in patients with phaeochromocytoma.
E. Smooth muscle relaxation is optimal at 5 minutes and lasts approximately 1 hour.

A

Insulinoma is a contraindication to the use of glucagon
Glucagon is a potent hypotonic agent. If 1mg of glucagon is injected intravenously it takes
approximately 1 minute to work and lasts about 10–20 minutes. Intravenously administered
glucagon decreases discomfort during barium enema examinations. Glucagon administration
is generally safe but is contraindicated in patients with insulinoma and phaeochromocytoma.
Diabetes is not a recognized contraindication.

How well did you know this?
1
Not at all
2
3
4
5
Perfectly
84
Q

A 72-year-old man attends for a barium enema examination. He has no known allergies. In
considering administration of intravenous hyoscine-N-butylbromide (Buscopan), which factor
in his medical history is it most important to be aware of? [B4 Q35]

a. prostatism
b. type I diabetes
c. glaucoma
d. migraine
e. unstable cardiac disease

A

Unstable cardiac disease
Buscopan is commonly used in radiological practice as a smooth muscle relaxant. As a non-
selective muscarinic antagonist, it produces other autonomic responses including pupillary
dilatation and tachycardia and may potentially precipitate an attack of acute angle-closure
glaucoma that requires prompt treatment to prevent permanent visual loss. However, most
glaucoma is of the open-angle form, which is unaffected by Buscopan, and it is therefore
advised that routine enquiry about a history of glaucoma is unnecessary. Instead, patient
information leaflets should advise all patients to attend hospital immediately should they
develop painful, blurred vision. Routine enquiry about prostatism, porphyria and myasthenia
gravis are also not recommended. However, in patients with unstable cardiac disease, the
tachycardia and slight increase in diastolic blood pressure caused by Buscopan carry the
potential risk of arrhythmia. The presence of unstable cardiac disease is therefore deemed to
be the only potential reason to withhold Buscopan.

How well did you know this?
1
Not at all
2
3
4
5
Perfectly
85
Q

A 75-year-old man is undergoing a CT colonography examination for investigation of a change
in bowel habit. He has difficulty retaining the CO2 for adequate bowel distension. Which of
the following segments of colon is likely to be better distended on the prone scan? [B1 Q69]

A. Caecum.
B. Transverse colon.
C. Rectosigmoid.
D. Ascending colon.
E. Hepatic flexure.

A

Rectosigmoid.

On a prone scan during a CT colonography examination, the rectosigmoid is generally better
distended than on the supine scan because it is a more posteriorly placed structure and air gets
displaced to the non-dependent position. The other segments named are usually better
distended on the supine scan, particularly the caecum and transverse colon, as these are more
anteriorly placed within the abdomen

How well did you know this?
1
Not at all
2
3
4
5
Perfectly
86
Q

Which is the most common source of false positives in cathartically prepared CT colonography
by Computer aided detection (CAD)? [B3 Q33]

A. Haustral folds
B. Untagged/poorly tagged stool
C. Ileocecal valve D. Electronic cleansing and tagging artefact
E. Extrinsic compression

A

Haustral folds
Most common source of FP in CAD in cathartically prepared CT colonography.
Untagged/poorly tagged stool is the second most common FP in cathartically prepared and
most common source of FP in non-cathartically prepared CT colonography

How well did you know this?
1
Not at all
2
3
4
5
Perfectly
87
Q

A 37-year-old man on the intensive care unit, 3 weeks post-operatively for complicated small
bowel resection for Crohn’s disease, is noted to have an oozing from a site on his anterior
abdominal wall. His renal function remains normal and inflammatory markers are persistently
elevated with White Cell Count (WCC) 15.9 and C-reactive Protein (CRP) 124. On a CT of
the abdomen 5 days ago, no collection was demonstrated. CT fistulography is being considered.
Which is the best answer with regards to CT fistulography? [B3 Q42]

A. Conventional fistulography has a higher spatial resolution
B. Iodinated contrast should be used, diluted 1 in 100
C. Oral contrast is helpful in most cases
D. Intravenous (IV) contrast should be avoided
E. CT fistulography has a greater temporal resolution

A

Conventional fistulography has a higher spatial resolution
CTF helps to delineate fistulous tracks in critically ill patients where prolonged contrast studies
are not feasible. Iodinated contrast (300mg iodine/ml) should be diluted 1 in 10, with a volume
based on an estimate of the length of bowel or volume of cavity being investigated. Oral
contrast can confuse the origin of intraluminal contrast and is best avoided, whereas IV contrast
is helpful. Conventional fistulography has a higher spatial and temporal resolution.

How well did you know this?
1
Not at all
2
3
4
5
Perfectly
88
Q

A 54-year-old woman with a sensation of incomplete evacuation on defecation undergoes
conventional defecography. Following introduction of barium paste into the rectum, in which
position should the patient be placed for imaging? [B4 Q7]

a. supine
b. prone
c. left lateral
d. right lateral
e. sitting

A

Conventional defecography is used in the imaging evaluation of obstructed defecation. Barium
paste is instilled into the rectum with a Foley catheter, with the patient in the left lateral position
Prior opacification of the small bowel, bladder, vagina or peritoneum may also be performed
to aid diagnosis. With the patient sitting on a commode placed on the footrest of a standard
fluoroscopic table, static images are first obtained at rest and with contraction of the pelvic floor
muscles. A cine-loop of evacuation is then obtained until the rectum is empty or three 30-
second attempts at evacuation have been made. A variety of conditions may be demonstrated,
including rectocele, enterocele, rectal intussusception and anismus.

How well did you know this?
1
Not at all
2
3
4
5
Perfectly
89
Q

During double-contrast barium enema, a prone overcouch film with the tube angled 45’
caudally, centred 5 cm above the posterior superior iliac spines, is performed to optimally
visualize which segment of the large bowel? [B4 Q92]

a. caecum
b. hepatic flexure
c. transverse colon
d. splenic flexure
e. sigmoid colon

A

Sigmoid colon
The prone-angled, over-couch view performed as described separates overlying loops of
sigmoid colon

How well did you know this?
1
Not at all
2
3
4
5
Perfectly
90
Q

A 73-year-old woman presents with intermittent lower gastrointestinal bleeding and iron
deficiency anaemia. She is clinically suspected to have angiodysplasia. What are the most
likely findings on barium enema? [B4 Q73]

a. normal appearances
b. multiple small polyps in the colon
c. multiple shallow ulcers in the colon
d. multiple, serpiginous, filling defects in the colon
e. a focal, irregular, circumferential narrowing in the colon

A

Normal appearance
In angiodysplasia, there is degenerative dilatation of the normal vessels in the submucosa of
the bowel wall. It is associated with increasing age, and in about 20% of cases with aortic
stenosis. It occurs most commonly in the right colon and presents with intermittent, low-grade
bleeding. Barium enema shows no abnormality, as the lesion is submucosal, but increased
tracer accumulation may be seen at the site of haemorrhage on 99m Tc-labelled red cell
scanning. Angiography, if performed, may demonstrate a cluster of vessels along the
antimesenteric border during the arterial phase and early opacification of the draining ileocolic
vein.

How well did you know this?
1
Not at all
2
3
4
5
Perfectly
91
Q

In the staging of rectal cancer by MRI, which sequence provides optimum visualization of the
tumour? [B4 Q80]

a. T1W
b. contrast enhanced T1W
c. T2W
d. FLAIR
e. proton density

A

T2W

MR is a highly accurate method of local staging of rectal cancer, with better assessment of
locoregional nodal involvement than CT and clear depiction of the mesorectal fascia, allowing
accurate prediction of whether the circumferential resection margin will be tumour free. T2W images provide optimal visualization of the tumour, which appears as an intermediate signal-
intensity mass. Contrast-enhanced T1W images result in enhancement of the normal bowel
wall as well as the tumour, which may lead to upstaging. FLAIR sequences are not routinely
used for rectal cancer staging.

How well did you know this?
1
Not at all
2
3
4
5
Perfectly
92
Q

Ischaemic Colitis

A 78-year-old previously well female is admitted with acute abdominal pain and diarrhoea.
Contrast-enhanced CT of the abdomen and pelvis shows thickening of a 13cm segment of
proximal descending colon and mucosal hyperenhancement. The rest of the colon is normal,
and the small bowel is unaffected. There is a small amount of free fluid in the pelvis. Which
one of the following diagnoses is most likely? [B2 Q14]

a. Crohn’s colitis
b. Ulcerative colitis
c. Ischaemic colitis
d. Infectious colitis
e. Pseudomembranous colitis

A

Ischaemic colitis
Crohn’s colitis is relatively unlikely due to lack of prior history or small bowel involvement
and age of the patient. Ulcerative colitis and pseudomembranous colitis are both unlikely as
the rectum is usually involved in these two conditions
. Infectious colitis does not normally
affect the left-sided colon only, regardless of the underlying pathogen. Ischaemic colitis is the
most likely diagnosis of those listed. It typically affects a segment of bowel, with most cases
having left-sided colonic involvement.

How well did you know this?
1
Not at all
2
3
4
5
Perfectly
93
Q

A 64-year-old man presents to A&E with onset of severe watery diarrhoea and abdominal pain.
An AXR is performed which shows dilated large bowel and nodular haustral fold thickening.
The patient has a CT scan with oral and intravenous contrast. The CT scan shows large bowel
dilatation with diffuse bowel wall thickening. Some of the oral contrast given has become
trapped between the oedematous haustral folds, causing alternating bands of high and low
attenuation. What is the most likely underlying diagnosis? [B1 Q55]

A. Ulcerative colitis.
B. Crohn’s colitis.
C. Ischaemic colitis.
D. Pseudomembranous colitis.
E. Bacillary dysentery.

A

Pseudomembranous colitis
The CT sign of oral contrast becoming trapped between thickened oedematous haustral folds
is the ‘accordion’ sign, which is highly suggestive of pseudomembranous colitis. Bowel wall
thickening, peri-colonic stranding of fat, and ascites may be seen in all forms of colitis.
Pseudomembranous colitis can be segmental but is more commonly a pancolitis.

How well did you know this?
1
Not at all
2
3
4
5
Perfectly
94
Q

A 76-year-old male on ITU has a CT abdomen and pelvis for the investigation of abdominal
pain, pyrexia, and diarrhoea. The CT reveals 12mm diffuse large bowel wall thickening with
intense mucosal enhancement and low attenuation of the submucosa involving the entire colon
including the rectum, and a small volume of ascites. Which one of the following diagnoses is
the most likely to explain the above findings? [B2 Q47]

a. Crohn’s colitis
b. Pseudomembranous colitis
c. Ischaemic colitis
d. Yersinia
e. Giardiasis

A

Pseudomembranous colitis
Pseudomembranous colitis results from overgrowth of Clostridium difficile most commonly
due to broad spectrum antibiotic use in the hospital population. Ascites is often present in
severe cases and wall thickening >10 mm is highly suggestive of this diagnosis. A layered
pattern of enhancement is often present in severe cases with oedema in the submucosa
producing low attenuation in the wall, deep to the enhancing mucosa. The accordion sign is
caused by marked submucosal oedema producing thickening of the colonic haustra. The rectum
is involved in most cases but any location within the large bowel may be involved.

How well did you know this?
1
Not at all
2
3
4
5
Perfectly
95
Q

A 60-year-old woman presents with abdominal pain and diarrhoea six weeks post-hip
replacement surgery, with her recovery being complicated by a hospital-acquired pneumonia.
CT is performed. When considering a diagnosis of pseudomembranous colitis (PMC) which is
the most common finding? [B3 Q5]

A. Intense mucosal enhancement
B. Enlarged peri-colic blood vessels
C. An irregular or discontinuous mucosal lining
D. Peri-colonic fat stranding
E. Colonic wall thickening

A

Although all the above are recognised findings in PMC, colonic wall thickening from minor to
gross thickening is the most common feature. The relatively minimal pericolonic stranding with
marked colonic wall thickening
may help to distinguish PMC from other colonic pathologies.

How well did you know this?
1
Not at all
2
3
4
5
Perfectly
96
Q

A 70-year-old hospitalized male patient presents with watery diarrhoea and abdominal pain.
CT of the abdomen demonstrates marked circumferential bowel wall thickening involving the
entire colon, with minimal peri-colonic stranding and a small amount of ascites. The small
bowel appears normal. What is the most likely diagnosis? [B4 Q2]

a. Crohn’s disease
b. ischaemic colitis
c. diverticulitis
d. pseudomembranous colitis
e. ulcerative colitis

A

Pseudomembranous colitis
Pseudomembranous colitis is an acute infectious colitis caused by Clostridium difficile and its
toxins A and B; this pathogen has become increasingly common largely due to widespread use
of broad-spectrum antibiotics. The commonest CT finding is of colonic wall thickening (due
to mural oedema and the presence of pseudomembranes), which is typically greater than in
other causes of colitis apart from Crohn’s disease. Pericolonic inflammatory changes are
disproportionately mild relative to the marked wall thickening. Ascites is common, and this,
together with the lack of small bowel involvement, can help to distinguish pseudomembranous
colitis from Crohn’s colitis. Ischaemic colitis demonstrates less wall thickening, and is usually
segmental, tending to affect the watershed areas of the colon.

How well did you know this?
1
Not at all
2
3
4
5
Perfectly
97
Q

A 35-year-old male with known ulcerative colitis presents to A&E with severe abdominal pain,
pyrexia, and diarrhoea. There is no peritonism. Toxic megacolon is suspected clinically. Which
one of the following is the most appropriate as first line imaging? [B2 Q30]

a. CT
b. Plain abdominal film
c. Double contrast barium enema
d. Single contrast water-soluble enema
e. Targeted bowel ultrasound

A

Plain abdominal film
Toxic megacolon is a complication of ulcerative colitis, Crohn’s and other forms of acute
colitis. It has a poor prognosis with up to 20% mortality. Plain abdominal radiography should
be the first line investigation for suspected toxic megacolon and can be repeated 24 or 48 hourly
if necessary. It can often confirm the diagnosis without the need for CT, which is especially
useful when considering radiation dose issues in this group of young patients. Typical features
on plain film include transverse colon dilatation >5.5 cm, loss of normal haustral folds,
thumbprinting of the colon and the presence of mucosal islands (pseudopolyps). CT better
demonstrates potential complications of toxic megacolon such as perforation of the bowel.

How well did you know this?
1
Not at all
2
3
4
5
Perfectly
98
Q

As part of an investigation for altered bowel habit, a 32-year-old female has a double contrast
barium enema performed. Findings include deep and superficial aphthous ulceration from the
caecum proximally to the sigmoid colon and the presence of pseudodiverticula. Which one of
the following is most likely? [B2 Q31]

a. Crohn’s disease
b. Ulcerative colitis
c. Tuberculosis
d. Yersinia
e. Lymphoma

A

Crohn’s disease
These features are highly suggestive of Crohn’s disease. Signs on double contrast barium
enema that favour a diagnosis of Crohn’s disease over ulcerative colitis include apthoid ulcers,
deep ulcers, discontinuous ulceration, rectal sparing, pseudodiverticulae, fistulae and abscess
formation.
Ulcerative colitis can be suggested by rectal involvement, continuous pathology
with no skip lesions and the presence of mucosal granularity. However, these features may also
be present in Crohn’s and are not specific for ulcerative colitis. Although tuberculosis is a
mimic, colonic involvement in this pattern is uncommon compared with Crohn’s disease.

How well did you know this?
1
Not at all
2
3
4
5
Perfectly
99
Q

A 34-year-old woman presents with bloody diarrhoea and abdominal pain. Which feature on
barium enema favours a diagnosis of ulcerative colitis rather than Crohn’s disease? [B4 Q29]

a. thickened ileocaecal valve
b. circumferential wall involvement
c. fistula formation d. skip lesions
e. normal rectum

A

Circumferential wall involvement
Ulcerative colitis and Crohn’s disease are idiopathic inflammatory diseases of the bowel.
Ulcerative colitis predominantly involves the mucosa and submucosa, and characteristically
produces continuous, circumferential involvement of the colon. Crohn’s disease produces
transmural inflammation, may affect the entire gastrointestinal tract, and is characterized by
eccentric and discontinuous involvement. Typical features of ulcerative colitis include
predominantly left-sided colonic involvement with rectosigmoid involvement in 95% of cases,
a patulous ileo-caecal valve and shallow ulceration. Typical features of Crohn’s disease include
skip lesions (discontinuous disease), terminal ileal involvement with a thickened ileocaecal
valve and fistula formation.

How well did you know this?
1
Not at all
2
3
4
5
Perfectly
100
Q

A seven-year-old boy on chemotherapy for acute leukaemia develops severe right iliac fossa
pain and diarrhoea. CT shows ascending colon and caecal wall thickening, with inflammation
extending to involve the appendix and terminal ileum and fat stranding in the adjacent
mesentery. The most likely diagnosis is: [B2 Q6]

a. Typhlitis
b. Crohn’s disease
c. Acute appendicitis
d. Necrotising enterocolitis
e. Acute leukaemic infiltration

A

Typhlitis

Typhlitis, or neutropaenic enterocolitis, is acute inflammation of the caecum, ascending colon,
terminal ileum, or appendix. It is typically described in children with neutropaenia secondary
to lymphoma, leukaemia, and immunosuppression. Concentric, often marked, bowel wall
thickening with pericolic inflammatory changes is typical, and such changes in a young,
immunosuppressed child should raise suspicion of typhlitis as a cause. Perforation is a risk
factor and therefore contrast examinations are usually avoided.

How well did you know this?
1
Not at all
2
3
4
5
Perfectly
101
Q

A 45-year-old woman with a previous history of treatment for advanced carcinoma of the
cervix 8 years earlier presents with constipation and rectal bleeding. She undergoes CT of the
abdomen and pelvis, which demonstrates a narrowed rectum with symmetrical wall thickening,
perirectal inflammatory changes, thickening of the perirectal fascia and an increase in the AP
diameter of the presacral space. What is the most likely diagnosis?

a. colorectal carcinoma
b. ulcerative colitis
c. radiation injury of the rectum
d. Hirschsprung’s disease
e. lymphoma

A

Radiation injury of the rectum
Gastrointestinal complications following external radiotherapy are becoming more frequent as
survival rates of patients with abdominal cancer improve, and they may present up to 15 years
following irradiation. The colon and rectum are commonly affected following irradiation for
pelvic and genitourinary tract malignancies. Chronic radiation colitis usually presents with
strictures, whereas rectal injury manifests as a narrowed, thickened, poorly distensible rectum,
with proliferation of the perirectal fat and thickening of the perirectal fascia. There is an
increased incidence of colorectal carcinoma following pelvic irradiation, but a short irregular
segment of narrowing would be more likely. Inflammatory bowel disease may result in
circumferential rectal wall thickening with widening of the presacral space but is less likely
given this history. Hirschsprung’s disease usually presents in early childhood. The rectum is
an uncommon site for gastrointestinal lymphoma

How well did you know this?
1
Not at all
2
3
4
5
Perfectly
102
Q

Familial Adenomatous Polyposis

A 20-year-old male with a recent history of medulloblastoma now presents with vague
abdominal pain, PR bleeding, and weight loss. Innumerable colonic polyps are demonstrated
on colonoscopy. What is the most likely unifying diagnosis? [B1 Q74]

A. Familial adenomatous polyposis.
B. Turcot syndrome.
C. Gardner syndrome.
D. Lynch syndrome.
E. Chronic inflammatory bowel disease

A

Turcot syndrome
FAP is a rare autosomal dominant condition resulting in the growth of hundreds of
adenomatous polyps in the large bowel. Clinical symptoms commence from the third decade
and include abdominal pain and PR bleeding. Colorectal cancer develops in almost all before
the age of 40 years. Turcot syndrome is characterized by the association of colonic polyps like
FAP and central nervous system tumours, typically medulloblastoma and glioblastoma
multiforme. The combination of intestinal polyposis (identical to FAP) and numerous osteomas
and epidermal cysts is typical of Gardner syndrome. Lynch syndrome is hereditary non-
polyposis colorectal cancer. Colorectal carcinoma occurs earlier than in the average population.
There is also an association with ovarian and endometrial malignancy. Chronic inflammatory
bowel disease is not associated with CNS malignancy.

How well did you know this?
1
Not at all
2
3
4
5
Perfectly
103
Q

A 25-year-old male presents with abdominal cramps and pain with rectal bleeding.
Colonoscopy is normal. CT enteroclysis is performed as part of the investigation, which reveals
multiple sessile polyps throughout the jejunum and ileum. Subsequent biopsies reveal these
polyps to be hamartomas. Which one of the following syndromes is he most likely to be
diagnosed with? [B2 Q19]

a. Peutz–Jeghers
b. Cowden’s
c. Turcot’s
d. Familial polyposis
e. Gardner’s

A

Peutz–Jeghers
Peutz–Jeghers syndrome is most consistent with these findings. It is an autosomal dominant
syndrome but often arises as a spontaneous mutation. Hamartomas are found throughout the
gastro-intestinal tract, except for the oesophagus. The polyps have almost no malignant
potential, but life expectancy is decreased due to associated cancers arising in the stomach,
duodenum, colon, and ovary. Gardner’s syndrome and familial polyposis are both associated
with small bowel adenomas in approximately 5% of cases. Cowden’s syndrome does involve
hamartomatous polyps, but these are typically rectosigmoid, and small bowel involvement is
not a feature. Small bowel polyps are not a feature of Turcot’s syndrome.

How well did you know this?
1
Not at all
2
3
4
5
Perfectly
104
Q

A 25-year-old woman presents with cramping abdominal pain and bleeding per rectum. On
examination she has mucocutaneous pigmentation of her mucous membranes and face. A
small-bowel follow-through examination demonstrates small-bowel intussusception. Which
other finding is most likely to be demonstrated? [B4 Q20]

a. separation and displacement of small bowel loops
b. localized outpouching of the antimesenteric border of the distal ileum
c. generalized irregular fold thickening
d. multiple filling defects in the small bowel
e. generalized dilatation of the small bowel

A

Multiple filling defects in the small bowel
Peutz–Jegher syndrome is characterized by multiple benign hamartomatous intestinal polyps
and mucocutaneous pigmentation. It is familial in 50% of cases, with an autosomal dominant
inheritance, and sporadic in 50%. It is the most common polyposis syndrome to involve the
small intestine, and frequently presents with intussusception. Typical findings are of multiple
hamartomatous polyps in the small bowel, and less commonly the colon and stomach. Patients
are at increased risk of gastrointestinal malignancy, but also of tumours of the pancreas, breast,
ovary, endometrium and testis.

How well did you know this?
1
Not at all
2
3
4
5
Perfectly
105
Q

A 17-year-old female undergoes screening colonoscopy and is found to have multiple
adenomatous polyps throughout the colon. OGD and biopsy reveal multiple hamartomas of the
stomach and duodenum. She subsequently has investigation for a painful jaw that reveals a
1cm round, discrete, dense lesion in the mandible. Which one of the following syndromes is
the most likely underlying diagnosis? [B2 Q20]

a. Lynch syndrome
b. Cronkhite–Canada syndrome
c. Familial adenomatous polyposis
d. Gardner’s syndrome
e. Peutz–Jegher syndrome

A

**Gardner’s syndrome **
Gardner’s syndrome is an autosomal dominant condition with colonic polyps present in all
patients. Small bowel, duodenal and stomach polyps are also a feature. Extra-intestinal features
include osteomas of membranous bone (typically the mandible as described in the question),
other soft-tissue tumours and periampullary carcinomas. Osteomas are not a feature of the other
conditions. Cronkhite–Canada syndrome and Peutz–Jegher syndrome are associated with
multiple hamartomatous polyps of the colon and stomach. Cronkhite-Canada syndrome is a
sporadic non-familial disorder. Lynch syndrome, or hereditary nonpolyposis colorectal
carcinoma (HNPCC), is associated with increased risk of colorectal adenomas and other
malignancies such as endometrial and other gastro-intestinal tract malignancies.

How well did you know this?
1
Not at all
2
3
4
5
Perfectly
106
Q

A 26-year-old female presents with a 1-day history of right iliac fossa (RIF) pain. She is mid-
cycle and prone to mittleschmerz-type pain but reports that this pain is more severe than
previously. Serum inflammatory markers are elevated. Clinical examination reveals tenderness
in the RIF, but no rebound. Due to the compounding gynaecological history, a CT is requested.
This reveals a thickened caecum and thickened appendix, which appears to have a defect in the
wall on the multiplanar reformatted images. There is a calcified density present in the orifice
of the appendix. There is a loculated fluid collection adjacent to the appendix, which has air
bubbles within it. There is also fluid in the pelvis. A perforated appendix is removed at surgery.
Which of the CT findings is most specific for detecting a perforated appendix? [B1 Q9]

A. Presence of a faecolith.
B. Identification of a wall defect.
C. Fluid in the pelvis.
D. Adjacent abscess formation.
E. Enlarged regional lymph nodes.

A

Adjacent abscess formation.
Abscess formation has been found to be the most specific finding in appendiceal perforation,
along with extraluminal gas and small bowel ileus. Abscess formation is also one of the least
sensitive findings. Regional mesenteric lymph nodes are the most sensitive but are reasonably
non-specific. A focal wall defect, if seen, is reasonably sensitive and specific. Appendicolith is
only found in 50% on CT and has a specificity of 70%.

How well did you know this?
1
Not at all
2
3
4
5
Perfectly
107
Q

An ultrasound of the abdomen is performed on a 21-year-old female presenting to A&E with
acute right iliac fossa pain, pyrexia, tenderness and guarding. Which one of the following
findings would suggest perforation of the appendix? [B2 Q56]

a. Appendix diameter of 8mm
b. Appendix wall thickness of 4mm
c. Decreased resistance of arterial waveform
d. Loss of visualisation of hyperechoic submucosa
e. Increased echogenicity of surrounding fat

A

Loss of visualisation of hyperechoic submucosa
The use of ultrasound for the diagnosis of acute appendicitis is particularly useful in children
and women of child-bearing age. Findings indicating acute appendicitis include a tubular non-
compressible blind-ending structure with diameter >6 mm and wall thickness >2 mm, although
these signs do not necessarily indicate perforation. Features suggesting perforation include a
fluid collection adjacent to the appendix, gas bubbles near the appendix and loss of
visualisation of the submucosal layer.

How well did you know this?
1
Not at all
2
3
4
5
Perfectly
108
Q

A 23-year-old man presents with acute lower abdominal pain. An abdominal radiograph
demonstrates a rounded, laminated calcific density projected over the right lower quadrant.
What is the approximate likelihood of a diagnosis of acute appendicitis? [B4 Q31]
a. 10%
b. 30%
c. 50%
d. 70%
e. 90%

A

90%
A laminated calcified appendicolith is seen in only 7–15% of patients with acute appendicitis.
However, the presence of acute abdominal pain with an appendicolith on abdominal plain film
indicates a 90% probability of acute appendicitis, and also indicates a high probability of
gangrene/ perforation. Other plain film signs of acute appendicitis include caecal wall
thickening, small bowel obstruction and focal extraluminal gas collections

How well did you know this?
1
Not at all
2
3
4
5
Perfectly
109
Q

A 65-year-old man presents with a several-week history of lower abdominal pain and diarrhoea.
On examination he has tenderness and guarding in the left lower quadrant. On contrast-
enhanced CT, the inferior mesenteric vein is dilated, with a thin rim of enhancement around a
central area of low density. What is the most likely additional pathology demonstrated on the
CT? [B4 Q13]

a. sigmoid diverticulitis
b. appendicitis
c. Crohn’s disease
d. pancreatitis
e. caecal malignancy

A

Sigmoid diverticulitis
The inferior mesenteric vein provides venous drainage for the rectum, sigmoid and descending
colon, and is a potential route of spread of neoplastic and inflammatory conditions. Inferior
mesenteric vein thrombosis may occur secondary to an inflammatory process, most commonly
diverticulitis, or malignancy. Other potential causes include hypercoagulable states, surgery,
trauma and bowel obstruction. Appearances are of an enlarged vein with rim enhancement
surrounding central low-density thrombus. Superior mesenteric vein thrombosis is much more
common (95% of mesenteric venous thrombosis) and may follow an inflammatory or
neoplastic process affecting the small intestine, caecum, and ascending and transverse colon.

How well did you know this?
1
Not at all
2
3
4
5
Perfectly
110
Q

A 40-year-old man is admitted to the surgical ward with acute abdominal pain and subsequently
a CT abdomen and pelvis is requested. The findings include a 3cm oval mass with central fat
density adjacent to the sigmoid colon and with associated fat stranding. Which one of the
following is the most likely diagnosis? [B2 Q5]

a. Diverticulitis
b. Epiploic appendagitis
c. Mesenteric lymphadenitis
d. Meckel’s diverticulitis
e. Infected enteric duplication cyst

A

Epiploic appendagitis
Epiploic appendagitis is inflammation of one of the epiploic appendages of the colon, with the
sigmoid being the commonest site. It typically presents with acute abdominal pain and is an
important radiological diagnosis as it can often mimic appendicitis, and management is
conservative. The diagnosis is usually made on CT with the features described in the question.
Ultrasound is rarely used for diagnosis, and features include a non-compressible hyperechoic
mass with hypoechoic margins.

How well did you know this?
1
Not at all
2
3
4
5
Perfectly
111
Q

A usual fit and well 30-year-old female presents with left-sided abdominal pain. Normal uterus
and ovaries are identified. In the left iliac fossa, there is a 4-cm hyperechoic mass which is non-compressible between the colon and abdominal wall. No colonic wall changes are identified.
There is no free fluid. The lesion is surrounded by a hyperechoic border with no Doppler flow.
Which is the most likely diagnosis? [B3 Q14]

A. Epiploic appendigitis
B. Diverticulitis
C. Omental infarction
D. Omental metastases
E. Mesenteric panniculitis

A

Self-limiting condition with fat density mass 1-4cm between colon and abdominal wall. The
absence of blood flow on Doppler due to torsion helps distinguish from acute diverticulitis.
Omental infarcts are usually large, right-sided and cake-like in appearance.

How well did you know this?
1
Not at all
2
3
4
5
Perfectly
112
Q

A 34-year-old man presents with acute left lower quadrant pain following unaccustomed
exercise. CT of the abdomen demonstrates a 2.5 cm oval lesion with attenuation value of–60
HU abutting the sigmoid colon, with surrounding inflammatory changes. The sigmoid colon
itself appears normal. What is the most likely diagnosis? [B4 Q9]

a. omental infarction
b. diverticulitis
c. epiploic appendagitis
d. liposarcoma
e. appendicitis

A

Epiploic appendagitis
Acute epiploic appendagitis is thought to result from torsion of one of the fatty epiploic
appendages arising from the serosal surface of the colon. It usually occurs in young men,
presenting as acute lower quadrant pain, and is associated with obesity and unaccustomed
exercise. Typical CT findings are of an oval peri-colonic fat density lesion of ,5cm, with
surrounding inflammatory changes, most commonly in the sigmoid, descending or right hemi-
colon. Right-sided epiploic appendagitis may be mistaken clinically for appendicitis. Omental
infarction typically appears as a larger, heterogeneous lesion, usually affecting the caecum or
ascending colon. Acute diverticulitis usually occurs in older patients and is associated with
colonic diverticula and wall thickening. Liposarcoma is rare but is included in the differential
of a fat-containing intra-abdominal mass.

How well did you know this?
1
Not at all
2
3
4
5
Perfectly
113
Q

A 39-year-old male complains of severe, colicky left lower abdominal pain and rectal bleeding.
He has experienced intermittent abdominal pain for the last 3–4 months. There is no previous
history of medical problems. On examination he has left lower abdominal tenderness without
signs of peritonism. A CT examination is performed which reveals a focal intraluminal
abnormality, with the appearance of a mass within the sigmoid colon. There are concentric
rings of soft tissue and fatty attenuation giving a ‘target’ like appearance. Mesenteric vessels
are seen to course into the lesion. At the most distal point of the abnormality there is a more
discrete low attenuation mass measuring approximately 3 cm in size of fatty attenuation. The
large bowel distal to the sigmoid lesion is collapsed and proximal to the lesion there are
multiple loops of small bowel and a dilated colon. What is the most likely underlying pathology
for this condition? [B1 Q61]

A. Benign tumour.
B. Malignant tumour.
C. Inverted diverticulum.
D. Idiopathic.
E. Inflammatory bowel disease.

A

Benign tumour

The patient has presented with a colo-colic intussusception secondary to a lipoma of the colonic
wall. The intussusception has been complicated by large bowel obstruction.

Intussusception is caused by prolapse of a portion of the bowel into the lumen of the adjoining
bowel lumen segment. Intussusception is most commonly a disease of young children and most
commonly occurs in an ileocolic location. In children, over 90% of intussusceptions are
idiopathic and no lead point is identified. In adult patients approximately 80% of
intussusceptions are caused by lead point lesions.

The CT appearances described are typical of an intussusception. The low attenuation distal
lead point represents the lipoma. Ultrasound is often used to diagnose intussusception in
children and will show a mass with echogenic rings representing the fat in the invaginated
mesentery.

How well did you know this?
1
Not at all
2
3
4
5
Perfectly
114
Q

A 50-year-old woman undergoes CT colonography for a change in bowel habit, which
demonstrates a well-defined, broad-based, 3 cm submucosal mass of density–40 HU in the
ascending colon. It is noted to change shape between prone and supine images. What is the
most likely diagnosis? [B4 74]

a. adenomatous polyp
b. endometriosis
c. primary pneumatosis coli
d. lipoma
e. enteric duplication cyst

A

Lipoma
The colon is the commonest site for gastrointestinal lipomas. They typically appear as a broad-
based mass but may develop a short pedicle because of repeated peristaltic activity. They are
soft lesions, and a change in shape or size may be noted on compression. At CT colonography,
endoluminal features are non-specific, but the demonstration of fat attenuation on 2 D images
is diagnostic. Primary pneumatosis coli usually appears as a cluster of air-filled cysts in the left
colon. These may mimic polyps at endoscopy or on endoluminal CT colonography, but the
demonstration of air attenuation on 2D images is again diagnostic. Endometriosis and enteric
duplication cyst are causes of submucosal lesions but are of soft-tissue and fluid density
respectively. Adenomatous polyps are mucosal lesions of soft-tissue density.

115
Q

A67-year-old man presents with a sensation of incomplete evacuation and passage of thick
mucus per rectum. Serum electrolytes demonstrate hypokalaemia and hyponatraemia. He
undergoes barium enema, which demonstrates a broad-based, papillary, 2 cm lesion in the
rectum with poor mucosal coating of barium. What is the most likely diagnosis? [B4 Q82]

a. lipoma
b. tubular adenoma
c. villous adenoma
d. colorectal carcinoma
e. solitary rectal ulcer syndrome

A

Villous adenoma

Villous adenomas are a histological subtype of adenomatous polyps with predominantly villous
elements, representing 10% of adenomatous polyps. Typical appearances are of a broad-based
lesion often over 2cm in diameter, with frond-like surface projections. They have a higher
malignant potential than the other subtypes of adenomatous polyp (tubular and tubule-villous
adenomas), which increases further with the size of the adenoma. Lesions under 5cm have a
9% risk of malignant transformation to adenocarcinoma, whereas lesions over 10cm have a
100% risk of malignant transformation
. Villous adenomas are associated with excretion of large amounts of thick mucus,
which may result in diarrhoea and electrolyte depletion, as well as poor mucosal coating at
barium enema. Lipomas are typically seen as smooth, rounded, submucosal masses. Tubular
adenomas are usually ,10mm and, like lipomas, are not associated with electrolyte depletion.
Solitary rectal ulcer syndrome may appear as polypoid lesions in the rectum and be associated
with mucus secretion; however, it is normally associated with ulceration and is usually seen in
young women.

116
Q

A 73-year-old man presents with lower abdominal pain and a change in bowel habit. A contrast
enema demonstrates a stricture in the sigmoid colon. Which feature would favour a diagnosis
of colorectal carcinoma rather than diverticulitis? [B4 Q50]

a. long (>10 cm) segment of involvement
b. mucosal ulceration
c. presence of a colo-vesical fistula
d. multiple diverticula in the sigmoid colon
e. smoothly tapered stricture margins

A

Mucosal ulceration

The differentiation between complicated diverticular disease and a perforating colorectal
carcinoma may be difficult. Both may appear as a focal area of eccentric luminal narrowing on
contrast enema. A longer segment of involvement favours diverticulitis, as well as other
inflammatory causes of colitis. In addition, stricture margins in diverticulitis tend to be
smoothly tapered rather than the abrupt narrowing of carcinoma. Mucosal ulceration occurs in
most cases of colorectal carcinoma and is not a particular feature of diverticulitis. Fistula
formation may occur in both conditions, most commonly between the colon and the bladder,
though this is more commonly seen in diverticulitis. The presence of diverticula does not
exclude the possibility of colorectal carcinoma, and the two conditions may coexist, as both
are common in elderly people.

117
Q

What is the primary imaging investigation for staging of colon cancer diagnosed at
colonoscopy? [B4 Q12]

a. CT of the abdomen and pelvis
b. CT of the thorax, abdomen, and pelvis
c. abdominal ultrasound scan
d. double-contrast barium enema
e. 18FDG PET/CT

A

CT of the thorax, abdomen, and pelvis

Patients with colon cancer diagnosed endoscopically or suspected following barium enema
should be imaged for staging purposes. Objectives of staging include determination of size and
local extent of tumour, assessment of the extension of tumour into adjacent structures, and
detection of local and distant nodal involvement and presence of metastatic disease. Abdominal
ultrasound scan alone is not considered sufficient, and CTof the chest, abdomen and pelvis with
oral and intravenous iodinated contrast medium is the primary imaging investigation. The liver
is the commonest site of distant metastases, but pulmonary metastases occur in 5–50% of
patients. 18 FDG PET/CT is not used in initial staging but is particularly useful for detecting
recurrent disease

118
Q

A 65-year-old man, being investigated for iron deficiency anaemia, altered bowel habit, and
weight loss, is diagnosed with colon cancer. A staging CT demonstrates irregularity to the outer
bowel wall at the site of tumour, a cluster of three lymph nodes with the largest individual node measuring 0.9cm, and no evidence of distant metastases. What is the most likely TNM stage?
[B1 Q 52]

A. T2, N0, M0.
B. T2, N1, M0.
C. T3, N0, M0.
D. T3, N1, M0.
E. T4, N1, M0.

A

T3, N1, M0.

T3 implies invasion beyond the muscularis propria into the pericolic fat. CT is quoted as 80%
accurate in detecting extramural spread and nodal stage. T1 and T2 tumours are well-defined
with no extension beyond the bowel contour. CT features of a T4 tumour include nodular
penetration of the tumour through the peritonealized areas of the muscle coat or an advancing
edge of the tumour penetrating adjacent organs. Nodal positivity is based on any node >1 cm
or a group of three or more nodes.

119
Q

A 52-year-old male with a metal heart valve has a transrectal ultrasound performed to stage
rectal carcinoma as MRI is contraindicated. A 3cm hypoechoic mass is identified from three
to seven o’clock in the lower rectum. It extends through an inner hypoechoic layer and into the
outer hypoechoic layer, but the outermost hyperechoic layer is intact and unaffected. What is
the correct T staging (TNM system) based on these observations? [B2 Q57]

a. T0
b. T1
c. T2
d. T3
e. T4

A

T2

The layers of the rectum are well demonstrated at transrectal ultrasound. The innermost
hyperechoic layer represents the balloon-mucosa interface, the middle hyperechoic layer
represents the submucosa, and the outermost hyperechoic layer represents the serosa. The
tumour described in the question extends through the submucosa into the muscularis propria
(outer hypoechoic layer) but does not involve the serosa. T1 disease is limited to the submucosa,
T2 is limited to the muscularis propria, T3 extends through the serosa and T4 represents
invasion of adjacent organs. The correct staging for the tumour described in the question is
therefore T2.

120
Q

A 70-year-old man with sudden onset left-sided abdominal pain underwent evaluation with CT.
Fat stranding and colonic wall thickening is noted in the distal descending/ proximal sigmoid
colon. Diverticula are noted. Which feature is most likely to represent a colonic tumour rather
than diverticulitis? [B3 Q13]

A. Colonic wall thickening
B. Peri-colonic fat stranding
C. Fluid in the sigmoid mesentery
D. Engorgement of the mesenteric vessels E. Increased permeability on CT perfusion

A

Increased blood volume, increased blood flow, decreased mean transit time and increased
permeability are parameters on CT perfusion which have been shown to favour a diagnosis of
cancer rather than diverticulitis. C and D have high positive predictive values in diagnosing
diverticulitis and A and B occur in both.

121
Q

In the MR staging of rectal tumours, which of the following is the single best answer? [B3 Q49]

A. A distance of less than 3mm from the meso-rectal fascia is most likely to reflect
circumferential resection margin (CRM)
B. There is a prognostic difference between different types of T3 tumours depending on tumour
size
C. Untreated mucinous and non-mucinous tumours are difficult to distinguish on T2 sequences
D. Untreated tumour signal is similar to muscle on T2
E. Non-mucinous tumour signal intensity is similar to fat

A

There is a prognostic difference between different types of T3 tumours depending on tumour size

The MERCURY Trial showed patients with T3 disease with more than 5mm of extramural
spread have a markedly worse prognosis than those with less than 5mm spread beyond the
outer muscle layer. A distance less than 2mm from mesorectal fascia indicated CRM
involvement. Tumour signal is usually in between SI of muscle and fat. Mucinous tumour is
higher signal than non-mucinous which is higher than fat on T2.

122
Q

A patient with a history of inflammatory bowel disease, treated with colonic resection and J
pouch anastomosis, presents to the surgical team in your hospital. The operation was 3 months
ago, and the initial post-operative period was unremarkable. His post-operative pouchogram
was reported as normal and he underwent a reversal of his defunctioning ileostomy 6 weeks
ago. He now presents with central and lower abdominal pain associated with nausea and
vomiting, but no diarrhoea. The surgeons request a pouchogram, which shows a small blind
ending lumen at the superior aspect of the pouch. A follow-up CT scan shows dilated small
bowel with a transition point in the ileum, beyond which the bowel is non-distended. The J
pouch has mild inflammatory change in the surrounding fat. There is also a small amount of
free fluid in the pelvis. The wall of the pouch is not thickened. What is the most likely diagnosis?

A. Small bowel obstruction, as can occur in up to 30% of these patients.
B. Pouchitis.
C. Pouch fistula.
D. Recurrence of Crohn’s in the pouch and affected segment of bowel.
E. Pouch leak.

A

Small bowel obstruction, as can occur in up to 30% of these patients

Total colonic resection with J pouch anastomosis is carried out in patients with ulcerative colitis
or familial adenomatous polyposis (FAP) to resect the entire colon but retain anal defaecation.
One of the main contraindications to total colectomy and J pouch formation is Crohn’s disease
and such a procedure is uncommonly carried out in this situation due to the high recurrence
rate. Pouchitis and small bowel obstruction both occur in up to 30% of patients who undergo
this procedure. The radiological features of pouchitis are non-specific but include wall
thickening and increased enhancement of the bowel wall, with peri-pouch fat-stranding. Fat-
stranding on its own is common because of both the surgery and possible inflammatory change
due to previous proctitis. Small bowel obstruction most commonly occurs in the region of the
ileostomy. Free fluid is often seen in small bowel obstruction. The absence of fluid cavities or
air pockets is against a leak from the pouch anastomosis and the clinical features are not
consistent with fistula. The blind ending lumen described is the normal appearance seen on a
pouchogram.

123
Q

Which of the following frequencies would most help in the differentiation between recurrent
tumour and fibrosis in a 62-year-old with prior chemoradiotherapy for low rectal cancer? [B3
Q7]

A. Axial and coronal balanced gradient echo
B. Saggital T2 fat suppressed fat-spin echo (FSE)
C. Axial TIW fat suppressed FSE before and after gadolinium
D. Saggital TIW fat suppressed FSE post gadolinium
E. Coronal TIW fat suppressed FSE post gadolinium

A

Axial T1W fat suppressed FSE before and after gadolinium
May help differentiate tumour from fibrosis and improves destruction of tumour spread to
adjacent structures/vessels. Axial and coronal balanced gradient echo are rapidly acquired
breath hold sequences providing anatomical overviews. Granulation tissue, haematoma and radiation-induced inflammatory change can all display SI like tumour. Peripheral enhancement
with central necrosis is commonly seen in recurrent disease.

124
Q

On review of a pelvic MRI of a patient who has previously received radiotherapy, high signal
intensity change is noted on the lumbar spine and sacrum. On the T2 images, which of the
following would also be expected to be demonstrated post-radiotherapy? [B3 Q39]

A. Bladder wall thickened
B. Ovaries increased SI
C. Rectum decreased SI
D. Granulation tissue with decreased SI
E. Fibrous tissue with decreased SI

A

Bladder wall thickened
The urinary bladder and rectum are thickened and oedematous and can show increased T2 and
SI. Post-radiotherapy, the ovaries tend to atrophy and therefore decrease in SI but bone marrow,
fibrous, glandular tissue can also show increased SI.

125
Q

Which anatomical structure separates the right and left subphrenic spaces? [B4 Q40]

a. gastro-hepatic ligament
b. foramen of Winslow
c. falciform ligament
d. Morison’s pouch
e. lesser omentum

A

**Falciform ligament **

The right subphrenic space is in communication around the liver with the anterior subhepatic
and posterior subhepatic (Morison’s pouch) spaces. The right subphrenic and subhepatic spaces
communicate freely with the right paracolic gutter and via this with the pelvic peritoneal cavity.
The left subphrenic space communicates with the left subhepatic space but is separated from
the right subphrenic space by the falciform ligament and from the left paracolic gutter by the
phrenico-colic ligament. The falciform ligament is a sickle-shaped fold of peritoneum that
attaches the ventral surface of the liver to the anterior abdominal wall. It extends in a
parasagittal plane from the umbilicus to the diaphragm and carries the ligamentum teres in its
free infero-posterior margin. The lesser sac is an isolated peritoneal compartment between the
stomach and pancreas, which communicates with the rest of the peritoneal cavity (greater sac)
only through the foramen of Winslow (epiploic foramen). The lesser omentum is composed of
the gastro-hepatic and hepatico-duodenal ligaments and suspends the stomach and duodenal
bulb from the inferior liver surface.

126
Q

A lesion is noted in the liver on CT and ultrasound. It is inferior, anterior, and to the left of the
right hepatic vein, but to the right of the middle hepatic vein. It is inferior of the confluence of
the right and left portal veins. According to the Couinaud system, what segment of the liver is
the lesion in?

A. Segment 4b.
B. Segment 5.
C. Segment 6.
D. Segment 7.
E. Segment 8.

A

Segment 5

127
Q

Which venous structure divides the liver into the right and left lobes? [B4 Q11]

a. right hepatic vein
b. middle hepatic vein
c. left hepatic vein
d. left portal vein
e. right portal vein

A

Middle hepatic vein

The functional segmental anatomy of the liver is based on the distribution of the three major
hepatic veins. The middle hepatic vein divides the liver into left and right lobes. The left hepatic
vein divides the left lobe into medial and lateral segments. The right hepatic vein divides the
right lobe into anterior and posterior segments. In addition, the four sections are further
subdivided in a transverse plane by an imaginary line drawn between the right and left portal
veins. Segments run in a clockwise fashion, with segments III, IV(b), V and VI lying below
the portal veins and segments VII, VIII, IV(a) and II lying above. The caudate lobe (segment
I) lies posterior and to the right of the inferior vena cava.

128
Q

A trans-jugular intrahepatic portosystemic shunt lies between the portal vein and which vessel?
[B4 Q59]

a. hepatic vein
b. inferior vena cava
c. aorta
d. common hepatic artery
e. left gastric vein

A

Hepatic Vein

A trans-jugular intrahepatic portosystemic shunt (TIPSS) is an endovascular procedure
performed to create a portosystemic shunt between the portal venous and hepatic venous
systems, for decompression of portal hypertension, particularly in patients with variceal
bleeding uncontrollable by endoscopic management. The shunt is normally formed between
the right hepatic vein and right portal vein. The right internal jugular vein is accessed, and via
this the right hepatic vein. Curved needle passes are made in an anterior direction to access the
right portal vein. The portosystemic pressure gradient is measured, and portal venography is
performed to enable planning of stent placement. A stent is deployed following balloon
dilatation of the stent tract. The goal is to decrease the portosystemic gradient to below
12mmHg and to see no significant filling of varices. Portosystemic shunts require close follow-
up, as there is a high incidence of shunt stenosis and occlusion.

129
Q

The gastroduodenal artery normally has its origin from which vessel? [B4 Q43]

a. superior mesenteric artery
b. common hepatic artery
c. left gastric artery
d. coeliac axis
e. aorta

A

Common hepatic artery

In 75% of cases, the gastroduodenal artery arises from the common hepatic artery before its
division into right and left branches. Less common origins include the left hepatic artery (4–
11%), the right hepatic artery (7%) and the superior mesenteric artery via a replaced hepatic
trunk (4–11%). The artery descends behind the first part of the duodenum, lying anterior to the
pancreas and to the left of the common bile duct. At this site, erosion of the posterior duodenal
wall by an ulcer may produce life-threatening haemorrhage if the gastroduodenal artery is involved. Its main branches are the posterior and anterior superior pancreaticoduodenal arteries,
and the right gastroepiploic artery.

130
Q

From which vessel does most of the arterial supply to the pancreas derive? [B4 Q100]

a. splenic artery
b. left gastric artery
c. superior pancreaticoduodenal artery
d. right hepatic artery
e. superior mesenteric artery

A

Splenic artery

The main arterial supply to the pancreas is from the splenic artery, which provides numerous
small branches into the pancreatic substance as it runs along the superior pancreatic border, as
well as several larger arteries including the dorsal pancreatic artery from its proximal end
(which may alternatively arise from the coeliac artery) and the arteria pancreatica magna
halfway along its length. The pancreatic head has a dual blood supply, from the superior
pancreaticoduodenal artery (derived from the gastroduodenal artery) and the inferior
pancreaticoduodenal artery (derived from the superior mesenteric artery). The transverse
pancreatic artery also runs along the length of the pancreas beside the main duct, and there are
multiple anastomoses between the various vessels, allowing multidirectional flow.

131
Q

A 45-year-old female is suspected to have focal areas of fat infiltration on ultrasound of the
liver. An MRI of the liver is requested for further assessment. What sequences are most useful
in confirming the diagnosis of focal fat infiltration? [B1 Q25]

A. T1WI pre and post gadolinium.
B. T1WI and T2WI.
C. T1WI and fat saturated T2WI.
D. Dual GE T1WI in phase and out of phase.
E. MR spectroscopy.

A

Dual GE T1WI in phase and out of phase.
Three basic MRI techniques are available for fat detection, which work based on the difference
in precession frequency between water and fat protons. These are chemical shift imaging,
frequency-selective imaging, and MR spectroscopy. Dual GE T1WI is the most useful
sequence in clinical practice. It is based on the phase interference effect or chemical shift
imaging of the second kind. When the fat and water protons are in phase there is constructive
interference and when they are out of phase there is destructive interference. By comparing the
signal intensities on the in-phase and out-of-phase images, fat detection is possible.

Chemical shift imaging of the first kind, or chemical shift spatial misregistration, occurs at fat–
water interfaces in the frequency-encoding direction, manifesting as alternating bands of high
and low signal. It is present in all standard non-fat-saturated sequences, but it can be subtle and
may be missed or mistaken for image noise. Frequency-selective imaging with selective
excitation or saturation depends on the homogeneity of the magnetic field and the size of the
lesion.
MR spectroscopy is too time-consuming for routine clinical use.

132
Q

A 50-year-old male undergoes MRI of the liver for further characterization of a suspected
haemangioma on ultrasound. In addition to the haemangioma, a peripheral wedge-shaped area
of enhancement is seen in the arterial phase, but no abnormality is seen in the corresponding
area in the non-contrast or portal venous phases. What is the diagnosis? [B1 Q37]

A. Hepatocellular carcinoma.
B. Hepatic infarct.
C. Transient hepatic intensity difference (THID).
D. Hyper-vascular metastasis.
E. Haemangioma.

A

Transient hepatic intensity difference (THID)

THID on MRI or transient hepatic attenuation difference (THAD) on CT is a pseudo lesion
caused by focal alteration in the haemodynamic of the liver due to either non-tumorous arterio-
portal shunt or obstruction of distal portal venous flow. THID or THAD is seen as a focal area
of enhancement in the arterial phase only, with no abnormality seen in the portal venous phase.
Features suggestive of THID or THAD include peripheral location, wedge shape, straight
margins, and normal vessels coursing through the area.

133
Q

A patient presents to the surgeons with a known history of gallstones, for which she underwent
an ERCP 2 years earlier. She has had recurring pain and mildly elevated liver function tests.
She underwent an MRCP/MRI liver prior to consideration for surgery. This showed several 8-
mm filling defects in the CBD. Which of the following MRI sequences is likely to be the most
helpful in trying to determine if these filling defects are due to pneumobilia, as opposed to
retained calculi? [B1 Q64]

A. Axial T2 steady-state GE.
B. Coronal thick slab MRCP.
C. Three-dimensional volume coronal MRCP.
D. Two-dimensional coronal oblique thin (4mm) MRCP.
E. Axial T1 in phase and out of phase GE.

A

Axial T2 steady-state GE
Axial imaging is generally going to be better than coronal imaging when trying to distinguish
between pneumobilia and calculi within the CBD. The biliary air causes an air/fluid level of
air lying on top of fluid in a non-dependent position in the CBD and this is more easily
appreciated on an axial image. Calculi tend to lie dependently within the CBD. As fluid is
hyperintense on T2WI and hypointense on T1WI, and both air and calculi are hypointense on
both these sequences, then both will stand out as being more conspicuous on T2WI.

134
Q

You have been asked to give a presentation on MRI of the liver to your radiological colleagues.
One of the audiences asks if any of the contrast agents used in MRI of the liver works best with
any sequence other than a T1WI sequence. What do you respond? [B1 Q65]

A. Yes, gadopentate dimeglumine.
B. Yes, mangafodipir trisodium. C. Yes, gadobenate dimeglumine (hepatocyte specific).
D. Yes, SPIO.
E. No, all liver contrast agents work best with T1WI

A

Yes, SPIO

Gadolinium chelates are extracellular agents. Gadolinium shortens the T1 relaxation time of
adjacent water protons, resulting in signal enhancement on T1WI. It can be used for lesion
detection, characterization, and liver vasculature assessment. Gadopentate dimeglumine
(Magnevist®) is a commonly used extracellular agent.

SPIO particles are reticulo-endothelial agents that are phagocytosed by Kupffer cells. SPIO
causes local magnetic field inhomogeneity and T2 and T2* shortening, resulting in signal loss
on T2WI and T2*WI. Most liver tumours, including HCC, are deficient in Kupffer cells (cf.
focal nodular hyperplasia) therefore after administration of SPIO the tumour appears
hyperintense relative to the background liver.

Mangafodipir trisodium (Teslascan®) is a manganese-based hepatocyte specific agent that
increases the signal intensity of the liver, bile ducts, and some hepatocyte-containing lesions
(e.g. FNH) at T1WI. Similarly, gadobenate dimeglumine (Multihance®) is a gadolinium-based
hepatocyte specific agent, which also works best with T1WI due to the T1 shortening effects
of gadolinium.

135
Q

A 42-year-old woman is being investigated following a new diagnosis of hepatitis C. Which
of the following would have the highest accuracy in the characterization of a focal liver lesion
in this patient? [B3 Q43]

A. Ultrasound
B. Doppler ultrasound
C. Microbubble contrast enhanced ultrasound (CEUS)
D. Combined US and AFP
E. Combined US and Doppler US

A

Microbubble contrast enhanced ultrasound (CEUS)
CEUS can be used to characterise focal liver lesions with 96.6% accuracy. Small HCC (1-2cm)
can be diagnosed due to arterial phase hypervascularity and washout characteristics.

136
Q

With regards to MRI in patients being investigated for Crohn’s disease, which of the following
is the least desirable characteristic of ultrafast sequences based on steady state precession? [B3
Q44]

A. A high sensitivity to motion artefact
B. The time taken to acquire an image
C. The degree of contrast between bowel wall and lumen
D. Degree of soft-tissue differentiation when compared to other sequences
E. A black boundary effect

A

A black boundary effect
These sequences have become the main MR technique for imaging small and large bowel in
Crohns disease in recent years. Uniform luminal opacification is obtained with high contrast
between bowel wall, lumen, and mesentery. The sequences are relatively insensitive to motion
artefact. Black boundary artefact can obscure small lesions on the bowel wall, although fat-
suppression can reduce the artefact.

137
Q

A 68-year-old man presents with acute abdominal pain. As well as other pathology, CT of the
abdomen reveals multiple linear branching structures with an attenuation value of–1000 HU in
the liver extending to the periphery. What are these appearances most likely to represent? [B4
Q30]

a. gas in the portal venous system
b. gas in the biliary tree
c. portal venous thrombosis
d. intrahepatic biliary dilatation
e. fatty infiltration of the liver

A

Gas in the portal venous system

Portal venous gas is identified on CT as linear branching structures of air density extending to
the periphery of the liver, presumably due to the direction of portal venous flow. The
commonest cause in adults is mesenteric infarction when it is a poor prognostic sign. In infants
the commonest cause is necrotizing enterocolitis when it does not necessarily imply a poor
outcome. In contrast, gas within the biliary tree is central and does not extend into the
peripheral 2 cm of the liver. In portal venous thrombosis, a focal hypodensity is seen within
the portal vein on contrast-enhanced CT. Intrahepatic biliary dilatation appears as dilated
branching ductal structures of fluid density. In fatty infiltration of the liver, vessels may appear
hyperattenuating on unenhanced CT in contrast to the hypodense liver.

138
Q

In normal anatomy, the portal vein is usually formed by the confluence of which vessels? [B4
Q90]

a. left and right portal veins
b. inferior and superior mesenteric veins
c. superior mesenteric and splenic veins
d. inferior mesenteric and splenic veins
e. left, middle and right hepatic veins

A

Superior mesenteric and splenic veins
The splenic and superior mesenteric veins join to form the main portal vein slightly to the right
of the midline behind the neck of the pancreas at L1–2 level. The extrahepatic portal vein is
about 8cm long, and divides into the right and left portal veins at the porta hepatis. The inferior
mesenteric vein most commonly drains into the splenic vein, but may drain into the
splenic/superior mesenteric vein confluence in 30% of cases or the superior mesenteric vein in
30%.

139
Q

What is the most common side effect associated with administration of superparamagnetic iron
oxide particles as a contrast agent during MRI? [B4 Q96]
a. urticarial skin rash
b. back pain
c. nephrotoxicity
d. nausea
e. headache

A

Back pain

The most common complication of administration of superparamagnetic iron oxide particles is
acute severe back pain, which is seen in approximately 4% of patients. This is thought to be a
side effect of particulate agents in general and lasts for the duration of the infusion and slightly
beyond. The risk is higher in patients with liver dysfunction, and when the infusion is
administered more rapidly than over the recommended 30 minutes. Slowing of the infusion
rate or termination of the infusion with recommencement after resolution of the back pain is
usually sufficient to alleviate the symptoms

140
Q

A 53-year-old male is investigated for recurrent episodes of biliary colic. Blood tests reveal
eosinophilia and normal liver function tests. Abdominal ultrasound demonstrates a 7cm cystic
structure with a thin hyperechoic wall and several smaller satellite cysts up to 2cm adjacent to
the lesion. Which one of the following diagnoses is most likely? [B2 Q28]

a. Hydatid cyst
b. Pyogenic abscess
c. Amoebic abscess
d. Schistosomiasis
e. Hepatocellular carcinoma

A

Hydatid cyst

The most likely diagnosis is hydatid cyst disease. This condition is caused by infection of the
liver with the parasite Echinococcus granulosus. Blood eosinophilia is present in up to 50% of
patients. It is more common in the right lobe of the liver and is multiple in 20% of cases.
Daughter cysts are typical. Percutaneous aspiration of the cyst is positive for hydatid disease
in 70%.

141
Q

USG of a 44-year-old man shows a 7cm cystic lesion in segment 8 of the liver. There is some
dependent debris and an apparent cyst within cyst appearance. Which is the most likely
diagnosis? [B3 Q17]

A. Haemangioma
B. Abscess
C. Hydatid disease
D. Hepatic cyst
E. Cystic metastasis

A

The cyst within cyst appearance represents daughter cyst. Other features include wall
calcification.

142
Q

A 34-year-old male visitor to the United Kingdom under the care of the medical team has
abnormal liver function tests. US reveals periportal hyper-echogenicity and gallbladder wall
thickening but no biliary dilatation. The patient has a liver MRI study. T2 images show high
signal intensity bands along the portal tracts and note is made of splenomegaly with siderotic
nodules. Gadolinium enhanced T1 images confirm enhanced periportal bands. Which of the
following is the most likely causative organism? [B3 Q50]

A. Strongyloides
B. Schistosomiasis
C. Fasciola hepatica flatworm
D. Ascariasis
E. Echinococcus

A

Schistosomiasis

Schistosomiasis is the most likely cause. Periportal fibrosis with signs of portal hypertension,
splenomegaly and siderotic nodules are typical features of hepatosplenic schistosomiasis.

143
Q

A previously well, 28-year-old man recently returned from the Far East becomes acutely
unwell with fever and right upper quadrant pain. Ultrasound scan demonstrates a well-defined,
rounded, 7 cm hypoechoic lesion in the right lobe of the liver contiguous with the liver capsule,
with fine homogeneous, low-level internal echoes and acoustic enhancement. What is the most
likely diagnosis? [B4 Q16]

a. pyogenic abscess
b. amoebic abscess
c. fungal abscess
d. hydatid disease
e. incidental simple hepatic cyst

A

Amoebic abscess
Pyogenic abscesses are the commonest type of liver abscess in developed countries and are
most frequently due to ascending cholangitis from benign or malignant obstructive biliary
disease. They are often poorly defined with irregular walls on ultrasound scan and may contain
debris or demonstrate intense hyper-echogenicity when containing gas. Amoebic abscesses
tend to occur in younger, more acutely unwell patients from high prevalence areas or with a
history of recent travel. They are treated medically whereas pyogenic abscesses usually require
percutaneous or surgical drainage. Fungal abscesses are usually multiple and occur in
immunosuppressed individuals. Hydatid disease tends to be asymptomatic or present with
biliary colic. Characteristic ultrasound scan features include daughter cysts and detachment of
the endocyst, giving rise to ‘floating membranes’ within the cyst cavity.

144
Q

A 43-year-old woman is incidentally found to have a well-defined, rounded, low-density, 2 cm
lesion in the liver on unenhanced CT. Contrast-enhanced CT demonstrates peripheral nodular
arterial enhancement with complete fill-in on delayed images. What is the most likely diagnosis?
[B4 Q25]

a. hepatic haemangioma
b. hepatocellular carcinoma
c. simple hepatic cyst
d. focal fatty infiltration
e. focal nodular hyperplasia

A

Hepatic haemangioma

Haemangiomas are the most common benign liver tumour. They are often asymptomatic but
may present with hepatomegaly or rarely spontaneous haemorrhage. Typical CT features of a
hepatic haemangioma are of a well-defined hypodense mass on unenhanced CT, with early
peripheral enhancement after intravenous contrast followed by complete fill-in on delayed
images. Hepatocellular carcinoma is seen as a hypodense mass and usually demonstrates
contrast enhancement during the arterial phase, but enhancement decreases on delayed images.
Focal fatty infiltration usually has a geographic distribution, and, like simple hepatic cysts,
does not demonstrate contrast enhancement. Focal nodular hyperplasia is usually isodense on
unenhanced CT and, although it tends to show intense transient arterial phase enhancement, is
often isodense during the portal phase. A central scar, if present, may demonstrate enhancement
on delayed images

145
Q

A 56-year-old male patient is referred for an ultrasound of abdomen prior to undergoing an
anterior resection for a proximal rectal carcinoma. The ultrasound reveals a 2cm lesion in the
right lobe of the liver, which is hyperechoic centrally with a hypoechoic rim. Which one of the
following cannot be considered in the differential for this lesion? [B1 Q12]

A. Metastases.
B. Haemangioma.
C. Sarcoid.
D. Candidiasis.
E. Lymphoma.

A

Haemangioma

The ultrasound findings describe a target lesion or bull’s eye lesion. Cavernous haemangiomas
can have unusual appearances, but a small lesion such as described will normally have a
uniform hyperechoic appearance. Of all the other lesions described, metastasis would be top of
the differential, although this appearance is not the most classical for a colonic metastasis.

Target lesions in liver ultrasound [STATdx]

Target sign: Echogenic centre surrounded by hypoechoic rim (bull’s-eye lesions)
Malignancy far outnumbers other causes Reverse target: Hypoechoic core with hyperechoic rim

Helpful Clues for Common Diagnoses

  1. Hepatic Metastases
    * Solid central tumour with hypoechoic halo
    * Halo most likely related to compressed hepatic tissue along with zone of cancer cell
    proliferation
    * Alternating layers of hyper- and hypoechoic tissue
    * Usually from aggressive primary tumours – Classic example: Bronchogenic carcinoma
  2. Hepatocellular Carcinoma
    * Background of cirrhosis, portal hypertension, ascites
    * Rare for cirrhotic livers to develop metastases from non-hepatic primary
    * Any mass in cirrhotic liver is more likely hepatocellular carcinoma than metastasis
  3. Hepatic Lymphoma
    * Vast majorities are uniformly hypoechoic
    * Splenomegaly or splenic lesions, lymphadenopathy, thickened bowel wall provide
    clues toward diagnosis
  4. Hepatic Adenoma
    * Usually iso-echoic or slightly hypo-echoic
    * Complications such as haemorrhage, central necrosis make centre echogenic
    * Occasional hypoechoic rim forms target-like appearance
  5. Fungal Hepatic Abscess
    * Often multiple lesions
    * Typically in immunocompromised patient
  6. Amebic Hepatic Abscess
    * Iso- to mildly hyperechoic center with hypoechoic halo
    * Abuts liver capsule
  7. Pyogenic Hepatic Abscess
    * Central hyperechoic inflammatory nodule surrounded by hypoechoic halo of fibrosis
    * Cluster sign: Cluster of small pyogenic abscesses that coalesce into single large cavity
    * Lobulated or irregular contour

Helpful Clues for Less Common Diagnoses

  1. Hepatic Atypical Haemangioma
    * Hypoechoic centre with thick or thin hyperechoic rim - “Typical atypical” appearance
    * Hypo echogenicity seem to be related to predominant fibrous stroma
    * Colour Doppler: Typically, undetectable colour flow in lesion – flow is too slow.
  2. Hepatic Hematoma
    * May have laceration tract leading to hepatic surface
    * Multiple organs involved if traumatic cause
146
Q

A 40-year-old female undergoes MRI of the liver, which demonstrates a 5-cm lesion that is
isointense to liver on T1WI and slightly hyperintense on T2WI. It has a central scar that is
hypointense on T1WI and hyperintense on T2WI. On contrast-enhanced dynamic MRI, the
lesion is hyperintense in the arterial phase, and isointense to liver in the portal venous phase
with delayed filling in of the central scar. What is the diagnosis? [B1 Q23]

A. Hepatic adenoma.
B. Fibrolamellar hepatoma.
C. Hypervascular metastasis.
D. Focal nodular hyperplasia (FNH).
E. Giant haemangioma.

A

Focal nodular hyperplasia (FNH)

This is the second most common benign liver tumour. It is thought to represent a hyperplastic
response of hepatocytes to an underlying vascular malformation. It is most common in young
adult females and is usually an asymptomatic solitary lesion. On histology, FNH consists of
hyperplastic hepatocytes and small bile ductules around a central scar. The bile ductules of
FNH do not communicate with the adjacent biliary tree.

At ultrasound, FNH is isoechoic or hypoechoic. Colour Doppler may show prominent central
vascularity.

At CT, FNH is typically slightly hyperattenuating or iso-attenuating to surrounding liver on
pre-contrast images. On post contrast images, FNH is hyperattenuating in the arterial phase
and iso-attenuating in the portal venous phase with hypoattenuating central scar. The scar
shows delayed enhancement.

At MRI, FNH is iso- to hypointense on T1WI and slightly hyper- to isointense on T2WI. The
central scar is hypointense on T1WI and hyperintense on T2WI. The enhancement pattern is
like that on CT.

If the appearances are atypical, MRI with hepatocyte-specific contrast agent (gadobenate
dimeglumine) may be useful in confirming the hepatocellular origin of the mass. With
gadobenate dimeglumine, FNH is iso- to hyperintense on the 1–3 hour delayed images in over
96% of cases.

Different MR Appearances of other lesions

Hepatic Adenoma [STATdx]

  • Far less common than FNH
  • Women on OC pills, pregnancy, Men on anabolic steroid etc.
  • Key imaging features for all modalities – Hypervascularity, Heterogeneity, Fat content,
    Haemorrhage, Encapsulation. [FNH is homogenous]
  • Ultrasound
    o Complex, hyper-/hypoechoic, heterogeneous mass with anechoic areas (fat,
    haemorrhage, necrosis, and calcification)
    o Capsule may be seen o Hyper-vascular (large peripheral arteries and veins)
    o *Intra-tumoral vein [Absent in FNH]
  • Angiogram
    o Centripetal flow
    o Peripheral enlarged hepatic artery as feeders
    o Hypo/avascular region (haemorrhage and necrosis)
  • CT
    o Encapsulation,
    o Haemorrhage (best seen in pre-contrast)
    o Intra-tumoral fat
    o Hypervascularity
    o Calcification
  • MR
    o Heterogenous T1 and T2 signals depending on content
    o Rim (pseudo-capsule) is both T1 and T2 dark
    o Gd-enhanced
    ▪ Arterial phase – Heterogeneously hyper-vascular in arterial phase
    ▪ Delayed phase – Pseudo-capsule is hyperintense to liver and adenoma
    o Primovist-enhanced
    ▪ No substantial uptake or retention in the delayed phase.
147
Q

A 55-year-old female with cirrhosis undergoes MRI of the liver, which demonstrates multiple
small nodules that are hypointense on T2WI and enhance following administration of
gadolinium in the arterial and portal venous phase. The nodules demonstrate uptake of
hepatocellular agent and super paramagnetic iron oxide (SPIO) particles. What is your
diagnosis? [B1 Q29]

A. Multifocal hepatocellular carcinoma (HCC).
B. Siderotic nodules.
C. Dysplastic nodules.
D. Regenerative nodules.
E. Multiple arterio-venous shunts.

A

Regenerative nodules
Regenerative nodules are formed in response to necrosis and altered circulation. They remain
enhanced in the portal venous phase as opposed to HCC, which typically demonstrates contrast
washout in the portal venous phase. Regenerative nodules have normal hepatocellular function
and Kupffer cell density and therefore demonstrate uptake of both hepatocellular agents and
SPIO particles. As dedifferentiation proceeds, the hepatocellular function and Kupffer cell
density reduce.

148
Q

An asymptomatic 46-year-old woman has an MR liver following an incidental finding of a
focal mass in the right lobe of the liver on ultrasound. The MR shows an 8cm isolated lesion.
It is high signal on T1-weighted sequences and isointense on T2-weighted sequences relative
to the normal liver parenchyma. The lesion is most likely to be which one of the following?
[B2 Q16]

a. Hepatocellular carcinoma
b. Liver metastasis
c. Haemangioma
d. Fibronodular hyperplasia
e. Adenoma

A

Adenoma

The lesion is most likely to be a hepatic adenoma. None of the other diagnoses typically share
these imaging characteristics. Adenomas are benign growths of hepatocytes and are most seen
in young women, particularly associated with oral contraceptive use. Eighty per cent are
solitary and found in the right lobe of the liver. The high signal on T1-weighted sequences is
due to the presence of fat and/or haemorrhage and can distinguish between this and many other
lesions in the liver which tend to be of low T1 signal on MR (e.g. metastases, HCC,
haemangiomas and FNH). Occasionally, imaging features can overlap with FNH and the two
lesions can be difficult to distinguish. However, the majority of FNH lesions are less than 5 cm
in size, whereas adenomas tend to be larger.

149
Q

A 41-year-old female has an MRI liver following a solitary 3cm lesion in the right lobe of the
liver. The lesion is isointense on T1-weighted and slightly hyperintense to liver parenchyma
on T2-weighted imaging. There is immediate intense homogenous enhancement with
gadolinium in the arterial phase, which becomes isointense on the venous phase. A central scar
is hypointense on T1 and hyperintense on T2-weighted sequences. Which one of the following
is the most likely diagnosis? [B2 Q48]

a. Adenoma
b. Cavernous haemangioma
c. Fibrolamellar carcinoma
d. Regenerative nodules
e. Focal nodular hyperplasia

A

Focal nodular hyperplasia
These imaging features are typical of focal nodular hyperplasia. This is the second most
common benign liver tumour and typically occurs in women more often than in men.
Adenomas are usually larger, enhance less brightly and do not typically have a central fibrous
scar. Cavernous haemangiomas are usually high signal on T2-weighted images, and of blood
pool intensity on contrast-enhanced T1-weighted images. Fibrolamellar carcinoma also has a
central scar, but this is typically of low signal intensity on T2-weighted imaging. Regenerative
nodules show high signal intensity on unenhanced T1-weighted imaging and do not have a scar.
(Ref: Marin D et al. Focal nodular hyperplasia: typical and atypical MRI findings with
emphasis on the use of contrast media.

150
Q

A 47-year-old female with a history of surgery for breast carcinoma is referred for ultrasound
after liver function tests show a mildly elevated alkaline phosphatase. The bile ducts are normal
but a 3cm hyperechoic liver lesion is seen in the right lobe. CT is recommended, which shows
a focal mass with nodular hyperenhancement of the periphery on arterial phase imaging
becoming isointense to the background liver on delayed phase scanning at five minutes. Which
one of the following is the most likely diagnosis? [B2 Q 52]

a. Fibrolamellar carcinoma
b. Adenoma c. Cavernous haemangioma
d. Adenocarcinoma metastases
e. Focal nodular hyperplasia

A

**Cavernous haemangioma **

Metastases may show peripheral enhancement with complete fill-in on delayed images, but
they typically show complete rather than nodular peripheral enhancement and washout on
delayed phase imaging. Only haemangiomas typically show peripheral nodular enhancement.
Cavernous haemangiomas are the most common benign liver tumours and are usually less than
4 cm in size. Seventy per cent are hyperechoic on ultrasound and they may show acoustic
enhancement.

151
Q

A 30-year-old woman on the oral contraceptive pill undergoes unenhanced CT of the abdomen,
which demonstrates a wellcircumscribed, slightly hypoattenuating mass in the liver. Which
additional radiological finding would favour a diagnosis of hepatic adenoma rather than focal
nodular hyperplasia? [B4 Q38]

a. measured lesion size of 3 cm
b. accompanying acute subcapsular haematoma
c. transient arterial-phase enhancement
d. normal uptake on 99mTc-labelled sulphur colloid scan
e. hypodense central stellate scar

A

Accompanying acute subcapsular haematoma

Focal nodular hyperplasia (FNH) is a benign hamartomatous malformation commonest in
young women. Lesions are usually smaller than 5cm and contain a central stellate scar in up to
one-third of cases. Hepatic adenomas are benign tumours averaging 8–10 cm in size, seen
predominantly in young women and related to oral contraceptive use. Lesions have a
propensity for spontaneous haemorrhage, presenting as subcapsular haematoma or
haemoperitoneum. FNH, though highly vascular, rarely undergoes spontaneous haemorrhage.
FNH usually contains sufficient functioning Kupffer cells to demonstrate normal or increased
uptake on 99m Tc-labelled sulphur colloid scan, whereas hepatic adenoma, composed of
hepatocytes and non-functioning Kupffer cells, appears as a focal photopenic lesion. Both
lesions demonstrate transient arterial enhancement following intravenous contrast.

152
Q

A 33-year-old female presents to A&E with right upper quadrant pain, hypotensive and
tachycardic. CT abdomen and pelvis reveals an 11 cm diameter well-defined heterogenous
mass within the right lobe of the liver, predominantly of low density but with three focal areas
of higher attenuation (>90HU) within it. There is layered high attenuation fluid within the
subhepatic and right subdiaphragmatic space tracking down to the pelvis. Which one of the
following is the correct combination of recommendations? [B2 Q53]

a. Adenoma– recommend surgical referral
b. Adenoma– recommend correct coagulopathy and rescan if it deteriorates
c. Adenoma– recommend endovascular embolization
d. Metastatic hepatocellular carcinoma– recommend gastroenterology referral
e. Trauma– needs CT thorax to clear other injuries

A

Adenoma – recommend endovascular embolization
Adenomas are vascular lesions comprising hepatocytes. They may occasionally present with
massive haemorrhage and are the most common liver lesion to do so in young people. In this
scenario there is active extravasation of contrast implying active bleeding and haemo-
peritoneum. Urgent embolisation is the most appropriate treatment to halt bleeding.
Conservative or surgical management is unlikely to provide rapid haemostasis. As a proportion
of adenomas become malignant, they are usually removed surgically.

153
Q

A 48-year-old presents with a right upper quadrant mass. He has a history of episodes of
constipation with no weight loss. CT shows a 6 cm mass in the left lobe of the liver, with
peripheral globular cloud-like enhancement. Delayed phase images show the mass ‘has filled
in’ with contrast, with minimal low attenuation central remaining. T2 MRI shows a well-
defined high signal mass in the left lobe of the liver. Which is the most likely diagnosis? [B3
Q9]

A. Cavernous haemangioma
B. Hyper-vascular liver metastasis
C. Focal Nodular Hyperplasia (FNH)
D. Hepatocellular Carcinoma (HCC)
E. Metastatic colon cancer

A

Cavernous haemangioma

Peripheral nodular enhancement with central fill-in is characteristic.

154
Q

A 40 -year-old woman presents with some poorly localising abdominal pain and is investigated
with CT. A non-contrast study shows a 4 cm iso-attenuating mass in the left lobe. Arterial
phase images show the mass to enhance with a small lower attenuation focus centrally. On the
PV phase, the mass is iso-attenuating but the central low attenuated focus persists. What is the
diagnosis? [B3 Q10]

A. FNH
B. Regional nodule
C. HCC
D. Hyper-vascular metastasis
E. Cavernous haemangioma

A

Fibronodular Hyperplasia

Typically, iso on non-contrast, hyperdense on arterial phase and iso-dense on porto-venous
phase. Presence of a central low attenuation scar can be seen in 50% but can also present in
adeno and hepatocellular carcinoma.

155
Q

A 40-year-old woman has an ultrasound for investigation of gallstones. She is otherwise fit and
well. A large lesion is found in the left lobe of the liver. CT is performed for further
characterisation pre-contrast, arterial and porto-venous phases. The 8 cm lesion appears
isointense to liver parenchyma on the non-contrast CT uniformly avidly enhancing in the
arterial phase and again isointense to liver parenchyma on the portal phase. A low attenuated
central scar is noted within the lesion. Which is the most likely diagnosis? [B3 Q18]
A. FNH
B. Hepatic adenoma
C. Atypical haemangioma
D. Focal fat
E. Fibrolamellar carcinoma (FLC)

A

FNH

FLC is usually a solidly lobulated well-defined turn, usually low attenuation on unenhanced
CT and an even lower attenuation central scar, often with punctate calcification. Delayed
enhancement of the scar can occur in both FNH and FLC. However, the scar does not calcify
in FNH.

156
Q

A 50-year-old male with a 2.5-cm hepatocellular carcinoma undergoes RFA. Which of the
following findings is uncommon in the immediate post-ablation period? [B1 Q71]

A. Transient peri-ablational hyperaemia.
B. Small number of tiny intra-lesional air bubbles.
C. Arterio-portal shunting.
D. Ablation zone larger than the primary lesion.
E. ‘Mural nodule in cyst’ pattern

A

Mural nodule in cyst’ pattern
RFA produces thermally induced coagulation necrosis, which manifests usually as an oval or
round defect on contrast-enhanced CT. The ablation zone is slightly larger than the actual
lesion to achieve curative treatment and prevent local recurrence, which is usually seen at the
margins of the ablation zone. The following findings are common in the immediate post
ablation period: transient peri-ablational hyperaemia, tiny air bubbles, and arterio-portal
shunting.

A ‘mural nodule in cyst’ indicates the development of a biloma as a complication of RFA. This
is usually seen several months after treatment. It is associated with interval enlargement of the
RFA zone.

157
Q

A 26-year-old female has an ultrasound scan for right upper quadrant pain and a heterogenous
5cm solitary liver lesion with central calcifications, and a hyperechoic scar is seen. Blood tests
reveal a negative alpha-fetoprotein. MR shows the lesion is hypointense on T1 and
hyperintense on T2-weighted imaging. The central scar is hypointense on both sequences.
Which of the following diagnoses is most likely? [B2 Q17]

a. Hepatic lymphoma
b. Hepatocellular carcinoma
c. Fibrolamellar carcinoma
d. Hepatoblastoma
e. Hepatic angiosarcoma

A

Fibrolamellar carcinoma
Fibrolamellar carcinoma occurs in young adults in the absence of normal risk factors for
hepatocellular carcinoma. On ultrasound, fibrolamellar carcinoma is of mixed or increased
echogenicity, and the hyperechoic central scar is often evident. On unenhanced CT the lesion
is of low attenuation, displaying heterogenous enhancement with intravenous contrast
administration. The central scar is typically of low signal on both T1- and T2-weighted imaging,
which can help differentiate it from FNH (whose scar typically is of low signal on T1 but high
signal on T2-weighted imaging). The central scar is present in up to 60% of patients.
Calcifications are present in up to 55% and are more common than in hepatocellular carcinoma.

158
Q

A 54-year-old male has a liver MR for characterisation of a 3cm low-attenuation lesion found
on staging CT for rectal carcinoma. Which one of the following characteristics would be most
worrying for a metastasis rather than a benign lesion? [B2 Q54]
a. Peripheral washout on delayed imaging
b. Intense arterial enhancement
c. Peripheral nodular enhancement
d. Presence of a pseudocapsule
e. Low signal intensity on T1-weighted imaging

A

Peripheral washout on delayed imaging

Peripheral washout of contrast on delayed imaging is virtually diagnostic of malignancy. On
post-gadolinium-enhanced T1-weighted images most metastases are hypovascular compared
with the surrounding liver and are most conspicuous at the portal phase of enhancement.
However, virtually all metastases exhibit a complete ring of peripheral enhancement, which is
best seen in the early arterial phase.

159
Q

A 51-year-old man with alcoholic cirrhosis presents with jaundice. CT of the abdomen reveals
an encapsulated, 20 mm focal area of low density in the liver, which demonstrates arterial-
phase enhancement and rapid washout on delayed imaging. What is the most likely diagnosis?
[B4 Q48]

a. regenerative nodule
b. dysplastic nodule
c. hepatocellular carcinoma
d. hepatic haemangioma
e. focal fatty sparing

A

Hepatocellular carcinoma

Nodules are a common finding in cirrhosis, and differentiation of benign nodules from
hepatocellular carcinoma (occurring in 7–12% of patients) is vital. Most nodules are
regenerative nodules, representing reparative attempts by hepatocytes in response to liver
injury. These are typically under 10mm in size and appear isodense to liver parenchyma on CT,
unless they contain iron deposits (siderotic nodules), in which case they may be slightly
hyperdense. Dysplastic nodules are proliferative premalignant lesions found in 15–25% of
cirrhotic livers. They resemble regenerative nodules on CT but are usually larger than 10mm.
Hepatocellular carcinomas usually appear as encapsulated hypodense masses that demonstrate
rapid arterial enhancement and early washout of contrast on delayed images. Hepatic
haemangioma usually appears as a low-density mass, but has different enhancement
characteristics, demonstrating peripheral enhancement with complete fill-in on delayed images.
Focal fatty sparing appears as an area of normal density in a generally hypodense liver and
does not demonstrate contrast enhancement

160
Q

A 45-year-old patient with cirrhosis is found to have a focal liver lesion on ultrasound scan,
clinically suspected to be hepatocellular carcinoma. What would be the expected appearances
of the lesion on T2W MR images following infusion of superparamagnetic iron oxide particles?
[B4 Q44]

a. increased signal intensity compared with rest of liver
b. decreased signal intensity compared with rest of liver
c. lesion signal intensity unchanged; rest of liver increased signal intensity d. lesion signal intensity unchanged; rest of liver decreased signal intensity
e. no effect on appearances on T2W images

A

Lesion signal intensity unchanged; rest of liver decreased signal intensity

Superparamagnetic iron oxide (SPIO) particles are iron-based particles of 30–150nm, which,
when administered as an infusion 1–4 hours prior to imaging, act as a negative MR contrast
agent. They target the reticuloendothelial system, being taken up by macrophages throughout
the body, but are preferentially accumulated by the Kupffer cells of the liver. Their
superparamagnetic properties result in T2 and T2* shortening of the tissues that accumulate
the particles, which show reduced signal intensity on T2W, T2* and, to a lesser extent, T1W
images. Most liver tumours do not exhibit uptake, as they are deficient in Kupffer cells.
However, as the rest of the liver accumulates SPIO and darkens preferentially, the tumour
appears of increased conspicuity. SPIO particles are particularly used, in combination with
gadolinium, to improve detection of hepatocellular carcinoma in cirrhotic patients, in whom
the parenchymal changes of fibrosis and regenerative nodules make detection with gadolinium
alone difficult

161
Q

During discussion with a patient due for Radio-frequency Ablation (RFA) treatment of HCC,
she asks you about potential increased risk of tumour seeding along needle track, which she
has read about on the internet. Which of the following statements is most true? [B3 Q25]

A. 0.5% risk of tumour seeding down needle track
B. Lesions with a subscapular location are at lower risk
C. Mortality rates range from 1.5-2%
D. Major complication rates range from 5-10%
E. Minor complication rates typically 10-15%

A

0.5 % risk of tumour seeding down needle track
Uncommon and late complication of RFA. In patients with HCC, tumour seeding occurred in
8/1610 cases in a multicentre study and 1/187 in a single series lesion with subcapsular location.
Invasive tumour pattern/poorly differentiated are at higher risk. Mortality rates are typically
0.1-0.5%, major complications 2.2-3.1%, and minor complications 5-8.9%.

162
Q

A 47-year-old male patient undergoes an MRI examination for further characterization of an
adrenal lesion. Axial gradient T1 in- and out-of-phase sequences confirm the benign nature of
the adrenal lesion. Incidentally, the liver and pancreas demonstrate a signal drop on the in-
phase images compared to out-of-phase images. What is your diagnosis and what additional
sequence would confirm the diagnosis? [B1 Q31]

A. Diffuse fatty infiltration. GE T2WI.
B. Diffuse fatty infiltration. SE T2WI.
C. Haemochromatosis. SE T2WI.
D. Haemochromatosis. GE T2WI.
E. Haemosiderosis. GE T2WI.
F. Haemosiderosis. SE T2WI.

A

Haemochromatosis.GET2WI

A dual GE T1 in- and out-of-phase sequence is routinely used in identifying lipid content within
an adrenal lesion. It is based on the phase interference effect. When the fat and water signals
are in-phase, there is constructive interference and when they are out-of-phase there is
destructive interference. This results in signal drop-off on the out-of-phase sequence.

The reverse effect of decreased signal intensity on in-phase images compared to out-of-phase
ones is seen in iron deposition diseases. This is because the echo time for the in-phase sequence
is longer than for the out-of-phase sequence, therefore the in-phase sequence is more
susceptible to the paramagnetic (dephasing) effects of iron.

Haemochromatosis is an autosomal recessive genetic disorder. There is abnormal deposition
of iron in parenchymal organs such as the liver, pancreas, heart, etc.

In hemosiderosis or secondary haemochromatosis, iron deposition is seen in the
reticuloendothelial system of the liver, spleen, and bone marrow. This type of deposition is not
associated with tissue damage.

A GE T2 sequence demonstrates signal loss due to the magnetic field inhomogeneity produced
by the paramagnetic effects of iron. GE sequences are more susceptible to the paramagnetic
effects than SE sequences, as there is no 180° rephasing pulse in gradient sequences.

163
Q

A40-year-old man with hyperpigmentation, arthalgia and diabetes mellitus is clinically
suspected to have primary haemochromatosis. What are the most likely findings on liver MRI
in this condition? [B4 Q66]

a. normal appearances of the liver
b. decreased signal intensity on T1W and T2W images
c. decreased signal intensity on T1W and increased signal intensity on T2W images
d. increased signal intensity on T1W and decreased signal intensity on T2W images
e. increased signal intensity on T1W and T2W images

A

Decreased signal intensity on T1W and T2W images
In primary haemochromatosis, there is increased duodenal absorption and parenchymal
retention of dietary iron, which is accumulated within the liver, pancreas, heart, and pituitary
gland. Intracellular iron deposits within hepatocytes result in a paramagnetic susceptibility effect, leading to marked shortening of T1 and T2 relaxation times of adjacent protons. This
manifests as a marked reduction in liver signal intensity on T2W and T2 Ã W images, and a
moderate loss of signal intensity on T1W images.

164
Q

A 52-year-old male has an unenhanced CT KUB for left renal colic. No cause for the pain is
discovered on the CT, however the liver is found to be of increased density relative to the
spleen. Which one of the following would be most likely to explain this incidental finding?
[B2 Q37]

a. Excess alcohol intake
b. Amiodarone use
c. Diabetes
d. Steroids
e. Previous chemotherapy

A

Amiodarone use

The normal liver is between 30 and 70 HU on unenhanced CT and should be 10–15 HU lower
than spleen density. On portal venous phase the liver will be approximately 25 HU less than
the spleen. Amiodarone contains iodine and can cause the liver to appear of increased density
on CT. Other causes include cisplatin use, haemochromatosis, Wilson disease and glycogen
storage diseases. The more common finding on CT is a liver of decreased density due to a fatty
liver. This has many causes including alcohol use, steroids, chemotherapy, diabetes and
nutritional causes.

165
Q

A 61-year-old man undergoes CT abdomen and pelvis for characterisation of a well-defined
hyperechoic area seen on ultrasound in the perihilar region of the liver. On CT, the area is of
decreased attenuation but has no obvious mass effect. There is no abnormal enhancement with
intravenous contrast administration. Which one of the following diagnoses is most likely? [B2
Q44]

a. Focal nodular hyperplasia
b. Focal fatty infiltration
c. Hepatic cyst
d. Liver haemangioma
e. Fibrolamellar carcinoma

A

Focal fatty infiltration

Focal fatty infiltration occurs typically in the periportal and centrilobar regions of the liver and
is commonest adjacent to the falciform ligament. Ultrasound features include a hyperechoic
area with geographic margins. CT shows an area of decreased attenuation which does not alter
the course of blood vessels or liver contour. The lesions are of high signal on T1-weighted MR
imaging, and isointense or low signal on T2-weighted imaging. Haemangiomas would also
typically be of increased echogenicity on ultrasound, but would be expected to show increased
peripheral enhancement with intravenous contrast on CT.

166
Q

A 45-year-old woman undergoes a follow-up staging CT of the chest, abdomen, and pelvis
after treatment for metastatic breast cancer. Compared with her initial staging scan, there is a
generalized decrease in the attenuation value of the liver. No focal liver lesion or other new
feature is seen. What is the most likely cause? [B4 Q91]

a. diffuse metastatic disease
b. fatty liver related to chemotherapy
c. hepatic venous congestion
d. amyloidosis
e. Budd–Chiari syndrome

A

Fatty liver related to chemotherapy

Chemotherapeutic agents are commonly associated with fatty liver. Diffuse fat deposition (the
commonest pattern) causes a generalized decrease in the attenuation value of the liver on CT,
and may be diagnosed with an absolute liver attenuation value of less than 40 HU on contrast-
enhanced CT. It may also be diagnosed at unenhanced CT if the liver attenuation value is at
least 10 HU less than that of the spleen. Liver metastases usually present as focal, low-
attenuation lesions on portal phase imaging. Hepatic venous congestion causes a diffuse
decrease in attenuation but is associated with enlargement of the inferior vena cava and hepatic
veins due to elevated central venous pressure. Amyloidosis can cause a generalized decrease
in liver attenuation, but more commonly appears as discrete areas of low attenuation with
reduced contrast enhancement. Budd–Chiari syndrome may also result in a diffuse decrease in
liver attenuation, but there is usually patchy liver enhancement and poor visualization of the
hepatic veins.

167
Q

A 44-year-old man undergoes ultrasound of the abdomen during which the liver is incidentally
noted to be of diffusely increased echogenicity, with attenuation of the ultrasound beam and
poor visualization of the intrahepatic architecture. Which of the following imaging features is
most likely in this condition? [B4 Q6]

a. liver echogenicity less than that of renal cortex on ultrasound scan
b. relatively hypo-attenuated intrahepatic vessels on unenhanced CT
c. liver attenuation 10 HU greater than that of the spleen on unenhanced CT
d. absolute liver attenuation of .40 HU on contrast-enhanced CT
e. loss of liver signal intensity on out-of-phase gradient echo MR images

A

Loss of liver signal intensity on out-of-phase gradient echo MR images

Fatty liver describes a spectrum of conditions characterized by triglyceride accumulation
within hepatocytes. It is common, affecting around 15% of the general population, but is more
prevalent among those with obesity, hyperlipidaemia, and high alcohol consumption. Fatty
liver may be diagnosed on ultrasound scan if liver echogenicity exceeds that of renal cortex,
with attenuation of the ultrasound beam, loss of definition of the diaphragm and poor
visualization of the intrahepatic architecture. CT features include absolute attenuation of less
than 40 HU on contrast-enhanced CT and, on unenhanced CT, liver attenuation at least 10 HU
less than that of spleen, and relatively hyperattenuating liver vasculature. Chemical shift
gradient echo imaging is the most widely used MR technique for assessment of fatty liver,
demonstrating Fatty liver describes a spectrum of conditions characterized by triglyceride
accumulation within hepatocytes. It is common, affecting around 15% of the general population, but is more prevalent among those with obesity, hyperlipidaemia and high alcohol
consumption. Fatty liver may be diagnosed on ultrasound scan if liver echogenicity exceeds
that of renal cortex, with attenuation of the ultrasound beam, loss of definition of the diaphragm
and poor visualization of the intrahepatic architecture. CT features include absolute attenuation
of less than 40 HU on contrast-enhanced CTand, on unenhanced CT, liver attenuation at least
10 HU less than that of spleen, and relatively hyperattenuating liver vasculature. Chemical shift
gradient echo imaging is the most widely used MR technique for assessment of fatty liver,
demonstrating

168
Q

Which is the most common MR feature of autoimmune hepatitis? [B3 Q31]

A. MR is diagnostic test for AIH
B. Surface nodules rarely present
C. PV thrombosis is frequently present
D. Fibrosis better evaluated on CT than MR
E. Enlarged pre-portal space is a recognised finding

A

Enlarged pre-portal space is a recognised finding
Although AIH has a wide variability in imaging appearances, from normal to end-stage chronic
liver disease, the most common feature on CT and MR in one series is surface nodularity.

169
Q

Regarding Wilson’s disease: [B3 Q34]

A. Is autosomal dominant in inheritance
B. Hyperdense nodules on non-enhanced CT is the most common finding
C. Honeycomb pattern is most evident on the pre-contrast study
D. The honeycomb pattern is more evident on T1 than T2
E. Occurs due to overproduction of copper

A

Hyperdense nodules on non-enhanced CT is the most common finding
Most cases demonstrate hyperdense nodules on the non-contrast study, some of which became
iso-dense with contrast. A honeycomb appearance is seen in portal and parenchymal phases
and on T2 sequences on MR.

170
Q

Which of the following is a cause of generalised increase in density pre-IV contrast on CT?
[B3 Q40]

A. Amiodarone treatment
B. Fatty infiltration
C. Malignant infiltration
D. Budd-Chiari
E. Amyloidosis

A

Amiodarone treatment

Fatty infiltration, malignant infiltration, Budd-Chiari and amyloidosis all demonstrate
generalized low density on pre-contrast CT.

171
Q

A 25-year-old man presents with jaundice and malaise. Ultrasound scan demonstrates a general
decrease in liver echogenicity and a well-distended gallbladder with a wall thickness of 4 mm.
No gallstones are seen, and the intra- and extrahepatic bile ducts appear normal. What is the
most likely diagnosis? [B4 Q39]

a. acute cholecystitis
b. cirrhosis
c. fatty liver
d. acute viral hepatitis
e. primary sclerosing cholangitis

A

Acute viral hepatitis

Acute hepatitis results in a diffuse decrease in liver echogenicity on ultrasound scan, with
increased brightness of the portal triads resulting in a ‘starry sky’ appearance. Other imaging
features include oedema of the gallbladder fossa and gallbladder wall thickening. Gallbladder
wall thickening (anterior wall thickness .3mm in a non-contracted gallbladder) may be seen in
a wide range of intrinsic and extrinsic conditions. The commonest intrinsic cause is
cholecystitis (acute and chronic), whereas common extrinsic causes include hepatitis,
hypoalbuminaemia, heart failure and renal failure

172
Q

A 86-year-old, otherwise well woman is admitted with abdominal pain and undergoes plain
abdominal radiography. This demonstrates a normal bowel gas pattern, but the liver and spleen
are noted to be of increased density with a stippled appearance. What is the most likely cause?
[B4 Q46]

a. haemochromatosis
b. thorotrastosis
c. amiodarone therapy
d. sickle cell anaemia
e. glycogen storage disease

A

Thorotrastosis

Thorotrast (thorium dioxide), an alpha-emitting radioactive
isotope of atomic number 90 and long half-life, was used as a
contrast agent until the mid-1950s, predominantly for cerebral
angiography and reticuloendothelial imaging. It is retained
indefinitely by the reticuloendothelial system, and results in
increased density of the liver, spleen, and lymph nodes with a
characteristic stippled appearance. It is associated with delayed
malignancies, including angiosarcoma, cholangiocarcinoma
and hepatocellular carcinoma. Haemochromatosis may result
in diffusely increased density of the liver and spleen, but
usually presents earlier in life. Amiodarone may result in
increased liver attenuation of 95–145 HU (normal 30–70 HU),
but splenic involvement is not usually a feature. Sickle cell anaemia can result in a shrunken
calcified spleen, but again is unlikely in this age group. Glycogen storage disease can result in
a generalized increase or decrease in hepatic density on CT, but increased splenic density is
not a feature.

Case courtesy of Dr Rohit Sharma, Radiopaedia.org, rID: 73597

173
Q

A 65-year-old male with a pancreatic head mass and obstructive jaundice undergoes
percutaneous cholangiogram and external biliary drain insertion via the right lobe of the liver.
The patient returns for a biliary stent insertion. On removing the external drain there is
significant arterial bleed from the puncture site. A selective coeliac axis angiogram does not
reveal any abnormality, but pulsatile bleeding persists. What would you do next? [B1 Q28]

A. Selective left gastric angiogram.
B. Selective superior mesenteric angiogram.
C. Selective inferior mesenteric angiogram.
D. Selective gastroduodenal artery angiogram.
E. Embolise coeliac axis.

A

Selective superior mesenteric angiogram

Variations in hepatic arterial anatomy are common. According to Michel classification, the
classic hepatic arterial anatomy with the hepatic artery proper dividing into the right and left
hepatic arteries is seen in only 55% of the population. A replaced right hepatic artery from the
SMA is seen in 11% and an accessory right hepatic artery from the SMA is seen in 7% of the
population. A selective SMA angiogram should therefore be performed in this case.

A selective left gastric angiogram is not required as it is a branch of the coeliac axis. The
inferior mesenteric artery does not supply the liver.

The coeliac axis divides into the common hepatic, left gastric, and splenic arteries.
Embolization of the coeliac axis is therefore not an option.

174
Q

A 62-year-old male with acute myocardial infarction develops abdominal discomfort and
deranged liver function tests. A CT scan of the abdomen demonstrates heterogeneous liver
enhancement, poor enhancement of the hepatic veins and inferior vena cava (IVC), ascites and
bibasal pleural effusions. What additional feature would favour a diagnosis of passive hepatic
congestion instead of acute Budd–Chiari syndrome? [B1 Q30]

A. Flip-flop enhancement pattern of the liver.
B. Absent flow in hepatic veins.
C. Dilated hepatic veins and IVC.
D. Enlarged caudate lobe.
E. Hepatomegaly.

A

Dilated hepatic veins and IVC
Elevated right atrial/central venous pressure due to cardiac decompensation results in impaired
venous drainage from the liver, producing passive hepatic congestion. If prolonged, passive
hepatic congestion can result in cardiac cirrhosis. On CT imaging, retrograde enhancement of
dilated IVC and hepatic veins is seen in the arterial phase. In the portal venous phase, there is
delayed/reduced enhancement of the hepatic veins due to impaired venous drainage. There is
heterogenous enhancement of the liver parenchyma due to venous stasis. Other features of
cardiac failure may be evident. Acute Budd–Chiari syndrome is characterized by narrowed hepatic veins and intrahepatic IVC
(secondary to compression by the enlarged liver) and by flip-flop pattern of enhancement
between the arterial and the portal venous phases.

Budd-Chiari Syndrome [STATdx]

Greyscale Ultrasound Findings:

  • Hypertrophied caudate lobe (which is spared)
  • Narrowed hepatic veins, hard to visualised
  • Thrombus in hepatic vein

Colour Doppler Ultrasound Findings:

  • Hepatic veins and IVC
    o Absent or flat flow in hepatic veins
    o Reversed flow in hepatic vein or IVC
    o Bicoloured hepatic veins due to intrahepatic collateral pathways
  • Portal vein
    o Slow hepato-fugal flow
  • Hepatic artery
    o RI>0.75

NECT Findings:

  • Acute Phase
    o Enlarged, diffusely hypodense liver with ascites
    o Narrowed IVC and hepatic veins
    o Hyperdense IVC and hepatic vein (thrombus)
  • Chronic Phase
    o Heterogeneous hypodensity and atrophy of peripheral liver
    o Hypertrophy of the spared caudate lobe (often bigger than the right lobe)
    o Non-visualization of IVC and hepatic veins

CECT Findings:

  • Acute Phase
    o Classic flip-flop pattern
    ▪ Early → caudate and peri-IVC portion enhance more than peripheral liver
    ▪ Late → peripheral liver enhances more than central.
    o Narrowed hypodense hepatic veins and IVC with hyperdense walls.
  • Chronic Phase
    o Total obliteration of hepatic veins and IVC
    o Large multiple regenerative nodules
175
Q

A 35-year-old woman on the oral contraceptive pill presents with right upper quadrant pain,
shortness of breath and leg oedema. Ultrasound scan of the abdomen demonstrates
hepatosplenomegaly and ascites. The hepatic veins are not visualized on Doppler ultrasound
scan. What is the most likely diagnosis? [B4 Q45]
a. acute Budd–Chiari syndrome
b. primary biliary cirrhosis
c. passive hepatic congestion
d. hepatic veno-occlusive disease
e. viral hepatitis

A

Acute Budd Chiari Syndrome

Budd–Chiari syndrome is caused by obstruction of hepatic venous outflow, which may, in turn,
be caused by membranous obstruction of the suprahepatic IVC by a congenital web, or hepatic
venous thrombosis due to hypercoagulable state, tumour or trauma. Patients develop
hepatosplenomegaly and intractable ascites. Doppler ultrasound scan demonstrates non-
visualization of, or thrombus within, one or more hepatic veins. CT findings reflect severely
impaired blood flow to the liver, with a ‘flip-flop’ enhancement pattern after contrast
administration. Early images show prominent central liver enhancement with poor peripheral
enhancement, whereas delayed images show central washout with peripheral enhancement.
The caudate lobe is typically spared because of its separate venous drainage directly into the
IVC, and enhances normally. Passive hepatic congestion complicates heart failure, and results
in distended hepatic veins and IVC. Hepatic veno-occlusive disease refers to occlusion of small
centrilobular hepatic veins following radio- and chemotherapy in bone-marrow transplant
recipients, or related to alkaloid consumption. The main hepatic veins and IVC are normal.
Hepatic venous involvement is not a feature of viral hepatitis or primary biliary cirrhosis.

176
Q

A 50-year-old male patient is admitted with congestive cardiac failure and undergoes a CT
scan of the abdomen, which shows tortuous and prominent intrahepatic and extrahepatic
arterial branches with early filling of dilated hepatic veins and IVC. The arterial phase scan
shows mosaic perfusion with multiple enhancing foci. In the portal venous phase there is
homogenous enhancement of the liver, with the prominent hepatic veins and IVC noted. What
is the diagnosis? [B1 Q51]

A. Passive hepatic congestion.
B. Budd–Chiari syndrome.
C. Osler–Weber–Rendu syndrome.
D. Multifocal transient hepatic attenuation differences.
E. Von Meyerburg complex.

A

Osler–Weber–Rendu syndrome

Osler–Weber–Rendu syndrome or hereditary haemorrhagic telangiectasia (HHT) is a rare
autosomal dominant multisystem vascular disorder characterized by angio-dysplastic lesions
in which there is communication between arteries and veins of varying sizes. It commonly
affects the skin, lungs, and mucous membranes but any organ system may be involved.

The liver is the most common site of abdominal HHT. Lesions range from tiny telangiectasis
to transient perfusion abnormalities and large confluent vascular masses. Coronal maximum
intensity projection (MIP) images are useful in appreciating telangiectasis.

Liver involvement is associated with arterio-venous shunting, porto-venous shunting, or both,
resulting in hyperdynamic circulation, which may lead to high-output cardiac failure.
Budd–Chiari syndrome is hepatic vein thrombosis and Von Meyerburg complex is multiple
biliary hamartomas.

Osler–Weber–Rendu syndrome [STATdx]

  • Hereditary Haemorrhagic Telangiectasia
  • Autosomal dominant
  • Multi-system angio-dysplastic lesions
  • Mucocutaneous telangiectasias – scalp, nasopharynx, orbit
  • Visceral organs AVMs – lungs, brain, liver

Hepatic Imaging

Ultrasound – screening for hepatic involvement in HHT

Grey scale ultrasound – Dilated hepatic arteries, abnormal echoes

Pulsed doppler – High hepatic artery velocity with low resistive index
* Hepatic artery – Portal vein shunt → Pulsatile portal flow
* Hepatic artery – Hepatic vein shunt (Severe disease) → abnormal waveform

Colour doppler
* Tangled masses of enlarged tortuous arteries or
* Multiple aneurysms of hepatic arteriole branches within liver

CT
* Large tortuous extra and/or intra-heaptic arteries
* Early filing of hepatic and portal veins
* Enlargement of hepatic and portal veins
* Heterogeneous enhancement of hepatic parenchyma – THAD – wedge shaped
peripheral region of hyperenhancement in arterial phase
* Telangiectasias – small vascular spots
* Large confluent vascular masses – early and persistent enhancement in subsequent
phases → high output heart failure

MRI
* T1WI: Telangiectasia – hypo- or isointense; Regenerative nodules – bright
* T2WI: Telangiectasia – network of flow voids; Regenerative nodules – dark
* T1 C+

177
Q

A 55-year-old male liver transplant recipient undergoes Doppler ultrasound assessment at 1
year for deranged liver function tests. Colour Doppler imaging demonstrates a stenosis in the
hepatic artery, at the presumed anastomosis. Which of the following statements regarding the
associated pulsed Doppler findings is likely to be false? [B1 Q70]

A. Tardus-parvus arterial waveform distal to the stenosis.
B. Resistive index of 0.9 distal to the stenosis. C. Spectral broadening in the immediate post-stenotic portion.
D. Elevated peak systolic velocity at the stenosis.
E. Elevated end diastolic velocity at the stenosis

A

Resistive index of 0.9 distal to the stenosis

Knowledge of the vessel ‘waveform signature’ and stenosis flow dynamics is essential in
interpreting the Doppler findings. The hepatic artery is a low resistance vessel with continuous
antegrade flow during systole and diastole. Generally, low-resistance arteries have a resistive
index (RI) of 0.55–0.7. RIs higher or lower than this range are abnormal. RI is calculated by
using the formula RI = (peak systolic velocity – end diastolic velocity)/peak systolic velocity.
Most modern ultrasound machines calculate this automatically.

A stenosis results in increased arterial resistance to the blood flow proximal to the stenosis.
This causes a reduction in the end diastolic velocity disproportionately more than the peak
systolic velocity, producing a high-resistance waveform and high RI (due to a greater
difference between the peak systolic and end diastolic velocities). This finding is not specific
to stenosis and may be seen in the postprandial state, and in patients of advanced age and
diffuse peripheral microvascular disease or compression (cirrhosis, hepatic venous congestion,
cold ischaemia, and any stage of transplant rejection).

At and immediately distal to the stenosis there is turbulent flow and a jet phenomenon resulting
in an increase in the peak systolic and end diastolic velocity and spectral broadening.
Depending on the severity of the stenosis, the artery distal to the stenosis demonstrates the
following findings: tardus-parvus waveform that refers to the late and low systolic peak (i.e.,
increased acceleration time and reduced peak systolic velocity). Low RI due to a greater
reduction in peak systolic velocity compared to the end diastolic velocity.

Low RI may also be seen with distal vascular shunts (trauma, iatrogenic, cirrhosis, Osler–
Weber-Rendu syndrome).

178
Q

A42year old woman undergoes a CT abdomen and pelvis for the investigation of right upper
quadrant pain and deranged liver function tests. On early post-intravenous contrast images
there is prominent enhancement of the central liver and weak enhancement of the peripheral
liver. This pattern is reversed on delayed images. In addition, there is hypertrophy of the
caudate lobe. Which one of the following would most likely explain these findings? [B2 Q43]

a. Acute hepatitis
b. Cirrhosis
c. Budd–Chiari syndrome
d. Portal hypertension
e. Fatty liver

A

Budd–Chiari syndrome
Budd–Chiari syndrome is outflow obstruction of the hepatic veins due to a wide variety of
causes, but two-thirds are idiopathic. CT features include ‘flip-flop’ enhancement pattern as
described in the question, ascites, hepatosplenomegaly, gallbladder wall thickening and
increased portal vein diameter. An enlarged caudate lobe is seen in up to 88%, which enhances
normally due its venous drainage passing directly into the IVC.

179
Q

A 25-year-old female becomes unwell six hours after induced delivery for pre-eclampsia with
severe right upper quadrant pain, oedema, and nausea. CT of the abdomen and pelvis reveals
copious ascites and multiple wedge-shaped areas of liver non-enhancement consistent with
hepatic infarction. Which of the following is the most likely underlying cause? [B2 Q51]

a. Hepatic artery embolus
b. Portal vein thrombosis
c. SVC occlusion
d. HELLP syndrome
e. Splenic vein thrombosis

A

HELLP syndrome
Hepatic infarction is rare because of the dual blood supply to the liver via the hepatic arterial
system and the portal venous system. Isolated pathology in either of these vascular supplies is
unlikely to cause hepatic necrosis as the other supply will usually compensate. HELLP is
characterised by haemolysis, elevated liver enzymes and low platelets and is one of the causes
of liver infarction

180
Q

A 45-year-old woman undergoes abdominal ultrasound scan. The portal vein measures 16 mm
in diameter and demonstrates continuous monophasic flow without respiratory variation. Portal
vein flow velocity is hepatopetal and is measured to be 7 cm/s. What is the most likely
diagnosis? [B4 Q72]

a. normal findings
b. Budd–Chiari syndrome
c. portal hypertension
d. cavernous transformation of the portal vein
e. portal vein thrombosis

A

Portal hypertension
The normal portal vein measures up to 13mm in diameter when measured in the AP direction
where the portal vein crosses the inferior vena cava during quiet respiration in a supine patient.
Portal venous flow is normally 12–30cm/s and demonstrates respiratory variation but little or
no pulsatility, though this may be seen in thin patients. Normal flow is hepato-petal
(anterograde flow into the liver). Portal hypertension is defined as an increase in portal venous
pressure above 10mmHg and is most caused by cirrhosis in the western world. As portal
pressure increases, portal vein diameter increases, and portal flow loses its respiratory
fluctuation and becomes slow and turbulent. Reversed (hepato-fugal) flow may occur in 8% of
patients and is generally associated with a reduced portal vein diameter. Other findings include
portosystemic collaterals, splenomegaly, and ascites. In portal vein thrombosis, portal vein
diameter increases, but no flow is seen on Doppler ultrasound scan. Echogenic thrombus may
be seen within the lumen. Cavernous transformation of the portal vein may occur with chronic
portal vein thrombosis, representing a conglomerate of collateral veins. Budd–Chiari syndrome
affects the hepatic veins.

181
Q

A 45-year-old man 3 weeks post-op for liver transplantation, develops non-specific abdominal
pain with a slight rise in liver function tests. A Doppler ultrasound is requested. Which is the
most common vascular complication of orthoptic liver transplantation? [B3 Q48]

A. Hepatic artery thrombosis
B. Hepatic vein thrombosis
C. Portal vein thrombosis
D. IVC thrombosis
E. Hepatic artery stenosis

A

A Hepatic artery thrombosis occurs in 2-12% of cases, usually between 15-132 days post-
operatively. The donor celiac axis is anastmosed to the recipient hepatic artery in orthoptic
liver transplants. This can be either at the bifurcation of the hepatic into the right and left
hepatic arteries, or at the take-off of the gastroduodenal artery. As the hepatic artery is the only
vascular supply to the bile ducts, hepatic artery thrombosis and stenosis can lead to biliary
ischemia

182
Q

A 45-year-old man undergoes ultrasound scan of the abdomen 2 days following orthotopic
liver transplantation, which demonstrates periportal oedema and a small fluid collection at the
hilum of the liver. What is the most likely diagnosis? [B4 Q58]

a. graft rejection
b. hepatic arterial thrombosis
c. portal vein stenosis
d. bile leak
e. normal post-transplantation findings

A

Normal post-transplantation findings
Orthotopic liver transplantation is the treatment of choice for patients with end-stage liver
disease for which no other therapy is available. Surgery involves one arterial anastomosis
(hepatic artery), at least two venous anastomoses (portal vein and IVC) and a biliary
anastomosis, and complications may occur at any of these sites. Vascular complications are the
most frequent cause of graft loss, and most commonly involve the hepatic artery, with portal
venous and IVC complications being relatively infrequent. Biliary complications occur in up
to 34% of cases and are the second most common cause of liver dysfunction after graft rejection.
They include leak, stricture, and obstruction. Other complications include fluid collections,
infection, and malignancy. Normal findings following liver transplantation include a small
amount of free intraperitoneal fluid in the perihepatic region, especially at the hilum, or in the
fissure for the ligamentum teres, which usually resolves within a few weeks. Other normal
findings are a right fluid pleural effusion, and periportal oedema, attributed to lymphatic
channel dilatation due to lack of normal lymphatic drainage.

183
Q

A patient with a history of alcohol abuse presents to A&E with epigastric pain. His
haemoglobin is 8 g/dl on admission. An oesophago-gastroduodenoscopy (OGD) and
ultrasound of abdomen are requested. The ultrasound of abdomen shows multiple hyperechoic
lesions in the liver. There is no evidence of gallstones. Prior to the OGD the patient becomes
acutely unwell, and a CT scan is requested. This shows evidence of air and fluid in the
subhepatic space. It also reveals a focal enhancing lesion causing prominence of the head of
the pancreas. What is the most likely diagnosis? [B1 Q3]

A. Pancreatitis with associated peripancreatic abscess.
B. Pancreatic carcinoma and liver metastases.
C. Pancreatitis and peptic ulcer perforation in an alcoholic patient.
D. Islet cell tumour with liver metastases.
E. Cholangiocarcinoma with liver metastases.

A

Islet cell tumour with liver metastases

Whilst this tumour is rare, the CT findings indicate a focal mass lesion in the head of the
pancreas. Hyperechoic metastases in the liver are suggestive of islet cell tumour, rather than
pancreatic carcinoma metastases, even though pancreatic carcinoma is more common. The islet
cell tumour could be a gastrinoma, which is most found in the head of the pancreas and is
malignant in 60%. It is also associated with peptic ulcer disease and the finding of air and fluid
in the subhepatic space suggests a perforated duodenal ulcer.

Causes of hyper-echoic liver lesion

Benign

  • Hepatic haemangioma
  • FNH
  • Hepatic adenoma with high fat content
  • Focal fatty change
  • Hepatic AML
  • Inflammatory pseudotumor of the liver
  • Lipoma

Malignant

  • Hepatic mets
    o Colorectal
    o Treated breast
    o PNET
    o RCC
    o Thyroid
    o Melanoma
    o Choriocarcinoma
  • HCC especially in cirrhotic liver
  • Cholangiocarcinoma
184
Q

A 71-year-old woman with no significant past medical history has an abdominal ultrasound as
part of an investigation for right upper quadrant pain, anaemia, and weight loss. Multiple,
poorly defined, markedly echogenic lesions are seen throughout the liver. Biopsy reveals these
to be metastases. Which one of the following is most likely to be the primary tumour? [B2 Q49]

a. Adenocarcinoma of the colon
b. Melanoma
c. Invasive ductal carcinoma of the breast
d. Gastric cancer
e. Pancreatic ductal adenocarcinoma

A

Adenocarcinoma of the colon
The most common primary tumours that cause brightly echogenic liver metastases are colonic
adenocarcinoma, treated breast cancer and hepatoma. The differential here therefore lies
between breast cancer and colon cancer. Colon cancer makes up at least 50% of highly
echogenic metastases. In addition, the question states that the patient has no significant past
medical history, and therefore treated breast cancer is unlikely.

185
Q

A 54-year-old woman undergoes CT of the abdomen and pelvis for weight loss and is found to
have multiple, irregular, calcified, low-attenuation lesions in the liver, suggestive of metastases.
What is the most likely primary lesion? [B4 Q69]

a. invasive ductal carcinoma of the breast
b. mucinous adenocarcinoma of the gastrointestinal tract
c. osteosarcoma
d. non-small-cell lung carcinoma
e. carcinoid

A

Mucinous adenocarcinoma of the gastrointestinal tract

Calcified liver metastases represent up to 3% of liver metastases and are most commonly seen
with mucinous carcinomas of the gastrointestinal tract. They are also seen in osteosarcoma,
breast cancer, lung cancer and carcinoid, but these are less common

186
Q

A 63-year-old male has a CT abdomen and pelvis for the investigation of change in bowel habit
and weight loss. A sigmoid tumour is demonstrated and there is a solitary liver metastasis.
Which one of the following observations on CT would render the patient ineligible for curative
resection of the liver metastasis? [B2 Q55]

a. Presence of a single peripheral left lower lobe pulmonary metastasis
b. Presence of splenic metastasis
c. Direct extension of the liver metastasis into the right adrenal gland
d. Involvement of the caudate lobe
e. Peritoneal metastases

A

Peritoneal metastases

Contraindications to liver resection:
Uncontrollable extrahepatic disease such as: non-treatable primary tumour.
Widespread pulmonary disease.
Locoregional recurrence
Peritoneal disease
Extensive nodal disease, such as retroperitoneal, mediastinal, or portal nodes.
Bone or CNS metastases.

Patients with extrahepatic disease that should be considered for liver resection include resectable/ablatable pulmonary metastases; resectable/ablatable isolated extrahepatic sites – for example, spleen, adrenal or resectable local recurrence; and local direct extension of liver metastases to, for example, the diaphragm or adrenal glands, which can be resected.

187
Q

Which of the following causes of diffuse hepatic surface nodularity can be most difficult to
distinguish from cirrhosis?

A. Treated breast cancer metastases to the liver
B. Chronic Budd-Chiari syndrome
C. Chronic portal vein thrombosis
D. Pseudomyxoma peritonei
E. Miliary metastases

A

Treated breast cancer metastases to the liver
Post-chemotherapy changes to hepatic metastases from breast cancer can cause fine diffuse
nodularity resembling cirrhosis known as ‘pseudocirrhosis.’ B-D cause coarse nodularity rather
than fine and diffuse nodules in cirrhosis. Mets is unlikely to mimic cirrhosis.

188
Q

A 40-year-old male presents with an indeterminate liver lesion. He is Hep C positive with
previous history of Intravenous Drug Use (IVDU) and was successfully treated for T2NO
colorectal cancer last year. Ultrasound demonstrates a 1.5cm slightly hypoechoic lesion in
segment 8. This lesion takes up contrast inhomogenously in the arterial phase and demonstrates
washout in the porto-venous and delayed phases, becoming hypoechoic with regards to
surrounding liver parenchyma. Which is the most likely diagnosis? [B3 Q29]

A. Metastasis
B. Focal fatty sparing
C. Capillary haemangioma
D. Hepatic abscess
E. Regenerate nodule

A

Rapid washout and residual hypoechoic lesions are features of metastatic deposit.

189
Q

A 26-year-oldmanwithAIDSpresentswithweightloss.Heisnoted to have multiple raised purple
skin lesions on examination. Contrast-enhanced CT of the abdomen and pelvis demonstrates
multiple, sub centimetre, low-attenuation nodules within the liver, as well as high-attenuation
lymphadenopathy at the porta hepatis, retrocaval and aortocaval regions. What is the most
likely diagnosis? [B4 Q86]

a. fungal infection
b. multiple haemangiomas
c. lymphoma
d. Kaposi’s sarcoma
e. mycobacterial disease

A

Kaposi’s sarcoma

Kaposi’s sarcoma is a low-grade tumour of the blood and lymphatic vessels that primarily
affects the skin but may cause disseminated disease in other organs. It is an AIDS-defining
illness and is the most common AIDS-related neoplasm. The commonest manifestation is of
multiple raised purplish skin lesions, but lymphadenopathy is the second commonest feature
in AIDS-related Kaposi’s sarcoma. Typical appearances are of abdominopelvic lymph nodes
that enhance after intravenous contrast due to high vascularity, appearing to be of higher
attenuation than skeletal muscle. Liver involvement occurs in 34% of cases at autopsy, and
typically causes multiple 5–12mm nodules that are hyperechoic on ultrasound scan and of low
attenuation on CT. Skin lesions are present in most cases and help to distinguish Kaposi’s
sarcoma from other conditions such as fungal micro-abscesses and multiple haemangiomas,
which may have similar appearances on CT. Mycobacterial disease is characteristically
associated with low attenuation lymphadenopathy. Non-Hodgkin’s lymphoma is the second
commonest AIDS-related neoplasm, and may cause multiple low-attenuation liver lesions, but
it is not associated with skin lesions or high-attenuation lymphadenopathy

190
Q

A 50-year–old male is admitted under the surgical team having presented with upper abdominal
pain and raised inflammatory markers. Suspecting acute cholecystitis, an ultrasound is
requested, but due to large body habitus there is poor visualization of his gallbladder. To further
evaluate hepatobiliary scintigraphy using 99mTc-labelled iminodiacetic acid is arranged.
Which of the following findings are consistent with acute cholecystitis? [B1 Q75]

A. Non-visualization of the gallbladder at 1 and 4 hours.
B. Non-visualization of the gallbladder at 1 hour but seen at 4 hours.
C. Visualization of the gallbladder at 1 hour.
D. Visualization of the gallbladder at 30 minutes after morphine administration.
E. Hepatobiliary scintigraphy is not appropriate for investigation of acute cholecystitis.

A

Non-visualization of the gallbladder at 1 and 4 hours

Hepatobiliary scintigraphy is most used to evaluate suspected acute cholecystitis.

A minimum of 2 hours fasting is required. Following prompt uptake by the liver, the radiotracer
is excreted into the biliary system and drains into the small bowel. Activity should be
demonstrated within the gallbladder by 1 hour. Morphine can be used during the scan to relax
the sphincter of Oddi, thus pushing radiolabelled bile into the gallbladder. Acute cholecystitis
is characterized by non-visualization of the gallbladder at both 1 and 4 hours or at 30 minutes
following morphine administration. Non-visualization of the gallbladder at 1 hour, but seen at
4 hours, is indicative of chronic cholecystitis. A false-positive diagnosis of acute cholecystitis
can occur with previous cholecystectomy, gallbladder agenesis, and tumour obstructing the
cystic duct.

191
Q

Which of the following is a cause of non-visualisation of the gall bladder with trimethylbromo-
im–indolacetic acid (TBIDA) and no bowel activity? [B3 Q41]

A. Opiate usage
B. Acute cholecystitis
C. IV feeding
D. Biliary pancreatitis
E. Severe diffuse hepatocellular disease

A

Opiate usage
Opiates, severe hepatitis and CBD obstruction are causes of non-visualisation and no bowel
activity. B-D cause bowel activity.

192
Q

During MRCP, which substance may be administered to improve visualization of the
pancreatic ducts? [B4 Q63]

a. glucagon
b. secretin c. cholecystokinin
d. gastrin
e. Buscopan

A

Secretin
Secretin is a hormone normally secreted by the duodenal mucosa in response to acid within the
lumen. It has many physiological effects on the gastrointestinal tract, including stimulation of
pancreatic secretions and a transient increase in tone of the sphincter of Oddi. When given
intravenously immediately prior to imaging, it results in distension of the pancreatic ductal
system and can significantly improve visualization of the pancreatic main and branch ducts on
MRCP, as well as provide information about the secretory reserve capacity of the pancreas.
Effects on the biliary tree are less pronounced and not usually appreciable on imaging. Side
effects include abdominal pain, bloating and diarrhoea. Glucagon increases bile flow and has
been suggested to improve visualization of the biliary tree at MRCP. Cholecystokinin
stimulates gallbladder contraction and is sometimes used in hepatobiliary scintigraphy.
Buscopan is widely used in imaging for its inhibition of intestinal motility.

193
Q

A neonate is investigated for obstructive jaundice and as part of the investigation has a
hepatobiliary iminodiacetic acid (HIDA) nuclear medicine scan. This shows a photopaenic area
within the liver and lack of visualisation of the small bowel. Which one of the following
conditions would be most consistent with these findings? [B2 Q24]

a. Enteric duplication cyst
b. Biliary duct atresia
c. Choledochal cyst
d. Pancreatic pseudocyst
e. Hepatic cyst

A

Choledochal cyst

The only one of the listed diagnoses that would have both these features on HIDA scan is a
choledochal cyst. This is a congenital condition characterised by dilatation of the common bile
duct and common hepatic duct. Patients typically present in childhood with right upper
quadrant pain, a mass and/or obstructive jaundice. Although the diagnosis is usually made with
MRCP, HIDA scan can show typical features that include a
photopaenic area in the liver representing the dilated CBD/CHD.
Although a hepatic cyst would also show a photopaenic area
within the liver, small bowel visualisation would be expected.
Congenital biliary atresia would cause lack of small bowel
visualisation, but the whole liver would take up HIDA and
photopaenia would not be present.

194
Q

A 64-year-old woman presents with jaundice. An abdominal ultrasound scan demonstrates the
intrahepatic biliary ducts to be of similar calibre to the adjacent portal veins. The extrahepatic common bile duct measures 5 mm in diameter. No gallstones are seen. What is the most
appropriate further imaging investigation? [B4 Q42]

a. CT of the abdomen
b. endoscopic ultrasound scan
c. MRCP
d. ERCP
e. no imaging indicated as normal findings

A

MRCP

Intrahepatic bile ducts are considered dilated when they exceed 40% of the diameter of the
adjacent portal veins. The upper limit of normal for the extrahepatic common bile duct is 5mm
in adults (increasing after age 60 by approximately 1mm/decade). Appearances here are
indicative of biliary obstruction at the level of the hilum, and MRCP is the investigation of
choice after ultrasound scan in these circumstances. MRCP reliably demonstrates the extent of
ductal involvement, allowing planning of surgery or treatment; with malignant causes, it may
provide further staging information. CT is the investigation of choice when ultrasound scan
indicates obstruction below the hilum. If ultrasound scan demonstrates ductal stones, ERCP is
the investigation of choice for confirmation and therapeutic intervention. Endoscopic
ultrasound scan is particularly useful for detecting small ductal stones and periampullary
tumours. Percutaneous transhepatic cholangiography is reserved for cases when ERCP is not
possible.

195
Q

A 22-year-old woman with known medullary sponge kidney presents with recurrent upper
abdominal pain and jaundice. Cholangiography demonstrates segmental saccular dilatation of
the intrahepatic bile ducts and ectasia of the extrahepatic ducts. What is the most likely
diagnosis? [B4 Q98]

a. choledochocele
b. choledochal cyst
c. primary sclerosing cholangitis
d. Caroli’s disease
e. polycystic liver disease

A

Caroli’s disease
Caroli’s disease is a rare congenital disorder characterized by multifocal segmental saccular
dilatation of the intrahepatic bile ducts. It presents in childhood and early adulthood with upper
abdominal pain, fever and transient jaundice. Up to 80% of patients have associated medullary
sponge kidney. Typical CT findings are of multiple cystic structures with a central enhancing
‘dot’, representing the portal vein radicles surrounded by dilated ducts. Cholangiography is
diagnostic and demonstrates saccular dilatation of the intrahepatic ducts of up to 5cm in
diameter, with frequent associated ectasia of the extrahepatic ducts. Choledochal cysts
primarily cause cystic dilatation of the common bile duct, but there may be associated
intrahepatic biliary dilatation. A choledochocele is a cystic dilatation of the intraduodenal
portion of the common bile duct. Primary sclerosing cholangitis classically causes multifocal
strictures of the intrahepatic and extrahepatic ducts, alternating with segments of dilatation. In
polycystic liver disease, there is no communication of the cysts with the biliary tree.

196
Q

A patient has an ultrasound scan carried out on a radiographer’s ultrasound list. The
radiographer notices an unusual finding and asks you to check the images. The liver, kidneys,
and spleen appear unremarkable. There are gallstones in the gallbladder, but also in the fundus
of the gallbladder, and there is a reverberation artefact that gives a comet tail appearance. This
finding is pathognomonic of a condition. Which of the following statements is true regarding
this condition? [B1 Q57]

A. Adenomyomatosis is caused by abnormal deposits of cholesterol esters in foam cells in the
lamina propria.
B. Cholesterolosis is caused by the rupture of Rokitansky–Aschoff sinuses with subsequent
intramural leak of bile causing an inflammatory reaction.
C. Xanthogranulomatous cholecystitis is characterized by an increase in the number and height
of glandular elements in the gallbladder.
D. Xanthogranulomatous cholecystitis is associated with gallbladder carcinoma in around 10%
of cases.
E. Adenomyomatosis is associated with cholesterolosis in up to a third of patients.

A

Adenomyomatosis is associated with cholesterolosis in up to a third of patients

Option A describes the cause of cholesterolosis. Option B describes the features of
xanthogranulomatous cholecystitis (XGC). Option C describes the features of
adenomyomatosis. Option D is true, but the ultrasound features described are not those of XGC,
so this is not true regarding the condition described in the clinical scenario.

Adenomyomatosis and cholesterolosis are both classed as types of hyperplastic cholecystosis.
Adenomyomatosis has two pathognomonic descriptions. Firstly, the ‘pearl necklace’
appearance on oral cholecystogram (OCG) (the same appearance can be seen on MRCP).
Secondly, comet tail artefact seen on ultrasound, caused by reverberation artefact between
cholesterol crystals in Rokitansky–Ashcoff sinuses.

Gall bladder adenomyomatosis [Radiopaedia]

  • Hyperplastic cholecystoses
  • Hyperplasia of the wall with the formation of Rokitansky-Ashoff sinuses (intramural
    diverticula lined by mucosal epithelium) penetrating into the muscular wall
  • Cholesterol crystal precipitate in the bile trapped in Rokitansky-Ashoff sinuses.

Radiology
* Ultrasound
o Mural thickening – diffuse, focal, annular [Annular form mimics gallbladder
carcinoma]
o Comet-tail artifact
* CT
o Non-specific wall thickening and enhancement.
o Rosary sign – enhancing epithelium within intramural diverticula, surrounded by
the relatively unenhanced hypertrophied gall bladder muscularis.
* MRI/ MRCP (Oral Cholecystogram) o Mural thickening
o Focal sessile mass
o Hourglass configuration in annular type
o Pearl Necklace Sign – Curvilinear arrangement of multiple rounded T2 bright
intramural cavities.

197
Q

A 41-year-old woman has an outpatient ultrasound scan for intermittent right upper quadrant
pain. Five 5mm gallstones and sludge are present. In addition, there is wall thickening of the
gallbladder fundus with multiple foci of increased echogenicity within the wall, each associated
with bright artefacts deep to them. Which one of the following is the most likely diagnosis?
[B2 Q22]

a. Porcelain gallbladder
b. Emphysematous cholecystitis
c. Acute cholecystitis
d. Adenomyomatosis of the gallbladder
e. Gallbladder carcinoma

A

Adenomyomatosis of the gallbladder

The correct diagnosis is adenomyomatosis. This is an uncommon condition, more common in
females, and is associated with gallstones in most cases. It is characterised by generalised or
focal mural thickening with intramural diverticula (Rokitansky–Aschoff sinuses). The
ultrasound artefact from cholesterol crystals in the sinuses produces bright ‘comet-tail’
reverberation artefacts.

198
Q

A40-year-old woman undergoes abdominal ultrasound scan, which demonstrates three small,
rounded, echogenic structures in relation to the anterior wall of the gallbladder. There is no
posterior acoustic shadowing, and appearances remain constant with variation in patient
position. The remainder of the gallbladder and biliary tree appear unremarkable. What is the
most likely diagnosis? [B4 Q47]

a. gallstones
b. cholesterol polyps
c. adenomyomatosis
d. gallbladder carcinoma
e. strawberry gallbladder

A

Cholestrol polyps

Cholesterolosis is a form of hyperplastic cholecystosis in which triglycerides, cholesterol
precursors and cholesterol esters accumulate within the lamina propria of the gallbladder wall.
Most cases are of the planar type, termed ‘strawberry gallbladder’ after the resemblance of the
gallbladder mucosa to the surface of a strawberry, and produce no detectable ultrasound
changes. In a minority of cases, cholesterol polyps are formed, which are the commonest type
of gallbladder polyp. They are generally small, multiple echogenic lesions adjacent to the
gallbladder wall. Non-mobility and a lack of posterior acoustic shadowing helps to distinguish
polyps from gallstones. Small size and multiplicity distinguish them from gallbladder
malignancy, though, rarely, metastatic disease may produce multiple polypoid lesions,
particularly malignant melanoma. Adenomyomatosis, the other form of hyperplastic cholecystosis, results in mucosal hyperplasia and thickening of the muscular layer of the
gallbladder. It is characterized by bright reflections and comet-tail artefacts from the
gallbladder wall on ultrasound scan.

199
Q

A 45-year-old male presents with a history of jaundice and RUQ pain. An ultrasound of the
abdomen demonstrates an impacted calculus in the gallbladder neck with dilatation of the
intrahepatic ducts. An MRCP is requested to exclude Mirizzi syndrome. What additional
features on MRCP confirm the diagnosis of Mirizzi syndrome? [B1 Q26]

A. Dilated common hepatic duct.
B. Dilated common hepatic and common bile ducts.
C. Dilated common hepatic duct with normal common bile duct.
D. Double duct sign.
E. Normal ducts.

A

Dilated common hepatic duct with normal common bile duct

Mirizzi syndrome is a functional hepatic syndrome caused by extrinsic compression of the
CHD by a calculus impacted in the gallbladder neck or cystic duct. Low insertion of the cystic
duct into the CHD is a predisposing factor.

Typical features at imaging include extrinsic compression of the CHD, a gallstone in the
gallbladder neck or cystic duct, dilatation of the intrahepatic ducts and CHD proximally, and a
normal CBD. Rarely, inflammation around an impacted calculus leads to a stricture formation
mimicking a periductal infiltrating cholangiocarcinoma.

200
Q

A 60-year-old diabetic male presents with a history of fever and right upper quadrant pain.
Ultrasound of the abdomen demonstrates curvilinear high-amplitude echoes in the gallbladder
wall with reverberation artefact and multiple high-amplitude echoes in the gallbladder lumen.
What is the diagnosis? [B1 Q27]

A. Acute cholecystitis.
B. Emphysematous cholecystitis.
C. Adenomyomatosis.
D. Chronic cholecystitis.
E. Cholesterosis.

A

Emphysematous cholecystitis

This is a rare form of acute cholecystitis. Most patients are between 50 and 70 years of age. It
is more common in men (male to female ratio of 2:1) and in those with diabetes and peripheral
vascular disease. Emphysematous cholecystitis is a surgical emergency because there is an
increased risk of gallbladder perforation and increased mortality rate. The definitive treatment
is cholecystectomy, although in critically ill patients percutaneous cholecystostomy may be
used as a temporary measure.

201
Q

In a 67-year-old female patient with jaundice and gallbladder wall thickening on ultrasound
scan, which feature on CT favours a diagnosis of xanthogranulomatous cholecystitis rather
than gallbladder carcinoma? [B4 Q60]
a. pericholecystic fat infiltration
b. intramural hypoattenuating nodules throughout the gallbladder
c. biliary obstruction
d. hepatic extension
e. regional lymphadenopathy

A

Intramural hypoattenuating nodules throughout the gallbladder

Xanthogranulomatous cholecystitis (XGC) is an uncommon inflammatory disease of the
gallbladder, which is characterized by multiple intramural nodules and proliferative fibrosis. It
is thought to result from rupture and extravasation of bile and mucus following the occlusion
of Rokitansky–Aschoff sinuses. There is considerable overlap between the clinical and
radiological features of XGC and gallbladder carcinoma. Pericholecystic infiltration, biliary
obstruction, regional lymphadenopathy and hepatic involvement may be seen in both
conditions, and the difference in incidence between the two conditions is not statistically
significant. Only the presence of multiple intramural hypoattenuating nodules (representing
xanthogranulomas) occupying a large area of the thickened gallbladder wall allows a diagnosis
of XGC to be made with any degree of certainty. Similar hypoattenuating intramural nodules
(representing haemorrhage and necrosis) may be seen less commonly in gallbladder carcinoma,
but these tend to occupy a much smaller proportion of the thickened gallbladder wall

202
Q

A 39-year-old woman has an ultrasound scan for right upper quadrant pain and jaundice which
reveals biliary ductal dilatation to the level of the common hepatic duct adjacent to a stone in
the gallbladder neck. The gallbladder is thick-walled and tender. MRCP confirms these
findings and excludes common duct stones. Which one of the following is the most likely
diagnosis? [B2 Q45]

a. Primary sclerosing cholangitis
b. Mirizzi syndrome
c. Caroli’s disease
d. Fascioliasis
e. Acute cholecystitis

A

Mirizzi syndrome

Mirizzi syndrome is narrowing of the common hepatic duct caused by a gallstone impacted in
the neck of the gallbladder or the cystic duct. The stricture is smooth and often concave to the
right as seen on ERCP. Fistulae can develop between the gallbladder and the common duct,
and the stone may pass into the common duct. It is associated with acute cholecystitis.
Fascioliasis is caused by liver fluke infestation which may cause bile duct wall thickening and
multiple hepatic abscesses. Caroli’s disease is a congenital disorder characterised by cystic
dilatation of the intrahepatic bile ducts.

203
Q

Which of the following is not a feature of Mirrizzi sign? [B3 Q20]

A. Jaundice
B. Gall stone impacted in gall bladder neck
C. Dilatation of bile ducts above level of cystic duct
D. Smooth curved segmental stenosis of Common Hepatic Duct (CHD)
E. Dilated Common Bile Duct (CBD) below level of impacted stone

A

E In Mirrizzi’s sign, the CBD below the level of the impacted gallstone is not dilated.

204
Q

A patient is admitted with right upper quadrant (RUQ) pain to the surgical team and is referred
for ultrasound. On the ultrasound there is a curvilinear echogenic line at the margin of the
gallbladder and posterior acoustic shadowing in the gallbladder fossa. There is no evidence of
peristalsis, and the shadowing does not change on patient positioning. The sonographer states
that the patient’s pain has settled, and they are otherwise well. What is the most likely cause of
this appearance?

A. Bowel in the gallbladder fossa.
B. Porcelain gallbladder.
C. Gallstones.
D. Emphysematous cholecystitis.
E. Post-ERCP.

A

Porcelain gallbladder

It can sometimes be difficult to see a cause for this appearance, and several of the given options
could result in it. However, peristalsis should be seen in a healthy patient if the abnormality is
due to bowel. With gallstones, the appearance should change on positioning. There is no
mention of an ERCP in the history, and whilst air in the biliary tree is common after
sphincterotomy, it is often only seen in the gallbladder immediately after the procedure.
Patients with emphysematous cholecystitis are usually clinically unwell and are unlikely to be
asymptomatic. Porcelain gallbladder is associated with gallstones in 90% of cases. It is a
relevant finding to make as 10–20% of patients develop carcinoma of the gallbladder.

205
Q

A 65-year-old man presents with weight loss and obstructive jaundice. An ultrasound reveals
dilatation of the intra- and extrahepatic biliary system. MRCP reveals a stricture in the distal
common bile duct (CBD). The patient becomes septic and biliary drainage is required. Which
is the most appropriate method for this? [B1 Q10]

A. Percutaneous transhepatic cholangiography (PTC) and external drainage.
B. PTC with internal/external drainage.
C. ERCP with plastic stent insertion.
D. ERCP with metal stent insertion.
E. PTC/ERCP rendezvous procedure.

A

ERCP with plastic stent insertion

PTC is an appropriate approach, but biliary sepsis can cause bacteraemia during PTC and thus
ERCP is preferable in this case, if possible. Contraindications to PTC include prothrombin time
greater than 2 seconds higher than control, platelet count less than 100,000, ascites, hydatid
disease, and lack of access to surgical facilities. If an ERCP were to fail, the other options
would be viable alternatives. Metal stent insertion at the first instance is inappropriate unless it
is known that the biliary dilatation is due to inoperable malignancy. This is because metal stents
cannot be removed, whereas the plastic variety can be removed, if necessary

Bild Duct Infection [Core Radiology]

Ascending Cholangitis:
Symptoms: Pain, Fever, Jaundice (Charcot’s Triad)
Imaging: hyperenhancement and thickening of the walls of bild ducts
Treatment: Endoscopic biliary intervention if not respond to conservative treatment.

Ascending Cholangitis [STATdx]

  • Def: – Pyogenic infection of biliary tree as result of biliary obstruction
  • Causes
    o Stones
    o Biliary strictures (Benign or malignant)
    o Recent intervention or hepato-biliary surgery
  • Symptoms – Pain, Fever, Jaundice (Charcot’s triad)
  • Treatment – broad spectrum antibiotics and drainage (usually ERCP)
  • CT and MR findings:
    o Biliary Trees
    ▪ Dilatation of intra and/or extrahepatic ducts with abrupt transition
    ▪ Biliary dilatation may be central, diffuse, or segmental
    o Bile duct wall – thickening with hyperenhancement
    o Purulent Bile or Pus – high density of CT, intermediate to low on T1, T2.
    o Liver ▪ Heterogeneous liver enhancement (wedge-shaped, peribiliary, patchy, or
    diffuse)
    ▪ Associated liver abscesses (1/4 of cases)
    ▪ Portal vein thrombosis
  • Ultrasound findings
    o Dilatation, stenosis, and thickening of bile duct walls with intraluminal echogenic
    debris
  • Top Differential Diagnoses
    o Primary sclerosing cholangitis
    o Recurrent pyogenic, AIDS-related, or chemotherapy-related cholangitis
206
Q

A 36-year-old man with ulcerative colitis develops progressive jaundice and pruritis. CT of the
abdomen demonstrates multiple areas of dilatation and stenosis of tortuous intrahepatic bile
ducts, with wall thickening and contrast enhancement of the extrahepatic bile ducts. What is
the most likely diagnosis? [B4 Q49]

a. primary sclerosing cholangitis
b. choledocholithiasis
c. primary biliary cirrhosis
d. ascending cholangitis
e. chronic pancreatitis

A

Primary sclerosing cholangitis

Primary sclerosing cholangitis is an idiopathic, progressive, fibrosing, inflammatory disorder
of the biliary tree, causing multifocal strictures, cholestasis, and biliary cirrhosis. There is an
association with inflammatory bowel disease and autoimmune conditions. In most cases, both
intra- and extrahepatic ducts are involved, and classic appearances on cholangiography are of
a ‘string-of-beads’ appearance with alternating segments of dilatation and stenosis. Biliary
cirrhosis develops in up to 49% of cases, and there is an increased risk of cholangiocarcinoma.
In primary biliary cirrhosis, disease is limited to the intrahepatic bile ducts. In ascending
cholangitis, there may be biliary dilatation and pneumobilia, but multifocal strictures are not a
feature. CT features of choledocholithiasis include biliary dilatation and visualization of a stone
in the bile duct, but again strictures are not a feature. Chronic pancreatitis may result in a
smooth inflammatory stricture of the intrapancreatic portion of the common bile duct.

207
Q

A 32-year-old man has an ultrasound scan for obstructive jaundice. Areas of intrahepatic duct
dilatation are seen, with increased echogenicity of the portal triads. ERCP reveals alternating
segments of dilatation and stenosis of both the intra- and extrahepatic ducts. Which one of the
following diagnoses is most likely? [B2 Q23]

a. Primary sclerosing cholangitis
b. Primary biliary sclerosis
c. Ascending cholangitis
d. Choledochal cyst
e. Congenital hepatic fibrosis

A

Primary sclerosing cholangitis
These ultrasounds and ERCP features are typical of primary sclerosing cholangitis, which is
an idiopathic condition characterised by progressive fibrosis of the biliary tree. It primarily
affects young men with inflammatory bowel disease (more common in ulcerative colitis than
Crohn’s) although pancreatitis, liver cirrhosis and chronic active hepatitis are other associated
conditions. Primary biliary cirrhosis may also cause scattered areas of focal intrahepatic duct
dilatation, but this condition is much more common in females and the extrahepatic ducts are
not involved.

Primary Sclerosing Cholangitis [STATDx]
Def: - Immune-mediated disease-causing progressive inflammation, fibrosis, and stricturing of
intrahepatic and extrahepatic ducts

Imaging:
* Multifocal “beaded” strictures of intra- and extrahepatic ducts with intervening sites of
dilated and normal ducts
* Pruned appearance of biliary tree develops over time

  • Active inflammation → Thickening and hyperenhancement of bile duct wall suggests
    active inflammation
  • Peripheral intrahepatic ductal strictures → Visualization of greater than expected
    number of peripheral ducts on MRCP
  • Pseudo-tumour of Caudate → Chronic involvement results in atrophy of peripheral
    liver, massive hypertrophy of central liver/caudate and lobulated, rounded liver contour
  • Frequent periductal and perivascular fibrosis and confluent fibrosis in central liver →
    Low density on CT and T2 hypointense on MR
  • Oedema and inflammation → Periphery patchy T2 hyperintensity on MR
  • Hepatolithiasis, cholelithiasis, and choledocholithiasis are common → signal voids on
    MR
  • Extensive lymphadenopathy common
208
Q

A 72-year-old male patient presents to the surgical team with a 3-week history of increasing
painless jaundice. He has a past medical history of gallstones, prostatic carcinoma, and
ischaemic heart disease. There is no history of alcohol abuse. The LFTs are abnormal. Serum
bilirubin is 346. He is referred for an ultrasound scan of the abdomen, which identifies grossly
dilated intrahepatic bile ducts, but no evidence of a dilated CBD. The common hepatic duct
(CHD) is not clearly visible due to an isoechoic mass in the region of the porta hepatis at the
ductal confluence. A triple phase CT scan of the liver is carried out. The lesion is iso- to
hypoattenuating. There is limited arterial enhancement, with some portal venous enhancement
peripherally. On delayed images the lesion displays enhancement with mild peripheral washout.
What is the most likely pathology? [B1 Q18]

A. Cholangiocarcinoma.
B. Portal metastasis.
C. Hepatocellular carcinoma.
D. Benign biliary stricture.
E. Cavernous haemangioma.

A

Cholangiocarcinoma.

Specifically a Klatskin tumour, as it occurs at the porta hepatis. Cholangiocarcinoma can be
iso- to hyperechoic on ultrasound. On CT and MRI, it shows delayed enhancement in 74%.
Many conditions predispose to cholangiocarcinoma and gallstones are identified in 20–50% of
patients with cholangiocarcinoma. Hepatocellular carcinoma would be the next most likely
diagnosis. It can have a variable ultrasonographic appearance. Hepatocellular carcinoma
usually demonstrates arterial phase enhancement (80%). Prostate does not commonly
metastasize to the liver and would again demonstrate arterial phase enhancement classically.
Whilst haemangiomas are classically hyperechoic on ultrasound, larger lesions can appear
heterogeneously hypoechoic (40%)

Cholangiocarcinoma [Core Radiology]

  • Def: a highly malignant tumour of the biliary ductal epithelium.
  • Site:
    o Klastin tumour: A hilar tumour (at the confluence of the right and left intrahepatic
    biliary ducts) is the most common tumour
    o Peripheral cholangiocarcinoma is rare.
  • Imaging
    o Intrahepatic ductal dilation and lobar atrophy, capsular retraction, tumour extend
    into the bile ducts.
  • Risk factors
    o Choledochal cyst(s).
    o Primary sclerosing cholangitis.
    o Familial adenomatous polyposis syndrome. o Clonorchis sinensis infection.
    o Thorium dioxide (alpha-emitter contrast agent), not used since the 1950s. Thorium
    dioxide is also associated with angiosarcoma and HCC.

Cholangiocarcinoma [STATdx]

Ultrasonographic Findings: depend on anatomic site and local tumour extension

  • Intrahepatic

o Mass with ill-defined margin, mixed and heterogeneous echotexture
o Isolated thickening of IHBD or intraductal mass with upstream ductal dilatation
o Capsular retraction

  • Hilar CCA (Klatskin tumour)

o Dilatation of intrahepatic ducts without extrahepatic ductal dilatation
o Apparent non-union of dilated right and left IHBDs
o Primary tumour may not be discernible or appears as small, infiltrative iso-
/hyperechoic mass in hilar region
o Nodular or polypoid mass in bile ducts
o Mass effect, invasion of portal vein and hepatic artery

  • Extrahepatic

o Dilatation of intrahepatic and proximal extrahepatic bile duct
o Primary tumour often undetectable due to its deep location
o Ill-defined, solid, heterogeneous mass within or surrounding duct at point of
obstruction
o Exophytic or intraluminal
o May infiltrate into or cause mass effect on adjacent stomach and duodenum

  • Other signs of malignancy
    o Infiltration of liver parenchyma
    o Lymphatic spread; porta hepatis, para-celiac and peripancreatic lymph nodes
    o Liver metastases
  • Important negative findings
    o Absence of obstructing biliary stones
    o No obstructing pancreatic head mass or pancreatic ductal dilatation
  • Contrast-enhanced ultrasound
    o ICC has variable arterial-phase enhancement (hyper/iso/hypo/rim)
    o Rim enhancement is most specific type of arterial-phase enhancement
    o ICC shows rapid (< 1 minute) and marked washout in portal venous/late phase
    o Can differentiate enhancing tumour from avascular sludge/debris in bile ducts
209
Q

A 58-year-old male with unexplained elevated alkaline phosphatase has an MRCP and the
‘double-duct’ sign is observed. Which one of the following diagnoses is most likely to cause
this finding? [B2 Q42]
a. Acute pancreatitis
b. Annular pancreas
c. Pancreas divisum
d. Periampullary tumour
e. Duodenal perforation

A

Periampullary tumour
The ‘double-duct’ sign is dilatation of the main pancreatic duct and the common bile duct as
seen at ERCP and MRCP, and less commonly with CT and ultrasound. It occurs due to an
obstructing lesion at the ampulla, most commonly a carcinoma of the head of the pancreas (in
up to 77% of cases) or a carcinoma of the ampulla of Vater (in up to 52% of cases). The sign
may be absent if there is an accessory pancreatic duct or when the main pancreatic duct drains
into the minor papilla.

210
Q

A 60-year-old male has an abdominal ultrasound for the investigation of deranged LFTs. A 2
cm hyperechoic mass is seen at the porta hepatis. There is dilatation of the right and left hepatic
ducts, but the common bile duct is of normal calibre. A PET-CT is performed which shows an
FDG-avid lesion corresponding to the abnormality on ultrasound and no other findings. Which
of the following is the most likely diagnosis? [B2 Q60]

a. Caroli’s disease
b. Klatskin tumour
c. Periampullary tumour
d. Primary sclerosing cholangitis
e. Biliary cystadenoma

A

Klatskin tumour
Klatskin tumours are the most common form of cholangiocarcinoma, representing tumour at
the confluence of the hepatic ducts. The finding of a hyperechoic central porta hepatis mass at
ultrasound is typical. Risk factors include inflammatory bowel disease, primary sclerosing
cholangitis, Caroli’s disease and cholecystolithiasis. Cholangiocarcinomas have a very poor
prognosis with a five-year survival of less than 2%. They are FDG-avid and PET-CT is
typically performed in the pre-operative evaluation of these tumours.

211
Q

Which of the following is most likely to deem a cholangiocarcinoma irresectable? [B3 Q18]

A. Right portal vein invasion
B. Left hepatic artery invasion
C. PV branch to one lobe and hepatic artery to the other lobe invaded
D. Size > 2cm
E. Biliary duct dilatation

A

C

Irresectable tumours include those with main PV or main HA invasion, those with both right
and left PV or both right and left HA involvement, or involvement of a PV branch to one lobe
and a HA branch to other lobe.

212
Q

For relief of malignant biliary obstruction by percutaneous stenting, which factor is an
advantage of using a metallic rather than a plastic stent? [B4 Q93]

a. higher long-term patency rates
b. lower cost
c. easily removed if infection develops
d. does not shorten following deployment
e. higher surface area

A

Higher long-term patency rates

Percutaneous or endoscopic biliary stenting is usually performed to relieve jaundice in patients
with malignant biliary obstruction. Metallic or plastic stents may be used. Metallic stents have
a larger internal diameter and have higher long-term patency rates than plastic stents, as well
as a low surface area that reduces bacterial colonization and subsequent fibrous deposition.
However, metallic stents become incorporated into the bile duct mucosa and are not easily
removed, which may present problems if the stent becomes infected. They are also prone to
shortening following deployment, and a long-segment obstruction may therefore require
multiple stents. In addition, the cost of metallic stents is approximately 10 times that of plastic
stents.

213
Q

A patient is being worked up for a pancreatic neoplasm to assess potential resectability. Which
one of the following does not rule out surgery? [B1 Q15]

A. Extension of the tumour beyond the margins of the pancreas into duodenum.
B. Tumour involvement of adjacent organs.
C. Enlarged peripancreatic lymph nodes (>15 mm).
D. Encasement or obstruction of superior mesenteric vessels.
E. Peritoneal carcinomatosis

A

Enlarged peripancreatic lymph nodes (>15mm)
Enlarged regional nodes are a sign of unresectability, but nodes adjacent to the pancreas are
resected as part of Whipple’s procedure. The other factors all indicate that a pancreatic lesion
is unresectable. Other features of unresectable pancreatic carcinoma are liver metastases. Only
10–15% of pancreatic neoplasms are resectable at presentation.

Solid Pancreatic Epithelial Neoplasm [Core Radiology]

  1. Ductal Adenocarcinoma – majority 80-90%
    a. Commonest in head
    b. CT protocol (pre, late arterial PV)
    c. Hypodense, T1 dark mass
    d. Ductal obstruction is almost always – *consider alternative diagnosis if normal duct
    e. Distal pancreatic atrophy
    f. Mass can be occult i.e., iso-attenuating – suspect if duct is dilated.
    g. Features of unresectable tumours – encasement of SMA (>180
    ), extensive venous
    invasion, metastasis.
  2. Acinar cell Carcinoma
    a. Rare, aggressive, almost exclusively in elderly
    b. Produces large amount of lipase – Triad of lipase hypersecretion syndrome
    c. Subcutaneous fat necrosis, bone infarcts (arthralgia), eosinophilia.

Pancreatic Adenocarcinoma [STATdx]

  • Imaging:
    o Endoscopic USG – best for head of pancreas tumour, also for regional lymph nodes.
    o CT – local resectability
    o MRI – for subtle liver metastasis
    o PET – for distance disease and response to therapy
  • Syndromes associated with
    o Peutz-Jeghers syndrome
    o Familial malignant malenoma
    o Lynch syndrome
    o Breast ovarian cancer syndrome o Le-Fraumeni syndrome
  • Staging
    o T1 – <2cm; T2 – 2-4 cm; T3 – >4cm or extend beyond pancreas; T4 vascular
    invasion
    o N1 – 1-3 regional lymph nodes; N2 – > 3 regional lymph nodes
    o M1 – distant metastasis
  • Treatment
    o Whipple for resectable tumour (can be done without prior histology)
    o Neo-adjuvant radio/chemo for downstaging before surgery
    o Possible SMV reconstruction

Standard Whipple Procedure

  • Removal of
    o Pancreatic head, neck and uncinate process
    o Duodenum, pylorus, and gastric antrum
    o Gall bladder and distal CBD
    o Regional lymph nodes
  • Anastomoses of
    o Pancreatico-jejunostomy (end to end) [hand sewn – hard to see on CT]
    o Hepatico/choledocho-jejunostomy (end to side) [hand sewn]
    o Gastro-jejunostomy (end to side, with or without Roux loop) [stappled]
214
Q

A 56-year-old woman is diagnosed with pancreatic adenocarcinoma. Which one of the
following features on the pancreatic MR contraindicates curative surgery? [B2 Q11]

a. Splenic vein invasion
b. Tumour size of 2cm
c. Portal vein invasion
d. Hepatic artery invasion
e. Invasion of the second part of the duodenum

A

Hepatic artery invasion

The only widely recognised absolute contraindication to curative surgical resection of the
options listed is invasion of the hepatic artery. Invasion of the splenic and portal veins are
relative contraindications if the veins are not completely occluded. Invasion of the second part
of the duodenum is not a contraindication as it is resected at surgery. Other features that make
the tumour unsuitable for curative resection are distant metastases, ascites, distant organ
invasion, SMA/coeliac/aortic invasion and involved lymph nodes outside the boundaries of the
resection.

215
Q

A 58-year-old man with a history of alcohol abuse and diabetes presents with painless jaundice.
Liver function tests reveal an obstructive picture, and he undergoes an ultrasound of abdomen,
which reveals dilatation of the CBD and a hypoechoic region in the head of the pancreas. He
has a history of iodine allergy and undergoes MRI with dynamic gadolinium enhancement, as
an alternative to contrast-enhanced CT. Which finding in the pancreatic head is most in keeping
with the diagnosis of pancreatic adenocarcinoma? [B1 Q48]

A. Hypo-intensity on T1WI. B. Hyper-intensity on T2WI.
C. Hyper-intensity on a STIR sequence.
D. Hypo-intensity during arterial phase enhancement.
E. Hypo-intensity during portal venous phase enhancement

A

Hypo-intensity during arterial phase enhancement

Pancreatic adenocarcinoma is generally a hypo-vascular tumour at CT as well as MRI.
Dynamic contrast-enhanced CT has been the gold standard for the diagnosis of pancreatic
adenocarcinoma, but MRI is of value in those with renal failure or sensitivity to iodine-based
contrast media. Care should be taken in patients with a very low GFR (typically less than 30ml/
min) because of the risk of nephrogenic systemic fibrosis.

Contrast-enhanced MRI may have a lower false negative rate than CT, as approximately 10%
of pancreatic adenocarcinomas have been shown to be iso- rather than hypoattenuating, on both
the pancreatic and portal venous phases.

Chandarana et al. have reported that 25 of 25 neoplasms showed hypo-intensity during arterial
phase enhancement (and 20 remained hypointense in venous phase), whereas only 12 of 25
were hypointense on unenhanced T1WI, and only 11 of 25 were hyperintense on STIR/T2WI.

Imaging for pancreatic adenocarcinoma

  • Gold standard – CECT
    o Hypodense, however 10% of them is iso-dense on both pancreatic and PV phase.
  • Contrast-enhanced MRI – lower false negative than CT (order of decreasing sensitivity)
    o Arterial phase hypo-intense
    o Hypo-intense on T1
    o Hyper-intense on T2.
216
Q

A 58-year-old male has a CT staging scan following a diagnosis of adenocarcinoma of the body
of the pancreas. The tumour is 3cm in size and extends beyond the boundaries of the pancreas
but does not invade any vessels or adjacent organs. Two 1cm lymph nodes lie adjacent to the
tumour. No other nodes, or metastatic disease in the chest, abdomen, or pelvis, are identified.
The tumour is best staged as which one of the following? [B2 Q21]

a. T1N0M0
b. T1N1M0
c. T2N0M0
d. T3N0M0
e. T3N1M0

A

T3N1M0

T1 tumour is disease confined to the pancreas and less than 2 cm in diameter. T2 tumour is
also confined to the pancreas but greater than 2 cm in diameter. As the tumour extends beyond
the boundary of the pancreas, it is at least T3. Invasion of the coeliac or superior mesenteric
arteries would make this a T4 tumour, but as these features are not present it is T3. The presence of regional nodes makes it N1 rather than N0 (no nodes involved), and there is no metastatic
disease, so it is M0. Therefore, the correct radiological stage is T3N1M0.

217
Q

A 68-year-old female has a pancreatic MR for characterisation of an isolated lesion within the
pancreas seen initially on CT performed for unexplained weight loss. The lesion is 3cm in
diameter, isointense on T1, isointense on T2 STIR and hypointense to pancreatic parenchyma
during the arterial phase of gadolinium enhancement. It remains hypointense on the venous
and delayed phases of contrast enhancement. Which one of the following is the most likely
diagnosis? [B2 Q38]

a. Ductal adenocarcinoma
b. Insulinoma
c. Simple pancreatic cyst
d. Gastrinoma
e. Glucagonoma

A

Ductal adenocarcinoma

Insulinoma tends to be hyperintense on contrast-enhanced images. Gastrinoma is usually
hyperintense on STIR imaging and on contrast-enhanced sequences.

In a series of 25 patients, an article by Chandarana et al. showed that pancreatic
adenocarcinomas were either iso- or hypointense on T1-weighted imaging and iso- or
hyperintense on T2 or STIR. All adenocarcinomas were hypointense to pancreatic parenchyma
during the arterial phase of gadolinium enhancement on MR, 80% remained hypointense in the
venous phase of enhancement and 68% remained hypointense in the delayed phase.

218
Q

A 67-year-old man undergoes Whipple’s procedure for adenocarcinoma of the head of the
pancreas. Which finding is of most concern on CT of the abdomen performed 4 days
postoperatively for persistent pyrexia? [B4 Q78]

a. free intraperitoneal gas
b. aero-bilia in the left intrahepatic ducts
c. oral contrast within the afferent jejunal loop
d. small thin-walled fluid collection in Morison’s pouch
e. gas-containing fluid collection in the pancreatic bed

A

Gas-containing fluid collection in the pancreatic bed
The two main indications for Whipple’s procedure (radical pancreaticoduodenectomy) are
tumour in the periampullary region, and chronic pancreatitis involving the head and uncinate
process of the pancreas. Surgery is complex and involves gastrojejunostomy, pancreatico-
enterostomy and choledocho-jejunostomy. Common postoperative findings include
retroperitoneal fat stranding and transient, thin-walled fluid collections, which may be in the
pancreatic bed, perianastomotic or in Morison’s pouch. Free gas is common and aero-bilia is
seen in around 70% of cases, more commonly in the left intrahepatic ducts. Filling of the
afferent jejunal loop with contrast is normal and occurs in up to 44% of patients. The
commonest complications are delayed gastric emptying, pancreatico-jejunal leak and sepsis.
Anastomotic failure can occur at any of the sites, but the pancreatico-enterostomy is most
important because of the risk of leakage of pancreatic secretions. Anastomotic leak is associated with increased free gas, perianastomotic fluid and ascites. Focal septic collections
can occur anywhere, and complex or gas-containing areas are considered suspicious. Other
early complications include vascular injury and thrombosis, Clostridium difficile colitis and
pancreatitis of the pancreatic remnant

219
Q

A 50-year-old female patient is referred for an outpatient CT after an ultrasound carried out to
look for gallstones revealed a cystic lesion within the pancreas. The CT shows several large
cysts of over 2 cm in diameter, containing fluid measuring 3 HU in the head and body of the
pancreas. These cysts have thin enhancing walls. The pancreatic duct is not significantly
distended. On further questioning the patient denies a history of previous pancreatitis. An MRI
does not extend the diagnostic process. An FNA reveals fluid low in amylase, but with high
carcino-embryonic antigen (CEA) content. What is the most likely diagnosis? [B1 Q24]

A. Pancreatic pseudocyst.
B. Mucinous cystic neoplasm.
C. Microcystic pancreatic tumour.
D. Intraductal papillary mucinous tumour (main duct type).
E. Lymphangioma.

A

Mucinous cystic neoplasm

Mucin is detected in FNA fluid in these lesions. Pseudocysts are uncommon without a history
of pancreatitis and the aspirated fluid is high in amylase. The cysts in microcystic lesions are
usually smaller than 1cm, except in the oligo-cystic variant. They are also known as serous
cystadenomas. Intraductal papillary mucinous tumours cause dilatation of the main pancreatic
duct, side branch ducts or both. FNA, either percutaneous or via endoscopic ultrasound, has
been described as a low-risk procedure for differentiating pancreatic cystic lesions.

220
Q

A 65-year-old woman undergoes CT of the abdomen. An incidental finding of a well-defined
5 cm mass in the head of the pancreas is noted. It has a mean attenuation value of 5 HU and
contains multiple tiny cysts with a central nidus of calcification. There is no pancreatic duct
dilatation. What is the most likely diagnosis? [B4 Q10]

a. mucinous cystadenoma
b. main duct intraductal papillary mucinous tumour
c. serous cystadenoma
d. pancreatic pseudocyst
e. pancreatic insulinoma

A

Serous cystadenoma
Serous cystadenomas are benign neoplasia of the pancreas most seen in older women and are
frequently asymptomatic. Typical appearances are of a cystic lesion measuring up to 20 cm in
size, containing innumerable small cysts, though these may be difficult to discriminate, giving
the appearance of a solid mass. They may occur in any part of the pancreas but are slightly
more common in the pancreatic head and neck. Characteristic features include a central stellate
scar containing dystrophic calcification. Mucinous cystadenomas usually occur in the
pancreatic tail (90%) and, when multilocular, contain larger cysts of >2 cm in diameter.
Pancreatic pseudocysts are usually unilocular and occur following a history of pancreatitis.
Intraductal papillary mucinous tumours of the main duct are typically associated with dilatation
of the main pancreatic duct. Insulinomas are usually small (,2 cm) solid tumours that produce
symptoms early due to recurrent hypoglycaemia.

Cystic Pancreatic Epithelial Neoplasm [Core Radiology]

  1. Serous Cystadenoma:
    * Grandma tumour (demographic)
    * Multiple, small cysts (<2cm)
    * Could appear solid on CT, needs MR to confirm cystic nature
    * *Hyper-vascular (unique)
    * *No duct dilatation or tail atrophy [in contrast to adenocarcinoma]
    * *Central stellate calcification
  2. Mucinous Cystic Neoplasm:
    * Mother tumour (demographic)
    * Usually benign but have malignant potential so usually resected
    * Has capsule
  3. Solid and Papillary Epithelial Neoplasm (SPEN)
    * Daughter tumour (demographic)
    * Low malignant potential but typically resected.
    * Large mass with heterogenous solid and cystic areas with haemorrhage.
    * Has capsule
  4. Intra-ductal Papillary Mucinous Neoplasm (IPMN)
    * Grandfather tumour (but a lot varied)
    * Wide spectrum of benign, malignant
    * Main duct or side branch IPMN (main duct type has higher malignant potential)
    * A cystic intrapancreatic lesion in contiguity with the duct or side branch.
    * Any nodular or enhancing component should raise concern for malignancy.
221
Q

A 72-year-old woman has a pancreatic MR to investigate recurrent episodes of pancreatitis.
There is generalised pancreatic atrophy with dilatation of the main pancreatic duct and branch
ducts, particularly in the tail. No focal lesion or intraductal calculi are present. Which one of
the following diagnoses is most likely? [B2 Q18]

a. Microcystic cystadenoma
b. Intraductal papillary mucinous tumour of the pancreas
c. Cystic metastases
d. Cystic islet cell tumour
e. Pancreatic lipomatosis

A

Intraductal papillary mucinous tumour of the pancreas

Intraductal papillary mucinous tumour (IPMT) of the pancreas is a rare tumour. It tends to
present in the elderly population and can be a cause of recurrent pancreatitis. Two recognised
types include main duct IPMT, in which the main pancreatic duct is dilated, and branch duct
IPMT, in which the main duct is usually uninvolved. It is a risk factor for mucinous carcinoma
of the pancreas
. Pancreatic atrophy is often present. Imaging characteristics are often like those
seen in chronic pancreatitis, although calcification is not a feature of IPMT.

222
Q

Which is the most typical finding of serous cystadenoma of the pancreas? [B3 Q15]

A. Multiple cysts < 2cm separated by a fibrous septa coalescing into a central scar
B. Numerous sub centimetre cysts forming a homogenous pattern
C. A few large cysts > 2cm with septae in the head of the pancreas
D. Multi-cystic tumour larger than 10cm
E. Hyper-vascular solid tumour

A

Multiple cysts < 2 cm separated by a fibrous septa coalescing into a central scar

In serous cystadenoma of the pancreas, the polycystic pattern with cysts < 2cm represents 70%
of cases. The honeycomb pattern occurs in 20% and macro/oligocytic type in 10%. Giant
subcutaneous serous cystic change with intraluminal haemorrhage and solid serous
cystadenoma are atypical manifestations.

223
Q

Regarding intraductal papillary mucinous tumours (IPMT): [B3 Q35]

A. Can range from hypoplasia to invasive carcinoma
B. Is more common in females than mucinous cystic tumours
C. Has a higher incidence of malignancy in branch than main ducts
D. Focal wall thickening indicates malignancy with very high sensitivity and specificity
E. Parietal nodules indicate malignancy with very high sensitivity and specificity

A

Can range from hypoplasia to invasive carcinoma
Parietal nodules and focal wall thickening are useful indicators in detecting malignant branch
IPMT but there is a high incidence of false positive errors. IPMT has a slightly higher incidence
in males.

224
Q

A 58-year-old man with recurrent episodes of upper abdominal pain undergoes MRCP, which
demonstrates pancreatic atrophy and marked dilatation of the main pancreatic duct, which
contains high T2-signal material. A low T2-signal nodular filling defect is also identified within
the dilated duct. ERCP demonstrates thick mucus protruding from a bulging papilla. What is
the most likely diagnosis? [B4 Q67]

a. intraductal papillary mucinous tumour
b. chronic pancreatitis
c. mucinous cystadenoma
d. pancreatic pseudocyst
e. acute pancreatitis

A

Intraductal papillary mucinous tumour

Intraductal papillary mucinous tumour (IPMT) of the pancreas is characterized by a mucin-
producing tumour with dilatation of the main or branch ducts of the pancreas due to copious
secretions. They may arise in the main duct or branch duct. Main duct tumours typically cause
dilatation of all or part of the duct, which is filled with mucinous secretions, appearing
hyperintense on T2W images. A T2-hypointense intraductal filling defect may be identified,
which may represent the tumour or concretions of mucin. Chronic pancreatitis may result in
parenchymal atrophy and duct dilatation, with intraductal filling defects due to calculi or debris,
but a bulging papilla at ERCP makes IPMT more likely. The branch duct type usually consists of conglomerated communicating cysts or a unilocular cyst in the uncinate process; these
appearances may be mimicked by mucinous cystadenoma and pancreatic pseudocyst.

225
Q

A patient with a history of alcohol abuse presents to A&E with epigastric pain. His
haemoglobin is 8 g/dl on admission. An oesophago-gastroduodenoscopy (OGD) and
ultrasound of abdomen are requested. The ultrasound of abdomen shows multiple hyperechoic
lesions in the liver. There is no evidence of gallstones. Prior to the OGD the patient becomes
acutely unwell, and a CT scan is requested. This shows evidence of air and fluid in the
subhepatic space. It also reveals a focal enhancing lesion causing prominence of the head of
the pancreas. What is the most likely diagnosis? [B1 Q3]

A. Pancreatitis with associated peripancreatic abscess.
B. Pancreatic carcinoma and liver metastases.
C. Pancreatitis and peptic ulcer perforation in an alcoholic patient.
D. Islet cell tumour with liver metastases.
E. Cholangiocarcinoma with liver metastases.

A

Islet cell tumour with liver metastases

Whilst this tumour is rare, the CT findings indicate a focal mass lesion in the head of the
pancreas. Hyperechoic metastases in the liver are suggestive of islet cell tumour, rather than
pancreatic carcinoma metastases, even though pancreatic carcinoma is more common. The islet
cell tumour could be a gastrinoma, which is most found in the head of the pancreas and is
malignant in 60%. It is also associated with peptic ulcer disease and the finding of air and fluid
in the subhepatic space suggests a perforated duodenal ulcer.

Causes of hyper-echoic liver lesion

Benign

  • Hepatic haemangioma
  • FNH
  • Hepatic adenoma with high fat content
  • Focal fatty change
  • Hepatic AML
  • Inflammatory pseudotumor of the liver
  • Lipoma

Malignant

  • Hepatic mets
    o Colorectal
    o Treated breast
    o PNET
    o RCC
    o Thyroid
    o Melanoma
    o Choriocarcinoma
  • HCC especially in cirrhotic liver
  • Cholangiocarcinoma
226
Q

A young patient is diagnosed with multiple endocrine neoplasia (MEN) type 3 (also known as
type 2b) after an episode of bowel obstruction. Which one of the following features would he
be unlikely to have or develop in the future with this diagnosis? [B2 Q7]

a. Medullary carcinoma of the thyroid
b. Marfanoid appearance
c. Mucosal neuromas of the small bowel
d. Facial angiofibromas
e. Prognathism

A

Facial angiofibromas

MEN type 3 is a non-inherited syndrome primarily composing medullary thyroid carcinoma,
phaeochromocytomas and mucosal neuromas of the gastro-intestinal tract. Other features
include prognathism, marfanoid appearance and cutaneous neuromas. Facial angiofibromas are
associated with MEN type 1 and occur in greater than 80% of cases.

227
Q

A 48-year-old woman with upper abdominal pain is found to have a 4 cm hyper-vascular lesion
in the head of the pancreas on contrast-enhanced CT. She subsequently has an MR scan; the
lesion is of low intensity on fat-saturated T1-weighted and high intensity on T2-weighted
imaging. Which of the following is the most likely diagnosis? [B2 Q8]

a. Pancreatic adenocarcinoma
b. Gastrinoma
c. Insulinoma
d. Macrocystic adenoma
e. Pancreatic pseudocyst

A

Gastrinoma
Pancreatic adenocarcinoma is a hypovascular lesion. Macrocystic adenoma is also
hypovascular and is only rarely found in the head of the pancreas, with a predilection for the
tail. The differential therefore lies between insulinoma and gastrinoma. Although both CT and
MR imaging characteristics are similar, the majority of insulinomas are less than 1 cm in size,
whereas gastrinomas tend to be larger at presentation with an average size of approximately 3
cm. Gastrinoma is associated with peptic ulceration and Zollinger–Ellison syndrome.

228
Q

A 40-year-old with a Multiple Endocrine Neoplasia (MEN) syndrome has parathyroid
hyperplasia with hyperparathyroidism recently diagnosed. Which of the following is a feature
of MEN II rather than MEN I? [B3 Q21]

A. Phaeochromocytoma
B. Pancreatic islet cell tumour
C. Pituitary gland tumour
D. Adrenocortical adenoma
E. Carcinoid tumour

A

Phaeochromocytoma
Features of MEN2A include parathyroid hyperplasia, medullary T cell carcinoma of the thyroid,
and phaeochromocytoma. MEN2A may also be associated with carcinoid and Cushing’s.

229
Q

A 50-year-old woman presents with recurrent episodes of hypoglycaemia. Biochemistry
confirms endogenous insulin hypersecretion, and she undergoes multiphasic CT of the
abdomen. Which of the following is the most likely finding in the pancreas? [B4 Q15]

a. 1.5 cm arterially enhancing, solid mass in the head
b. 5 cm arterially enhancing, cystic mass in the tail
c. 1.5 cm arterially enhancing, cystic mass in the body
d. 1.5 cm non-enhancing, solid mass in the tail
e. 5 cm non-enhancing, cystic mass in the body

A

1.5 cm arterially enhancing, solid mass in the head
Insulinomas are rare tumours of the islet cells of the pancreas, which present at an early stage
with hypoglycaemic episodes. Diagnosis is made biochemically, by demonstrating fasting
hyperinsulinaemia, and the main purpose of imaging is to detect and localize accurately the
tumours, which tend to be small (,2cm) at presentation. Tumours may occur anywhere in the
pancreas, with 2–5% in an ectopic location. CT is considered the first-line investigation, but
newer techniques such as MR and functional imaging are being increasingly used. On CT,
insulinomas are typically solid and highly vascular, and are best visualized on arterial phase
imaging, when they demonstrate marked enhancement.

230
Q

Which of the following favour a diagnosis of glucagonoma rather than somatostatinoma? [B3
Q23]

A. Predilection for body/tail of pancreas
B. Average size > 4cm
C. Hyper-vascular
D. Duodenal obstruction
E. Association with Neurofibromatosis Type 1 (NFI)

A

Predilection for body/tail of pancreas

Glucagonomas are predominantly in pancreatic body/tail, mean size 6.4cm and hyper-vascular.
Somatostatinomas are predominantly in pancreatic head/duodenum of ampulla of Vater.

231
Q

A 35-year-old woman presents with severe watery diarrhoea. She has hypokalaemia. A
neuroendocrine tumour is diagnosed in the pancreatic body measuring 5cm. This is mildly
hyper vascular. Which of the following is the most likely diagnosis? [B3 Q24]

A. VIPoma (Verner Morrison Syndrome)
B. Somatostatinoma
C. Non-functional islet cell tumour
D. Glucagonoma
E. Gastrinoma

A

VIPOMA are usually in the pancreatic body/tail (75%). Extra-pancreatic locations include
retroperitoneal ganglioblastoma, phaeochromocytoma, lung, neuroblastoma (in children). 50-
80% of VIPOMAs undergo malignant transformation.

232
Q

A64-year-old woman presents to the dermatologist with erythematous maculopapular lesions
on her legs, buttocks, and face, and is diagnosed with necrolytic migratory erythema. Which
initial imaging investigation is most appropriate? [B4 Q17]

a. no imaging
b. mammography
c. CT of the brain
d. chest radiograph
e. CT of the abdomen

A

CT of the abdomen

Necrolytic migratory erythema is a rare dermatological condition with a strong association with
glucagonoma, an islet-cell tumour of the pancreas derived from alpha cells. Over 70% of
patients with glucagonoma demonstrate the condition, and they may also complain of weight
loss, diarrhoea, and diabetes. The association is considered strong enough to warrant thorough
investigation for pancreatic malignancy. Glucagonomas typically occur in the pancreatic body
or tail and are large (2.5–25cm) hyper-vascular tumours with solid and necrotic components.
They have a high rate of malignant transformation, and around 50% of patients have liver
metastases at the time of diagnosis.

233
Q

Which is the most likely source of a metastatic deposit to the pancreas? [B3 Q22]

A. Renal Cell Carcinoma (RCC)
B. Bronchogenic carcinoma
C. Breast cancer
D. Soft tissue sarcoma
E. Colon carcinoma

A

The most likely primary tumour leading to a metastatic deposit to the pancreas is renal cell
carcinoma
. These metastases are usually solitary and heterogeneously enhancing masses with
increased attenuation relative to the pancreas.

234
Q

A 63-year-old male patient is admitted with acute pancreatitis. During his admission survey he
is noted to have a Ranson score of 7 and he is transferred to the ICU. A CT scan is carried out
prior to ICU admission and shows a homogeneously enhancing enlarged pancreas with a fluid
collection in the tail. There are gallstones in the gallbladder, with a dilated duct. A further CT
is carried out on day 3 and this shows two further fluid collections in the tail of the pancreas,
with an area of poorly enhancing pancreas that involves over half of the gland. There is no
evidence of abscess. Which of the following options is most useful in detecting the severity of
this patient’s pancreatitis? [B1 Q41]

A. Ranson score to indicate severity of pancreatitis. CT is of value in detecting complications.
B. CT within 24 hours to show presence of gland swelling and/or necrosis.
C. CT scan within 24 hours to indicate absence of complications and evidence that the causative
factor has passed.
D. CT after 3 days showing necrosis and fluid collections.
E. CT scan after 3 days showing absence of abscess formation.

A

CT scan after 3 days showing necrosis and fluid collections

The Balthazar CT staging system grades pancreatitis based on the presence of gland
enlargement and/or fluid collections, as well as the presence of necrosis involving <30%, 30–
50% or >50% of the gland. This has been shown to be a more accurate predictor of severity of
pancreatitis and morbidity than the Ranson or Acute Physiology and Chronic Health Evaluation
II (APACHE II) criteria. This staging system is, however, most accurate when carried out after
48 hours, as the degree of pancreatic necrosis may not be apparent before this.

235
Q

A 45-year-old male patient presents to A&E with an 8-hour history of epigastric pain. There is
no history of alcohol intake. On examination he is tender in the epigastrium. The initial blood
tests reveal that his amylase is 1024. His WCC is slightly elevated at 15. His glucose, calcium,
PaO2, liver function tests (LFTs), lactate dehydrogenase (LDH) and serum electrolytes are all
normal. Following an erect CXR, what is the next most appropriate radiological investigation?
[B1 Q5]

A. Urgent CT scan to assess the pancreas.
B. CT within 24 hours.
C. Ultrasound scan within 24 hours.
D. Endoscopic retrograde cholangiopancreatography (ERCP) to look for ductal calculi.
E. MRCP on this admission to assess for ductal stones.

A

Ultrasound scan within 24 hours

CT is only indicated as an investigation in cases of pancreatitis with severe prognostic
indicators. This patient’s Ranson score is 1, which indicates a mild episode of pancreatitis and
therefore CT is not indicated. ERCP was formerly contraindicated in pancreatitis but is now
recognized as a treatment for obstructing stones in the ampulla that are causing the pancreatitis.
Ultimately further investigations can be directed based on the ultrasound findings.

Pancreatitis [Core Radiology]
Pancreatic parenchymal phase is later arterial (40s) is the most sensitive timing to detect subtle
low attenuation of necrosis.

CT imaging is not indicated in patients with clinical diagnosis of mild acute pancreatitis,
especially if they are improving. CT imaging may be negative or show a mildly oedematous
pancreas in these cases.

Acute pancreatitis

Causes: alcohol, stones, ERCP

Balthazar grading system: 14% MT for D, E (fluid collection is poor prognostic indicator)

A: Normal-appearing pancreas
B: Focal or diffuse pancreatic enlargement
C: Mild peripancreatic inflammatory changes
D: Single fluid collection
E: Two or more fluid collections

CT severity index (STSI)

o Assigns 0–4 points for Balthazar A–E
o Adds 0–6 points for necrosis to create a total score from 0-10. [ 0 points: 0% necrosis, 2
points: <30% necrosis, 4 points: 30–50% necrosis, 6 points: >50% necrosis]

Pancreatic and peripancreatic complications:

  1. Pancreatic necrosis:
    a. Focal area of non-enhancing pancreatic parenchyma
    b. Best seen 48-72 hours, late arterial phase
    c. Risk for infection and severe morbidity.
  2. Fluid collections: Peripancreatic fluid may resolve or may evolve either into peripancreatic
    abscess or pseudocyst.
  3. Pseudocyst: A pancreatic pseudocyst is a collection of pancreatic enzymes and fluid
    enclosed by a fibrous wall lacking an epithelial lining. The fibrous wall usually takes about
    4–6 weeks to mature.
  4. Pancreatic abscess:
    a. Thicker, more irregular wall to pseudocysts
    b. Gas locules

Extra-pancreatic complications:

  1. Extra-pancreatic pseudocyst
    a. nearly anywhere below the diaphragm.
    b. An intra-splenic pseudocyst → intra-splenic haemorrhage.
  2. Perihilar renal inflammation, which may lead to venous compression or thrombosis. 3. Bowel involvement, especially of the transverse colon.
  3. Vascular Complications
    a. Arterial bleeding from erosion into the splenic artery.
    b. Pseudoaneurysm of splenic artery
    c. Porto-splenic thrombosis, which may lead to portal hypertension.

Chronic pancreatitis

Causes: alcohol, pancreatic divisum.
Calcification in the distribution of pancreatic duct.

Autoimmune pancreatitis

Causes: Sjogren, IgG 4
Diffuse sausage shaped enlargement of pancreas.
Can be focal – mimics tumour, but no ductal dilatation.

Groove pancreatitis

Uncommon, young men, heavy drinkers
Focal pancreatitis of the groove between the head of the pancreas, duodenum, and common
bile duct.
Histology: Fibrosis in the pancreaticoduodenal groove.
Imaging: Duodenal thickening, stenosis, cystic changes of duodenal wall.
Differential: Adenoma head of pancreas.

236
Q

A 42-year-old man is referred for a CT scan by an upper GI surgeon. He has a long history of
recurrent upper abdominal pain, with more recent episodic vomiting. CT shows excess soft
tissue thickening between the head of pancreas and duodenum. Small cystic lesions are seen
along the medial wall of the duodenum. There is also mild dilatation of the common bile duct
and distension of the stomach and proximal duodenum. What is the most likely diagnosis? [B1
Q50]

A. Autoimmune pancreatitis.
B. Groove pancreatitis.
C. Pancreatitis related to ectopic or heterotopic pancreatic tissue.
D. Hereditary pancreatitis.
E. Pancreas divisum associated pancreatitis.

A

Groove pancreatitis

This is a rare form of chronic pancreatitis. It occurs due to inflammation in the pancreatico-
duodenal groove, the potential space between the pancreas, duodenum, and common bile duct.
The clinical manifestations are primarily due to duodenal and biliary obstruction. The small
cystic lesions in the duodenal wall refer to cystic dystrophy of the duodenum, which can be
associated with groove pancreatitis.
Autoimmune pancreatitis generally shows a diffusely enlarged gland with loss of lobular
architecture, a ‘sausage’ shape, and a peripheral ‘rind’ of hypoattenuation. There is usually a
non- dilated or diffusely narrowed pancreatic duct and a distal biliary stricture.

Hereditary pancreatitis has a young age of onset of typical features of pancreatitis, with at least
two acute attacks without an underlying cause.

Pancreas divisum associated pancreatitis is seen in young or middle-aged patients with
recurrent acute pancreatitis or chronic relapsing pancreatitis. MRCP or ERCP are optimal for
the diagnosis of the lack of communication between the ventral and dorsal pancreatic ducts.

Although ectopic pancreatic tissue is a proposed cause of groove pancreatitis, the ectopic or
heterotopic tissue is most seen in relation to the gastric wall.

237
Q

Which of the following most favours a diagnosis of an infected rather than a sterile post-
necrotic fluid collection, following acute pancreatitis? [B3 Q32]

A. Presence of gas bubbles
B. Size of collection > 5cm
C. Association with extra-pancreatic necrosis
D. Association with organised pancreatic necrosis
E. Association with central gland necrosis

A

Presence of gas bubbles
The only reliable feature on imaging indicating an infected collection is the presence of gas
bubbles. All three types of pancreatic necrosis organised central gland and extra-pancreatic are
associated with post-necrotic fluid collection. Necrotic pancreatitis becomes infected in 40-
70%, most often with gram negative enteric bacteria.

238
Q

An adolescent complains of chest and abdominal pain after suffering a handlebar injury whilst
out riding his bicycle. He undergoes a CT scan of abdomen, as the surgical team fear he may
have suffered a liver or splenic injury. In the recent past he has been complaining of loose
motions and his mother has noticed he has failed to thrive. He has a long history of respiratory
disease, which has been diagnosed as asthma. The CT scan shows low attenuation (-90 to -120
HU) in the region of the pancreas and air-trapping and cystic bronchiectasis in the upper lobes
of both lungs. Which of the following is the most likely underlying pathological process
explaining the appearance of the pancreas? [B1 Q46]

A. Chronic pancreatitis.
B. Congenital absence of the pancreas.
C. Lipomatous pseudohypertrophy of the pancreas.
D. Gluten enteropathy.
E. Shwachman–Diamond syndrome.

A

Lipomatous pseudohypertrophy of the pancreas.
The history is one of undiagnosed cystic fibrosis (CF), with respiratory disease and pancreatic
exocrine dysfunction. Pancreatic involvement in CF initially produces inhomogeneous
attenuation, then low attenuation, and then complete fatty infiltration and replacement.
Microcysts may develop and some of these may become small macroscopic cysts demonstrable
with CT. There may be scattered calcifications. On ultrasound, there is increased diffuse
echogenicity in keeping with fatty infiltration and fibrosis. CF is a major cause of pancreatic
exocrine failure in childhood. Pancreatic abnormalities are seen in 85–90% of CF patients.
However, the disease progresses to pancreatitis in less than 1% of CF patients. It predisposes
to pancreatic cancer.

Shwachman–Diamond syndrome is a rare congenital disorder characterized by pancreatic
exocrine insufficiency, bone marrow dysfunction, and skeletal abnormalities. Patients usually
present in infancy or early childhood with malabsorption and recurrent infections. Imaging
reveals pancreatic lipomatosis.

239
Q

In patients with cystic fibrosis, which gastrointestinal pathology may occur because of high-
dose lipase supplementation? [B4 Q57]

a. rectal prolapse
b. fibrosing colonopathy
c. pneumatosis intestinalis
d. gastro-oesophageal reflux
e. meconium ileus equivalent syndrome

A

Fibrosing colonopathy

Fibrosing colonopathy is a condition causing progressive submucosal fibrosis predominantly
affecting the proximal colon. It was first described in 1994 in children with cystic fibrosis
taking high-dose lipase supplementation to relieve the symptoms of exocrine pancreatic
insufficiency. It causes stricturing and longitudinal shortening of the right colon, and patients
present with obstruction. Overall, the gastrointestinal tract is affected in 85–90% of patients with cystic fibrosis, and all the above pathologies may occur, though only fibrosing
colonopathy is associated with high-dose lipase supplementation

240
Q

A 30-year-old man undergoes CT of the abdomen following a high-velocity collision during
an RTA. The scan reveals peripancreatic fat stranding and a superficial laceration in the tail of
the pancreas, which extends to less than 50% of the pancreatic thickness. What is the next most
appropriate step? [B1 Q45]

A. Laparotomy.
B. ERCP.
C. Supportive therapy.
D. Ultrasound to assess the pancreatic duct.
E. Diagnostic peritoneal lavage.

A

Supportive therapy

Injury to the pancreas is relatively uncommon, occurring in less than 2% of blunt abdominal
trauma patients. Direct signs of pancreatic injury at CT include laceration, transection,
enlargement, and inhomogeneous enhancement. Secondary signs include peripancreatic fat
stranding and fluid collections, haemorrhage, and thickening of the anterior pararenal fascia.

The management of pancreatic trauma depends on the integrity of the pancreatic duct. If it is
intact, the treatment is supportive and expectant. If the duct is disrupted, surgery or stenting at
ERCP is required. Although CT may not always directly demonstrate the pancreatic duct, the
likelihood of ductal injury may be inferred from secondary signs.

Wong et al. have devised a CT grading scheme, which is like the surgical classification of
Moore. Grade A injuries comprise pancreatitis or superficial laceration (<50% pancreatic
thickness), grade B1 is deep laceration (>50% pancreatic thickness), grade B2 is transection of
the pancreatic tail, grade C1 is deep laceration of the pancreatic head, and grade C2 is
transection of the pancreatic head.

Grade A injuries spare the duct and are usually seen with an intact duct by surgical grading.
Grade B and C injuries correlate with duct disruption. MR pancreatography is an alternative to
ERCP to assess the integrity of the pancreatic duct. The duct integrity cannot be reliably
assessed by ultrasound, particularly in the context of recent trauma.

241
Q

A45-year-old man is admitted after a road traffic accident in which he sustained abdominal
injuries. After fluid resuscitation he undergoes CT of the abdomen and pelvis with intravenous
contrast. This demonstrates a serpiginous area of attenuation value 130 HU at the splenic hilum
with surrounding lower-attenuation material. What is this most likely to represent? [B4 Q79]

a. active arterial extravasation
b. acute clotted blood
c. acute unclotted blood
d. splenic arterial calcification
e. ascites

A

Active arterial extravasation

In the evaluation of haemo-peritoneum by CT, attenuation values can help differentiate ascites,
unclotted blood, active bleeding, and haematoma. Blood usually has a higher measured
attenuation than other body fluids, but its appearance depends on the age, extent, and location
of haemorrhage. Unclotted blood has an attenuation value of 30–45 HU, but this may be lower
in patients with a lower serum haematocrit and if the haemorrhage is more than 48 hours old.
Clotted blood has an attenuation value of 45–70 HU, and identification of the area of
highestattenuation haematoma (sentinel clot) on CT indicates the site of bleeding. Active
arterial extravasation is seen as an area of higher attenuation resembling that in the aorta,
ranging from 85 HU to 370 HU. It may be surrounded by lower-attenuation haematoma. This
finding indicates the need for urgent embolization or surgical treatment.

242
Q

A 32-year-old woman with no significant past medical history has a CT scan as an outpatient
for right iliac fossa pain. No cause for the pain is discovered on this investigation. However, a
1 cm diameter, smoothly marginated, circular, homogenous area of tissue is seen next to the
splenic hilum. This area of tissue is isodense compared to normal splenic parenchyma. What
is this most likely to be? [B2 Q41]

a. Splenosis
b. Splenunculus
c. Lymphoma
d. Splenic hamartoma
e. Wandering spleen

A

Splenunculus

Splenunculus is most likely and is often seen incidentally. It is much more common than
splenosis and is more likely to occur at the splenic hilum than splenosis. A splenunculus, or
accessory spleen, is present in up to 30% of people and is most often located near the splenic
hilum but can occur anywhere in the abdomen. Spleno-gonadal fusion is a recognised entity
whereby the accessory splenic tissue is attached to the left ovary or testis. Splenosis occurs
following trauma, whereby splenic tissue auto transplants elsewhere in the abdomen, and can
also implant above the diaphragm if associated with diaphragm rupture. A wandering spleen
denotes abnormal mobility of the spleen on long peritoneal ligaments.

243
Q

A 64-year-old man with known sigmoid adenocarcinoma diagnosed at endoscopy undergoes
staging CT. A solitary, welldefined, rounded, homogeneous, 1 cm mass of mean attenuation
value identical to the spleen is seen near the splenic hilum. What is the most likely diagnosis?
[B4 Q32]

a. metastatic lymph node
b. wandering spleen
c. splenic artery aneurysm
d. polysplenia
e. accessory spleen

A

Accessory spleen (splenunculus)

Accessory spleen (splenunculus) is seen in 10–30% of the population and results from
developmental failure of fusion of the mesodermal buds that form the spleen. They appear as
small (usually ,10 mm), well-defined masses with identical attenuation and enhancement
characteristics as the spleen and are most commonly located near the splenic hilum. Following
splenectomy for haematological disorders, an accessory spleen may undergo hypertrophy and
result in recurrence of the original disorder. Regional lymph nodes from colorectal cancer
extend along the course of the main vessels supplying the segment of bowel. Wandering spleen
refers to a normal spleen positioned in an abnormal location within the abdomen due to laxity
of the splenic ligaments. Splenic artery aneurysms are focal dilatations of the splenic artery, which may be intra- or extra-splenic and are frequently calcified. Poly-splenia is a rare
congenital disorder associated with situs ambiguous and characterized by multiple small
spleens usually in the right abdomen.

244
Q

A 45-year-old woman with pleuritic chest pain and breathlessness undergoes CT pulmonary
angiogram for suspected acute pulmonary embolism, which demonstrates multiple irregular
areas of relatively poor enhancement in the visualized portion of the spleen. What is the most
likely cause? [B4 Q32]

a. normal arterial-phase enhancement
b. splenic infarction
c. splenic clefts
d. splenosis
e. spontaneous splenic rupture

A

Normal arterial-phase enhancement

CT pulmonary angiogram is performed during pulmonary arterial phase enhancement. During
arterial phase enhancement, variable rates of flow through the splenic parenchyma result in
heterogeneous enhancement, which may appear as alternating bands of high and low
attenuation, or give the impression of irregular, low-density mass lesions. Enhancement
becomes homogeneous in the portal venous phase. Splenic infarction is the most common
cause of (true) focal splenic defects, and typically appears as single or multiple, wedge-shaped,
peripheral, low-attenuation defects. Clefts in the splenic contour are common normal variants,
appearing as smoothly contoured, medially located defects, and should not be mistaken for
lacerations. Splenosis is the implantation of splenic tissue in ectopic sites following traumatic
rupture or splenectomy, and appears as multiple, small, homogeneous, enhancing masses that
may mimic peritoneal deposits. Spontaneous splenic rupture is rare, though it may be delayed
following trauma or be associated with splenomegaly. Appearances may include low-density,
linear, parenchymal lacerations and areas of mottled parenchymal enhancement representing
contusions.

245
Q

A 54-year-old man has a CT scan of renal tracts for suspected right renal colic. The right renal
tract is normal, but an incidental 6-cm well-defined cyst is noted within the spleen. There is no
past medical history of note. What is the most likely aetiology of the splenic cyst? [B1 Q63]

A. Previous trauma.
B. Echinococcal infection.
C. Congenital cyst.
D. Liquefied infarct.
E. Unilocular lymphangioma.

A

Previous trauma

Splenic cysts are classified into two major subtypes: true cysts and false cysts (post-traumatic
pseudocysts). This differentiation is based on the presence or absence of an epithelial lining.
They cannot be distinguished on imaging. On CT as both usually appear well-defined and of
fluid density.

True cysts, which constitute approximately 20% of all splenic cysts and show an epithelial
lining, are further divided into non-parasitic cysts and parasitic subtypes. Non-parasitic, true
cysts, primarily known as epidermoid cysts, are congenitally derived from peritoneal
mesothelium and represent only 2.5% of all splenic cysts. Most true cysts are related to parasitic
infection, usually hydatid disease.

False cysts, also known as post-traumatic pseudocysts, lack an epithelial lining and are
considered to represent the end stage of a previous intrasplenic hematoma. They account for
up to 80% of all splenic cysts. Patients may report a history of trauma to the left upper quadrant,
but up to 30% of patients do not recall any association with such an event. More rarely they
may be the result of previous infarction or infection.

True cysts are more likely to have slight wall trabeculation and thin peripheral septation (up to
86%), whereas false cysts are more likely to have mural calcification (up to 50%).

Isolated splenic lymphangioma is uncommon in adults, usually being diagnosed in childhood.
It usually appears as thin-walled, well-defined masses of low attenuation, without enhancement.
They may have curvilinear peripheral mural calcification.

246
Q

A64-year-old man undergoes renal ultrasound scan and is incidentally noted to have a well-
defined, rounded, 3 cm lesion in the spleen. It has a thin wall with curvilinear rim calcification
and contains low-level internal echoes. What is the most likely diagnosis? [B4 Q64]

a. post-traumatic (false) cyst
b. epidermoid cyst
c. echinococcal cyst
d. pyogenic abscess
e. pancreatic pseudocyst

A

Post traumatic (false) cyst Approximately 80% of all splenic cysts are post-traumatic cysts (also known as false cysts or
non-pancreatic pseudocysts), which represent the cystic end stage of trauma, infection, or
infarction. They are not true cysts, as they lack an epithelial wall. They often contain internal
echoes from debris and cyst wall calcification. True cysts may be parasitic (echinococcal) or
non-parasitic (epithelial). The spleen is involved in 0.9–8% of cases of echinococcal disease.
Features include daughter cysts, and multiple internal dependent echogenic foci resulting in a
‘snowstorm’ sign. Epidermoid cysts are congenital cysts with the same appearance as post-
traumatic cysts, but they are rarer, and rim calcification is less commonly seen. Pyogenic
abscesses tend to appear as irregular hypoechoic areas on ultrasound scan, and hyperechoic gas
bubbles may be visible. Intrasplenic pancreatic pseudocysts are seen in up to 5% of patients
with pancreatitis

247
Q

On contrast-enhanced CT of the abdomen, what is the most common abnormality of the spleen
seen in sarcoidosis? [B4 Q68]

a. capsular calcification
b. multiple low-attenuation nodules
c. splenomegaly
d. splenic rupture
e. multiple cystic lesions

A

**Splenomegaly **

Although symptoms directly referable to the spleen are unusual, autopsy studies have
demonstrated the spleen to be involved in 38–77% of patients with sarcoidosis. The commonest
abnormality demonstrated on imaging is splenomegaly, occurring in up to 60% of patients.
Multiple hypoattenuating nodules measuring up to 3cm may be seen distributed diffusely
throughout the spleen in around 15% of cases and may occur in the absence of splenomegaly.
The lesions appear hypointense on all sequences at MRI and are best seen on T1W or early
phase, gadolinium-enhanced, T2W fat-suppressed sequences. Abdominal or systemic
symptoms are more frequent in patients with nodular hepatosplenic sarcoidosis. Spontaneous
splenic rupture in sarcoidosis has been described but is very rare. Capsular calcification and
multiple cystic lesions are not features of the disease

248
Q

A 73-year-old female has a CT abdomen and pelvis for the investigation of anaemia and weight
loss. Massive splenomegaly (30cm) is present with no other abnormalities. Which of the
following conditions is most likely to be the underlying cause? [B2 Q58]

a. Sarcoidosis
b. Felty syndrome
c. Chronic myeloid leukaemia
d. Haemochromatosis
e. non-Hodgkin’s lymphoma

A

Chronic myeloid leukaemia

Splenomegaly is a relatively common finding in many different diseases, but massive
splenomegaly always indicates underlying pathology. Although there is no unifying definition,
it is often recognised to be enlargement of the spleen into the left lower quadrant of the
abdomen or crossing the midline. All the options listed are causes of splenomegaly, however
chronic myeloid leukaemia is the only listed cause of massive splenomegaly. Other causes of
massive splenomegaly include Gaucher’s disease, malaria, myelofibrosis, schistosomiasis and
Leishmaniasis.

249
Q

A patient with a known history of malignancy undergoes a CT scan of the chest, abdomen, and
pelvis for staging purposes. This examination identifies a solitary hypodense lesion in the
spleen measuring 4 cm in diameter, but no other evidence of metastatic disease. A PET-CT is
considered as a possible mechanism for determining whether this is a metastasis but is
considered not likely to be helpful. Which malignancy is the patient most likely to have? [B1
Q62]

A. Melanoma.
B. Lung carcinoma.
C. Lymphoma.
D. Renal cell carcinoma.
E. Colon carcinoma

A

Renal cell carcinoma.

All the other tumours are more likely to be FDG avid than renal cell carcinoma.

250
Q

A 24-year-old male patient is brought into A&E following a high-speed RTA. His blood
pressure was 90/60 mmHg, and his heart rate was 112 on admission, but these observations
respond well to intravenous fluids and the patient has remained stable since. He complains of
left-sided abdominal pain. A pneumothorax is noted on CXR, with associated left-sided rib
fractures. An urgent CT scan of chest and abdomen is carried out. This reveals fluid in the
abdomen. A cresenteric area of low attenuation is noted around the spleen. There is a further
area of hypoattenuation passing 4 cm into the splenic parenchyma, adjacent to the hilum. The
rest of the splenic parenchyma is of uniform attenuation. The CT also shows a flail segment of
chest and an area of lung contusion at the left base. Which of the following statements regarding
the spleen is true? [B1 Q7]

A. The appearances described represent subcapsular haematomas.
B. The appearances described represent a haematoma and a parenchymal laceration. The
presence of free fluid represents acute haemorrhage, and a laparotomy is indicated.
C. The appearances are consistent with a shattered spleen as the laceration extends to the hilum.
D. The appearances are consistent with a subcapsular haematoma and a splenic laceration.
Conservative management is appropriate with serial CT scans.
E. Whilst the appearances are consistent with a laceration and subcapsular haematoma,
radiological findings are not reliable in determining the need for a laparotomy.

A

Whilst the appearances are consistent with a laceration and subcapsular haematoma,
radiological findings are not reliable in determining the need for a laparotomy.

Splenic injuries can be graded 1–5 (American Association of Trauma Surgeons).

Grade 1 is a subcapsular haematoma that involves <25% of the splenic surface or a laceration
<1cm deep.
Grade 2 is a haematoma that involves 25–50% of the surface or a laceration up to 3cm deep.
Grade 3 is a haematoma involving >50% of splenic surface or 10 cm in length or a laceration
greater than 3 cm into the parenchyma.
Grade 4 is a laceration extending into the hilum that devascularizes up to 25% of the spleen.
Grade 5 is a shattered spleen, with multiple lacerations or a spleen avulsed from its vascular
bed.

Radiological findings do not correlate well with requirement for laparotomy in the more minor
splenic injuries. Clinical assessment is of more value, with surgery only indicated in unstable
patients. The value of radiology is in detecting other injuries and in quantifying the amount of
the haematoma due to the risk of delayed splenic rupture in more severe injuries.

251
Q

A 32-year-old male front seat passenger is involved in a road traffic accident and sustains blunt
abdominal trauma. He is admitted via the emergency department and CT reveals a splenic
laceration with subcapsular haematoma. Which one of the following associated injuries is most
likely to be found? [B2 Q2]

a. Diaphragmatic rupture
b. Injury to the liver
c. Injury to the left kidney
d. Ipsilateral rib fractures e. Injury to the small bowel mesentery

A

Ipsilateral rib fractures

All are potential associated injuries and should be actively searched for in the context of blunt
abdominal trauma. Rib fractures are found in up to 50% of patients with splenic injuries and
as such are the most common association. The left kidney is injured in 10% of patients with
splenic injury, and diaphragm rupture is even rarer. Diaphragm rupture may be difficult to
appreciate on axial slices and may be more evident on coronal reformats.

252
Q

Which of the following is correct regarding carcinoid of the GI tract? [B1 Q4]

A. A minority are asymptomatic when discovered.
B. The appendix is the most common site of occurrence, representing 33% of all carcinoids.
C. Over 50% are multiple.
D. Although malignant change is uncommon in appendiceal carcinoid, as this is the most
common site, it accounts for most malignant carcinoids.
E. The size of the tumour at diagnosis is related to the risk of metastatic spread.

A

The size of tumour at diagnosis is related to the risk of metastatic spread

Carcinoid is the 33% tumour, as
* 33% occur in the small bowel, (75% metastatic disease)
* 33% are multiple,
* 33% are malignant, and
* 33% are associated with a second malignancy.

Appendiceal carcinoid accounts for
* 50% of all carcinoids and
* 33% are symptomatic at presentation.
* 7% of metastatic disease

The size of the tumour at diagnosis is related to the risk of metastatic spread,
* 2% if the lesion is <1 cm,
* 85% if the lesion is > 2 cm.

253
Q

A 67-year-old man presents with abdominal pain, distension, and vomiting. Multiple dilated
loops of small bowel are seen on plain abdominal radiograph. A contrast-enhanced CT of the
abdomen and pelvis is performed, which shows small bowel dilatation to the terminal ileum,
where there is a nodular calcified mass with surrounding desmoplastic reaction. What is the
most likely cause of small bowel obstruction? [B4 Q36]

a. adhesions
b. carcinoid tumour
c. Crohn’s disease
d. previous irradiation
e. small bowel lymphoma

A

Carcinoid tumour

The commonest cause of small bowel obstruction in adults is adhesions, which are diagnosed
on CT when there is an abrupt calibre transition without an associated mass, surrounding
inflammatory changes or bowel wall thickening. Carcinoid tumours comprise 25% of all tumours of the small bowel. They are commonly asymptomatic but may present with pain or
obstruction, or with carcinoid syndrome in 7% of patients. Most occur in the ileum, and typical
appearances are of a calcified mass with surrounding desmoplastic reaction, retraction of the
mesentery and thickening of surrounding loops of bowel. Small bowel obstruction complicates
Crohn’s disease in approximately 15% of patients, due to thickening and submucosal oedema
of the bowel wall. Previous irradiation may result in adhesions and fibrotic changes in the
mesentery that can cause bowel obstruction. Lymphomatous involvement of the small bowel
causes circumferential bowel wall thickening, but this rarely results in obstruction.

GI carcinoid [STATdx]

Def: Well-differentiated neuroendocrine tumour from GI, can also from lung and GU

Site:

  • Midgut carcinoids (jejunoileal) (45%)
    o 90% arise in ileum, within 60 cm of ileocecal valve
    o Account for most cases of carcinoid syndrome
  • Gastric carcinoids (7%)
    o Gastrinoma (Zollinger-Ellison or MEN1 syndrome)
    o Pernicious anaemia or atrophic gastritis
  • Appendiceal carcinoids (16%)
    o Most common tumour of appendix
    o Usually found incidentally at appendectomy
  • Colorectal (31%)
    o Usually asymptomatic until large
  • Carcinoid syndrome = metastatic spread to liver
    o Spectrum of symptoms (flushing, diarrhea, asthma, pain, right heart failure)
    o Symptoms require systemic circulation of hormonal factors produced by carcinoid

Imaging:

  • Solitary, enhancing distal ileal mass with mesenteric metastases (More difficult to detect
    primary than metastatic foci)
  • Mesenteric mass due to direct invasion or nodal metastasis
    o Calcification is common
    o Spiculated with stellate pattern
    o Tethering, fixation, retraction of small bowel loops
    o Encasement and narrowing of mesenteric vessels
  • Liver metastases: Hyper vascular, Hyperechoic
    o Intense enhancement in arterial phase
    o Washout on portal venous and delayed phases
  • In-111 octreotide or somatostatin receptor scintigraphy Less sensitive but specific
  • Gallium-DOTATATE PET/CT: Localization, targeted therapy

Top Differential Diagnoses:

  • Sclerosing mesenteritis
    Infiltrated jejunal mesentery (not ileal) and usually larger
  • Gastrointestinal stromal tumour
    Hyper-vascular tumour, not associated with desmoplastic effect on mesentery
254
Q

An 83-year-old woman is investigated for weight loss, and undergoes contrast enhanced CT
scan of the chest, abdomen and pelvis. Multiple hypervascular metastases are found in the liver.
Which one of the following is most likely to be the primary tumour? [B2 Q9]

a. Adenocarcinoma of the stomach
b. Invasive ductal carcinoma of the breast
c. Carcinoid tumour
d. Adenocarcinoma of the sigmoid
e. Pancreatic ductal adenocarcinoma

A

Carcinoid tumour
Of the options listed, carcinoid tumour is the only primary tumour that typically causes hyper
vascular liver metastases. Other causes of hyper vascular liver metastases are pancreatic islet
cell tumours, phaeochromocytoma and renal cell carcinoma. Stomach, breast, lung and colon
cancers are associated with hypo vascular liver metastases. Liver metastases from carcinoid
tumours are more common with increasing size of the primary tumour. The incidence of
metastases depends on the location of the primary tumour, where approximately 30% of
carcinoids of the ileum metastasise compared to less than 5% of carcinoids of the appendix.

255
Q

A 45-year-old female has a CT for abdominal pain and weight loss. Findings include a soft-
tissue mass at the root of the small bowel mesentery with eccentric calcifications and tethering
of adjacent small bowel loops resulting in a moderate degree of small bowel obstruction. There
is a desmoplastic reaction within the surrounding mesentery. Which one of the following is the
most likely diagnosis? [B1 Q39]

a. Lymphoma
b. Carcinoid tumour
c. Melanoma metastases
d. Tuberculosis
e. Paraganglioma

A

Carcinoid tumour

These features are typical of carcinoid tumour. The desmoplastic reaction appears on CT as
thickened mesentery in a radiating pattern away from the soft-tissue mass, with beading of the
mesenteric vascular bundles.

256
Q

The CT of a 50-year-old man with abdominal pain shows a stellate enhancing mesenteric mass
with a radiology pattern of linear densities emanating from the mass. Adjacent small bowel
loops show tethering and thickening. Which is the most likely diagnosis? [B3 Q2]

A. Metastatic carcinoid tumour
B. Primary mesenteric mesothelioma
C. Desmoid tumour
D. Non-Hodgkin’s Lymphoma (NHL)
E. Abdominal TB

A

The stellate pattern is characteristic for carcinoid tumour

257
Q

Which of the following favours appendiceal carcinoid over appendiceal adenocarcinoma? [B3
Q3]

A. Bulbous swelling involving the distal third of the appendix
B. Slow growth
C. Direct extension into caecum
D. Ileocecal lymph node enlargement
E. Peri-appendiceal fat standing

A

Carcinoid often appears as a bulbous swelling of the distal third of the appendix, in contrast to
adenocarcinoma which tends to involve the proximal third and causes diffuse thickening of the
appendiceal wall.

258
Q

A 26-year-old man, with a previous history of a pancreato-colectomy for Gardner’s syndrome,
presents with vague abdominal discomfort and a CT scan is requested to ascertain the cause.
He is found to have a well-defined mass of homogenous density, which you suspect may be a
desmoid tumour, given the previous clinical history. Where in the abdomen is this most likely
to be located? [B1 Q68]

A. Abdominal wall.
B. Retroperitoneum.
C. Small bowel mesentery.
D. Pelvis.
E. Duodenal wall.

A

Small bowel mesentery
Desmoid tumours are non-malignant fibrous tumours that have a particular association with
FAP/Gardner syndrome. They may be locally infiltrative. On CT they are usually of
homogeneous density but can have well-defined or irregular margins. On MRI, the signal
intensity on T1WI is similar to muscle and on T2WI their signal can be variable. Lower signal
tumours on T2WI probably have a denser fibrous component. Desmoid tumours in association
with FAP/Gardner syndrome are most seen in the small bowel mesentery, followed by the
abdominal wall. Intra-abdominal desmoid tumours can also occur in the retroperitoneum and
pelvis, but these locations are more common in isolated desmoids.

259
Q

A 54-year-old man presents with persistent abdominal pain and fever. His amylase has been
normal, and colonoscopy and small bowel series were unremarkable during previous
investigation. He has a past medical history of thyroid disease. A CT of abdomen reveals ill-
defined rounded areas in the root of the mesentery, with adjacent mild lymphadenopathy. There
is some central calcification. A rim of preserved fat is seen surrounding the adjacent vessels.
What is the most likely diagnosis? [B1 Q47]

A. Sclerosing mesenteritis.
B. Desmoid tumour.
C. Carcinoid tumour.
D. Lymphoma.
E. Metastatic disease.

A

Sclerosing mesenteritis

This is a rare condition of unknown cause characterized by chronic mesenteric inflammation.
It is most frequently seen in the sixth decade and more commonly in males than females. It is
often associated with other inflammatory disorders such as retroperitoneal fibrosis, Riedel
thyroiditis, and sclerosing cholangitis. Symptoms include abdominal pain, nausea, fever,
intestinal obstruction or ischaemia, a mass, or diarrhoea. The CT findings can range from subtle
increased attenuation in the mesentery to a solid soft-tissue mass. The mass may envelop
vessels, but there may be preservation of fat around the vessels, the ‘fat halo’ sign. This finding
may help distinguish sclerosing mesenteritis from other mesenteric processes such as
lymphoma, carcinomatosis, or carcinoid tumour. Calcification may be present, usually in the
central necrotic portion. Enlarged mesenteric or retroperitoneal lymph nodes may also be
present.

Lymphoma will not display calcification unless it has undergone treatment. Carcinoid can
produce the appearance described, but the ‘fat halo’ sign favours sclerosing mesenteritis and
the soft tissue in carcinoid usually has a surrounding desmoplastic reaction. Metastatic disease
will not be confined to the root of the mesentery but will also involve the omentum or the
surfaces of the liver, spleen, or bowel. Ascites is also common with carcinomatosis but is not
associated with sclerosing mesenteritis. Mesenteric involvement in the case of desmoid
tumours is more often seen in cases related to familial adenomatous polyposis syndrome or
Gardner syndrome. They are usually large masses, measuring 15 cm or more at diagnosis. They
do not typically contain calcification.

Sclerosing Mesenteritis [STATdx]

  • Idiopathic inflammatory and fibrotic disorder of unknown aetiology affecting mesentery.
  • Symptoms
    o Acute abdominal pain
    o Bowel, urinary tract obstruction and bowel wall ischaemia in chronic phase
  • Histology
    o Mesenteric panniculitis
    o Mesenteric lipodystrophy
    o Retractile mesenteritis
  • Imaging o Acute
    ▪ Misty mesentery
    ▪ Thin pseudo capsule – 3 mm
    ▪ Cluster of mildly enlarged mesenteric lymph nodes.
    ▪ Usually left upper quadrant mesentery – root of jejunal mesentery
    ▪ Mesenteric vessels and nodes have halo of spared fat (fat ring or fat halo)
    o Chronic
    ▪ Discrete fibrotic soft tissue with desmoplastic reaction
    ▪ Stellate appearance with calcification within mass
    ▪ Encasement of mesenteric vessels (especially porto-mesenteric veins) with
    resultant collaterals – resultant narrowing and occlusion
    ▪ Small bowel wall oedema and mucosal hyperaemia as result of
    lymphatic/venous obstruction and ischemia
    ▪ Tethering of bowel loops can lead to bowel obstruction
260
Q

A 48-year-old male presents with abdominal pain, nausea, and weight loss. Contrast-enhanced
CT of the abdomen and pelvis reveals a heterogeneous, well-defined fatty mass at the root of
the small bowel mesentery. The mesenteric vessels are surrounded but not distorted by the
mass, and the vessels are surrounded by an apparent low-attenuation halo. The small bowel
and right colon are normal. Which is the most likely diagnosis? [B2 Q46]

a. Tuberculosis
b. Mesenteric lymphadenitis
c. Mesenteric panniculitis
d. Radiation enteritis
e. Mesenteric lipoma

A

Mesenteric panniculitis

These CT findings are typical of mesenteric panniculitis. This is an idiopathic, indolent
condition characterised by inflammation of the small bowel mesentery adipose tissue. Fibrosis
can predominate, in which case the CT appearances are of an infiltrative soft-tissue mass with
soft-tissue density strands radiating away from it. In this situation it has similar appearance to
lymphoma, carcinoid or desmoid tumours or retroperitoneal fibrosis, and biopsy is required to
differentiate. Mesenteric panniculitis often presents with non-specific symptoms such as
abdominal pain, weight loss, nausea, vomiting and pyrexia, and is usually indolent and self-
limiting.

261
Q

Which of the following is the hallmark of sclerosing mesenteritis on CT? [B3 Q1]

A. Increased density of mesenteric fat to 40-60 HU
B. Collateral vessel formation
C. Fatty necrotic cystic mass formation
D. Calcification within fat necrosis
E. Mesenteric lymph node enlargement > 10mm

A

Increased density of mesenteric fat to 40-60 HU
An increased attenuation of mesenteric fat (typically 40-60HU) is typical in sclerosing
mesenteritis on CT

262
Q

Which one of the following is false regarding peritoneal and mesenteric structures? [B1 Q14]

A. The lesser sac communicates with the rest of the abdominal cavity through the foramen of
Winslow.
B. The left paracolic gutter communicates with the left subphrenic space.
C. The falciform ligament connects to the left coronary ligament.
D. Part of the duodenum is suspended in the lesser omentum.
E. The right paracolic space communicates with the pouch of Douglas.

A

The left paracolic gutter communicates with the left subphrenic space.
There is no direct connection between these two spaces due to the phrenico-colic ligament. The
rest of the statements are correct.

263
Q

A 45-year-old man presents with acute abdominal pain. He has pyrexia and his inflammatory
markers are raised. The surgical team request a CT scan of abdomen for ‘? perforation’. The
CT reveals inflammatory change in the anterior pararenal space. Which of the following is least
likely to be the underlying cause for the CT finding? [B1 Q42]

A. Acute pancreatitis.
B. Gastric ulceration.
C. Diverticulitis of the descending colon.
D. Duodenal perforation.
E. Perforation of ascending colon due to neoplasm.

A

Gastric ulceration
The anterior pararenal space extends between the posterior parietal peritoneum and the anterior
renal fascia (Gerota’s fascia). It is bounded laterally by the lateral conal fascia.The pancreas,
second and third parts of the duodenum, and ascending and descending colon are located within
the anterior pararenal space, and disease in this space usually arises in these organs.The
stomach is intraperitoneal.

264
Q

A 67-year-old man presents to A&E with abdominal pain. Inflammatory markers are raised,
but serum electrolytes, amylase, haemoglobin, and coagulation are normal. He takes no regular
medication. On examination, there is a palpable mass in the right lower quadrant. CT reveals a
lobulated, hypoattenuating mass with thick walls, septa, and curvilinear calcifications. It is in
the RIF and displaces and distorts the adjacent psoas muscle. What is the most likely diagnosis?
[B1 Q44]

A. Pancreatic pseudocyst.
B. Pseudomyxoma retroperitonei.
C. Urinoma.
D. Haematoma.
E. Retroperitoneal liposarcoma.

A

Pseudomyxoma retroperitonei

The displacement of the psoas muscle indicates that this mass is most likely retroperitoneal.
Pseudomyxoma peritonei is a rare condition that is characterized by intraperitoneal
accumulation of gelatinous material owing to the rupture of a mucinous lesion of the appendix
or ovary, e.g. mucinous cystadenoma/cystadenocarcinoma. It may occur in the retroperitoneum,
where it is caused by the rupture of a mucinous lesion in the retrocaecal appendix and fixation
of the lesion to the posterior abdominal wall. Clinically, it results in abdominal pain and a
palpable mass. At CT, it appears as a multi-cystic mass with thick walls or septa that displace
and distort adjacent structures. Curvilinear or punctuate mural calcifications may also occur
and are highly suggestive.

Pancreatic pseudocysts usually occur in the peripancreatic space, but may occur in the abdomen,
pelvis, or mediastinum. They are associated with the clinical findings of pancreatitis and
elevation of serum amylase.

A urinoma is an encapsulated collection of chronically extravasated urine. There is usually a
history of trauma and an associated hydronephrosis.
Haematomas are associated with trauma, coagulopathy/anticoagulants, or a ruptured
abdominal aortic aneurysm. Chronic haematoma can result in low attenuation contents, but
acutely the haematoma will have higher attenuation than pure fluid due to clot formation.

Retroperitoneal liposarcoma is most commonly of a density between water and muscle
(myxoid type). It may have a solid, mixed, or pseudo-cystic pattern on CT. There may also be
macroscopic areas of lipid in well-differentiated liposarcomas. Patients present with abdominal
pain, weight loss, a palpable mass, and anaemia.

265
Q

A 71-year-old man is referred to CT for unexplained abdominal distension. Low attenuation
intraperitoneal collections with enhancing septae are demonstrated. There is scalloping of the
liver border and omental thickening. Which one of the following is most likely to be the
underlying cause? [B2 Q13]

a. Carcinoid tumour of the appendix
b. Cystadenocarcinoma of the appendix
c. Melanosis coli d. Mastocytosis
e. Retroperitoneal fibrosis

A

Cystadenocarcinoma of the appendix

The CT findings described are consistent with pseudomyxoma peritonei. This describes
abdominal distension secondary to the accumulation of large quantities of gelatinous ascites.
It is most caused by cystadenocarcinoma of the appendix in males and cystadenocarcinoma of
the ovary in females. Surgical debulking and intraperitoneal chemotherapy may be offered as
a treatment. Bowel obstruction is a frequent complication that may necessitate surgery.

266
Q

Which of the following favours pseudomyxoma peritonitis over sclerosing encapsulating
peritonitis? [B3 Q4]

A. Scalloping of the liver edge
B. Loculated fluid collection
C. Peritoneal calcification
D. Tethering of small bowel loops
E. Peritoneal enhancement

A

Scalloping of the liver edge
Pseudomyxoma peritonei is indicated by ascites and low attenuation masses. The ascites
contains septae, which are the margins of mucinous nodules, typically scalloping the liver edge.
Solid non-mucin-producing components cause soft tissue thickening of the peritoneal surface.

267
Q

A 47-year-old woman presents with progressive abdominal distension. CT of the abdomen
demonstrates loculated collections of very low-attenuation fluid in the peritoneal cavity
containing scattered curvilinear calcifications, and a scalloped contour to the liver. What is the
most likely diagnosis? [B4 Q14]

a. pancreatic pseudocysts
b. ascites due to liver cirrhosis with portal hypertension
c. peritoneal metastases
d. pseudomyxoma peritonei
e. peritoneal mesothelioma

A

Pseudomyxoma peritonei

Pseudomyxoma peritonei results from rupture of a benign or malignant mucin-producing
tumour, most commonly of the appendix or ovary, but also of the pancreas, stomach, and colon.
Typical findings are of large volume mucinous ascites, which appears on CT as very low
attenuation (mucinous) fluid collections in the omentum, mesentery, and peritoneal cavity,
often loculated and containing curvilinear or punctuate calcifications. A characteristic feature
is scalloping of the contour of the liver and splenic margins, which distinguishes mucinous
from serous ascites. Peritoneal metastases may also demonstrate loculated peritoneal fluid
collections, but there are typically nodular peritoneal densities and thickening of the greater
omentum (omental cake). Peritoneal mesothelioma usually presents with thickened mesentery,
omentum, peritoneum, and bowel wall, with a disproportionately small amount of ascites.

268
Q

A 35-year-old female undergoes an MRI of abdomen that shows multiple cystic lesions in the
pancreas. Each lesion consists of a cluster of small cysts with central scar. Multiple cysts and
solid lesions are also noted in both kidneys. What further investigation/s would you recommend?

A. Ophthalmology referral.
B. MRI of the brain.
C. MRI of the spine.
D. Molecular genetic testing and genetic counselling.
E. All the above.

A

All the above
All the findings are manifestations of von Hippel–Lindau (VHL) disease, which is a rare,
autosomal dominant, multisystem disorder characterized by the development of multiple
benign and malignant neoplasms.

The clinical manifestations are broad and include central nervous system (CNS) and retinal
hemangioblastomas, renal cysts and tumours, pancreatic cysts, and tumours,
phaeochromocytomas, endolymphatic sac tumours, and epididymal cystadenomas.

The diagnostic criteria for VHL include:

(i) >1 CNS hemangioblastoma,
(ii) one CNS hemangioblastoma + visceral manifestations of VHL, and
(iii) any manifestation + known family history of VHL.

The most common causes of death in VHL are renal cell carcinoma and neurologic
complications of cerebellar hemangioblastomas. Genetic counselling and screening are
important for early detection and treatment of VHL lesions.

Von-Hippel-Lindau Syndrome [STATdx]

  • Autosomal Dominant
  • Comprises of
    o Hemangioblastomas – arise from and so abuts the pia surface
    ▪ Cerebellum (posterior half)
    ▪ Brainstem (posterior medulla)
    ▪ Spinal Cord (posterior half)
    o Ocular angioma – retinal detachment and haemorrhage
    o Endolymphatic sac tumours (ELSTs) – large temporal bone mass posterior to
    internal auditory canal near vestibular duct.
    o Clear cell RCC
    o Cystadenomas – Pancreas
    o Pheochromocytomas
269
Q

A one-year-old boy is admitted unwell with generalised abdominal tenderness and guarding.
A supine plain abdominal film is requested, which shows a large oval radiolucency in the middle of the abdomen, with a well-defined linear opacity in the right upper quadrant. Which
one of the following conditions would best explain these appearances? [B2 Q25]

a. Enteric duplication cyst
b. Choledochal cyst
c. Pneumoperitoneum
d. Duodenal atresia
e. Caecal volvulus

A

Pneumoperitoneum

Whilst uncommon, the appearances seen on plain film are consistent with massive pneumoperitoneum. The oval radiolucency is called the ‘football sign’ and arises due to free
air collecting anterior to the intra-abdominal viscera. This sign is only seen in 2% of adults
with pneumoperitoneum due to the large quantities of air required to produce it. It is much
more common in infants who may present at a later stage. The opacity in the right upper
quadrant is produced by air outlining the falciform ligament, which again is a sign of
pneumoperitoneum. Causes of perforation in this age group include trauma, intussusception,
and complications of Meckel’s diverticulum.

270
Q

A 46-year-old man presents with severe abdominal pain. An erect chest radiograph shows free
intraperitoneal air below the diaphragm. What is the most likely cause? [B4 Q27]

a. perforated anterior wall duodenal ulcer
b. perforated posterior wall duodenal ulcer
c. perforated gastric ulcer
d. perforated appendix
e. diverticulitis with perforation

A

Perforation anterior wall duodenal ulcer

The commonest cause of a free intraperitoneal perforation is an anterior wall duodenal ulcer.
However, free peritoneal air is only apparent on an erect chest radiograph in 60% of perforated
duodenal ulcers. Possible causes include sealing of the perforation, adhesions preventing the
gas reaching the subphrenic space or insufficient time being allowed for gas to collect under
the diaphragm. Anterior wall gastric ulcers also perforate into the peritoneal cavity but are a
less common cause of pneumoperitoneum. Posterior wall duodenal and gastric ulcers perforate
into the lesser sac or retroperitoneal region rather than the peritoneal cavity. Perforated
appendix and diverticulitis may produce localized collections of extraluminal gas, but
pneumoperitoneum is rare.

271
Q

A 74-year-old man presents with severe abdominal pain and is admitted under the surgical
team with suspected perforation. He is too unwell to undergo an erect chest radiograph. What
is the most appropriate alternative plain film to detect the presence of free intraperitoneal gas?
[B4 Q41]

a. supine chest
b. erect abdomen
c. supine abdomen
d. left lateral decubitus abdomen e. right lateral decubitus abdomen

A

Left lateral decubitus

An erect chest radiograph is best for demonstrating a small pneumoperitoneum, enabling the
identification of as little as 1ml of free intraperitoneal gas. It is superior to an erect abdomen,
as the X-ray beam penetrates the diaphragmatic region almost tangentially, whereas, in an erect
abdomen, the divergent beam penetrates this area obliquely. However, if the patient is too
unwell to sit erect, the most appropriate projection is a left lateral decubitus abdominal
radiograph, performed with the patient lying on the left side, using a horizontal X-ray beam. In
this position, air will preferentially leave a perforated duodenal or antral ulcer, and any free gas
in the lesser sac will enter the main abdominal cavity. The supine abdominal radiograph may
demonstrate free gas in about 56% of patients with pneumoperitoneum. Characteristic features
include gas outlining both inner and outer walls of a bowel loop (Rigler’s sign), triangular
collections of gas between bowel loops, and outlining of the falciform ligament and other
peritoneal reflections by free gas. The supine chest radiograph is not useful in the detection of
free intraperitoneal gas.

272
Q

An 81-year-old man is investigated for anaemia of unknown cause. He has a barium enema as
an outpatient that is reported as normal but is subsequently admitted with a large gastro-
intestinal bleed. Initial upper gastro-intestinal endoscopy is normal. He is haemodynamically
unstable and therefore has a mesenteric angiogram, which shows early opacification and slow
emptying of the ileocolic vein. Which one of the following diagnoses is most likely? [B2 Q33]

a. Angiodysplasia
b. Diverticulosis
c. Meckel’s diverticulum
d. Adenomatous polyp
e. Radiation enteritis

A

Angiodysplasia

The typical angiographic feature of a Meckel’s diverticulum is presence of the vitelline artery.
Meckel’s diverticulum also typically (although not exclusively) presents in younger patients.
With diverticulosis, radiation enteritis and polyps, one might expect an abnormal barium enema,
and in addition these angiographic features are not typical. Angiodysplasia is the second most
common cause of gastro-intestinal bleed in the elderly population after diverticular disease. It
is due to dilatation of the submucosal vessels and occurs most commonly on the right side of
the colon. Angiographic features include visualisation of a cluster of vessels on the
antimesenteric border, early filling of the ileocolic vein in the arterial phase and delayed
emptying of the same vein.

273
Q

A 37-year-old female has a pelvic MRI for investigation of rectal pain and bleeding, following
a normal flexible sigmoidoscopy. This shows a thin-walled dumbbell-shaped 7cm multilocular
cyst. It is in contact anteriorly to the rectum and posteriorly to the presacral fascia, but
contained within the mesorectal fascia. Although the rectum is distorted by the mass, the
rectum and sigmoid are normal. What is the most likely diagnosis? [B2 Q34]

a. Rectal duplication cyst
b. Anterior sacral meningocoele c. Mucinous rectal carcinoma
d. Presacral dermoid
e. Tailgut cyst

A

Tailgut cyst
Tailgut cysts or cystic hamartomas are presacral, multilocular, mucous-secreting cysts found
distal to the normal embryonic termination of the hindgut. Small cysts may be asymptomatic,
but larger cysts may present with rectal pain or bleeding, constipation, anal fistulae or
recurrent rectal abscesses. A long and tail-like coccyx is often associated and can help
distinguish between many other cystic lesions in this region. Malignant transformation is a
complication, most commonly adenocarcinoma. Duplication cysts are most often unilocular
unless complicated by haemorrhage or infection, a meningocele is likely to arise from the sacral
foramina, piercing the presacral fascia, and dermoid tumour would be expected to contain fat
or layering of contents as seen in dermoid tumours in other locations.

274
Q

A 50-year-old female presents to A&E with acute abdominal pain. On examination there is
point tenderness over an area in the right iliac fossa. CT reveals a well-defined triangular area
of high-attenuation fat density anteriorly in the lower right abdomen. The large and small bowel
are normal. Which one of the following is most likely? [B2 Q40]

a. Segmental omental infarction
b. Rectus haematoma
c. Epiploic appendagitis
d. Carcinoid tumour
e. Mesenteric vein thrombosis

A

Segmental omental infarction
Segmental omental infarction is the most likely cause and most commonly affects the right half
of the greater omentum. It mimics surgical pathology such as appendicitis. High attenuation
streaks in the omental fat with apparent ‘mass effect’ in the absence of any other findings is
suggestive of the diagnosis. Point tenderness over the specific area of CT abnormality is often
discovered. Management is conservative.

275
Q

A duodenal diverticulum is noted on CT as an incidental finding. Which is the single best
answer? [B3 Q28]

A. Reported in up to 5% of upper Gastrointestinal (GI) fluoroscopic studies
B. Commonly symptomatic with high risk of inflammation
C. Mostly located on the lateral wall of the duodenum
D. Mostly arise from the third or fourth part of the duodenum
E. If perforated is easy to distinguish from a perforated duodenal ulcer

A

Reported in up to 5% of upper GI fluoroscopic studies
Common acquired outpouchings of the mucosa and submucosa located along the medial wall
of the duodenum within 2cm of the ampulla of Vater. 25% arise from D3/4. Reported in 1-5%
of upper GI fluoroscopic studies.

276
Q

Which of the following is a cause of smooth, regular, and thickened folds in non-dilated small
bowel? [B3 Q36]

A. Intraluminal haematoma
B. Crohn’s
C. Zollinger-Ellison
D. Tuberculosis
E. Carcinoid

A

B-D are cases of irregular and thickened distal folds in nondilated small bowel

277
Q

A child with a subacute history of vague abdominal pain is being investigated with ultrasound.
Inflammatory markers are normal and there is no clinical indication of an appendicitis. During
the study, an abnormality is detected in the right iliac fossa. Which of the following
sonographic features favours a duplication cyst rather than a Meckel’s diverticulum? [B3 Q47]

A. A double layered wall
B. Non-compressibility
C. Teardrop shape
D. A greater degree of irregular mucosa
E. An elongated tubular shape

A

A double layered wall

An inflamed Meckel’s diverticulum can be difficult to distinguish from a duplication cyst in
the terminal ileum. However, a characteristic double-layered wall is seen in duplication cysts,
whereas the other features are more often found in an inflamed Meckel’s diverticulum.

278
Q

A 70-year-old man presents with fresh bleeding per rectum. He undergoes resuscitation,
receiving 5 units of blood over the following 24 hours. Colonoscopy is unsuccessful in
detecting the source of the bleeding, and he continues to pass fresh blood, although he remains
haemodynamically stable. What is the most appropriate next investigation? [B4 Q18]

a. 99mTc-labelled red blood cell radionuclide imaging
b. CT angiography
c. repeat colonoscopy
d. digital subtraction mesenteric angiography
e. abdominal ultrasound scan

A

CT angiography

Several imaging methods are available for use in those patients in whom endoscopy fails to
detect the source of bleeding in gastrointestinal haemorrhage. Radionuclide imaging is non-
invasive and very sensitive, detecting bleeding rates as low as 0.1–0.5ml/min. Images are
acquired over several hours, enabling detection of intermittent and venous bleeding, but
anatomical localization can be insensitive and variable. Conventional angiography is invasive
and requires active bleeding at the time of both imaging and contrast injection. Higher bleeding
rates of 0.5–1ml/min are required, and motion artefact from bowel peristalsis may be
problematic, but it provides superior localization of the bleeding site and options for therapeutic
intervention. However, CT angiography is advocated as the most appropriate investigation, due to its wide availability, minimal invasiveness, and high sensitivity, detecting bleeding rates as
low as 0.3ml/min in animal models. In addition, it enables assessment of a pathological lesion
causing bleeding, which may be helpful in planning further management.

279
Q

Which of the following may improve the detection of Meckel’s diverticulum on a [99mTc]
pertechnetate study? [B4 Q54]

a. prior administration of cimetidine
b. prior administration of laxatives
c. prior administration of potassium perchlorate
d. maintenance of a full bladder
e. barium follow-through prior to study

A

Prior administration of Cimetidine
Cimetidine, a histamine H2 -receptor antagonist, may be used to increase uptake of [ 99m Tc]
pertechnetate by inhibiting its secretion from gastric mucosa. Pentagastrin and glucagon have
also been used to improve visualization, by stimulating uptake and decreasing peristalsis
respectively. Procedures such as colonoscopy and use of laxatives should be avoided prior to
the scan, as they may cause mucosal irritation. Potassium perchlorate should not be used to
block thyroid uptake, as it also blocks uptake of pertechnetate by the gastric mucosa. The
patient normally fasts for 3–4 hours and voids prior to the study, as a full stomach or bladder
may obscure an adjacent Meckel’s diverticulum. Barium studies should be avoided for 3–4
days prior to the study, as attenuation by the barium may hamper interpretation

280
Q

Which of the following best describes the intravenous iodinated contrast agent iodixanol
(Visipaque)? [B4 Q86]

a. ionic, high-osmolar, monomeric
b. non-ionic, iso-osmolar, dimeric
c. ionic, low-osmolar, dimeric
d. non-ionic, low-osmolar, monomeric
e. ionic, high-osmolar, dimeric

A

Non-ionic, iso-osmolar, dimeric

Modern water-soluble, iodinated, intravenous contrast media are based on the six-carbon ring
structure tri-iodobenzoic acid. Earlier contrast agents were ionic and high-osmolar, but the
toxicity of these agents prompted further development. To decrease osmolality while
maintaining an acceptable iodine concentration, the ratio between the number of iodine atoms
and the number of particles in solution has been decreased either by combining two tri-
iodinated benzene rings or by producing compounds that do not ionize in solution, or, more
recently, by both methods. The most recent agents such as iodixanol (Visipaque) and iotrolan
(Isovist) are non-ionic dimers with six iodine atoms per molecule in solution, enabling
satisfactory iodine concentrations to be obtained at iso-osmolality.

281
Q

A patient undergoes 111In-labelled white blood cell scintigraphy for investigation of suspected
occult sepsis. Which of these would be regarded as abnormal on imaging at 4 hours? [B4 Q87]

a. uptake in the large bowel b. splenic uptake greater than that of the liver
c. uptake in the bone marrow
d. diffuse uptake in the lungs
e. uptake in the thymus in children

A

Uptake in large bowel

Radiolabelled white cell imaging is used for detection of infection and inflammation. Images
reflect the distribution of white blood cells within the body and localize areas of infection or
inflammation. Imaging is usually performed at 18–24 hours, by which time blood pool activity
is normally no longer present, and the most intense uptake is seen in the spleen, followed by
the liver and then the bone marrow. Imaging is also usually performed at 2–6 hours for
investigation of suspected inflammatory bowel disease, as sloughed inflamed cells may move
distally and provide misleading information as to the affected site if only imaged at 24 hours.
Physiological diffuse lung uptake may be seen in the first 4 hours due to cellular activation
from in vitro cell manipulation, but normally decreases after this. Thymus activity may be seen
normally in children. Bowel and genitourinary activity are not normally seen, and
gastrointestinal activity is always abnormal. In general, focal activity outside the normal white
cell distribution, which is greater than that of the spleen, suggests the presence of an abscess.
Activity equal to that of the liver indicates a significant inflammatory focus. Activity less than
that of the bone marrow suggests a low-level inflammatory response.

282
Q

In the assessment of tumour response to treatment, what method of tumour measurement is
used in the RECIST (Response Criteria in Solid Tumours) criteria? [B4 Q88]

a. unidimensional (long axis dimension)
b. unidimensional (short axis dimension)
c. bidimensional (product of longest diameter and greatest perpendicular diameter)
d. bidimensional (product of longest diameter and shortest diameter)
e. volumetric

A

Unidimensional (long axis dimension)
The WHO response criteria were devised in 1981 to standardize the criteria used for measuring
therapeutic response in cancer patients. These criteria set out definitions of complete response,
partial response, no change, and progressive disease, based upon bidimensional measurements
of tumour lesions in the axial plane. The product of the longest diameter multiplied by the
greatest perpendicular diameter is calculated for each measurable lesion, and the sum of these
products is used to determine treatment response. The RECIST criteria were introduced in 2000
and were designed to be used in clinical trials. The criteria involve classifying the disease
burden into measurable and non-measurable disease, followed by selection of up to 10
representative target lesions. The sum of the long axis dimension of the target lesions in the
axial plane is used to determine the final response category. Potential concerns about the use
of the RECIST criteria include possible confusion arising when lymph node measurements are
performed using the short axis dimension and the increased workload involved for the
radiologist.

283
Q

For a standard portal-phase spiral CT of the abdomen and pelvis, after what period following
commencement of the intravenous contrast injection should image acquisition begin? [B4 Q95]
a. 0 seconds
b. 10–15 seconds
c. 25–30 seconds
d. 65–70 seconds
e. 120–180 seconds

A

65-70 seconds

For a standard, single-slice, spiral CT of the abdomen and pelvis in a single breathhold, 100–
150ml of 300mg/ml iodine intravenous contrast medium should be administered at 3–4ml/s.
For a portal phase examination, image acquisition should begin 65–70 s after commencement
of the injection.

Fundoplication

Procedure: the fundus of the stomach is wrapped posteriorly (or less commonly anteriorly)
around the distal oesophagus and lower oesophageal sphincter.

Aim: to augment the lower oesophageal sphincter, providing compression to minimize reflux

Types:
1. Nissen (360 posterior wrap)
2. Toupet (270 posterior wrap)
3. Dor (180 anterior wrap)

To check on CT: the wrap should be below the diaphragmatic hiatus.

Complications:

  1. High grade oesophageal hold up – may need endoscopic balloon dilatation.
  2. Wrap migration/ Slippage – distally to wrap the stomach, due to failure of sutures
    holding the wrap to the oesophagus. Sudden post-operative deterioration. Very dilated
    fundal bubble. Requires corrective surgery.
  3. Intra-thoracic wrap herniation – up to 30% at 1 year, not always symptomatic, 70%
    para-oesophageal type (normal GOJ position) and 30% sliding type (proximal
    migration of GOJ)
  4. Fundoplication dehiscence – no evidence of surgery found
    Surgery for Hiatus hernia
  5. Collis gastroplasty – use stapled cardia to form distal oesophagus, use if there’s
    oesophageal shortening seen in Ba swallow.
  6. Normal hernia repair using mesh.
  7. Correction of volvulus if present.